childbearing FINAL practice questions

Lakukan tugas rumah & ujian kamu dengan baik sekarang menggunakan Quizwiz!

22. Which documentation on a womans chart on postpartum day 14 indicates a normal involution process? a. Moderate bright red lochial flow b. Breasts firm and tender c. Fundus below the symphysis and nonpalpable d. Episiotomy slightly red and puffy

C (The fundus descends 1 cm per day; consequently, it is no longer palpable by postpartum day 14. The lochia should be changed by this day to serosa. Breasts are not part of the involution process. The episiotomy should not be red or puffy at this stage.)

22. In which situation would the nurse be called on to stimulate the fetal scalp? a. As part of fetal scalp blood sampling b. In response to tocolysis c. In preparation for fetal oxygen saturation monitoring d. To elicit an acceleration in the FHR

D (The scalp can be stimulated using digital pressure during a vaginal examination. Fetal scalp blood sampling involves swabbing the scalp with disinfectant before a sample is collected. The nurse stimulates the fetal scalp to elicit an acceleration of the FHR. Tocolysis is relaxation of the uterus. Fetal oxygen saturation monitoring involves the insertion of a sensor.)

The musculoskeletal system adapts to the changes that occur during pregnancy. A woman can expect to experience what change? a.Her center of gravity will shift backward. b.She will have increased lordosis. c.She will have increased abdominal muscle tone. d.She will notice decreased mobility of her pelvic joints.

b. She will have increased lordosis.

A nurse caring for a pregnant client must understand that the hormone essential for maintaining pregnancy is: a.Estrogen b.Human chorionic gonadotropin (hCG) c.Oxytocin d.Progesteron

d.Progesteron

23. A client at 39 weeks of gestation has been admitted for an external version. Which intervention would the nurse anticipate the provider to order? a. Tocolytic drug b. Contraction stress test (CST) c. Local anesthetic d. Foley catheter

ANS: A A tocolytic drug will relax the uterus before and during the version, thus making manipulation easier. CST is used to determine the fetal response to stress. A local anesthetic is not used with external version. Although the bladder should be emptied, catheterization is not necessary. DIF: Cognitive Level: Apply REF: p. 779 TOP: Nursing Process: Planning MSC: Client Needs: Physiologic Integrity

Which is the most common technique used for the termination of a pregnancy in the second trimester? a. Dilation and evacuation (D&E) b. Methotrexate administration c. Prostaglandin administration d. Vacuum aspiration

ANS: A D&E can be performed at any point up to 20 weeks of gestation. It is more commonly performed between 13 and 16 weeks of gestation. Methotrexate is a cytotoxic drug that causes early abortion by preventing fetal cell division. Prostaglandins are also used for early abortion and work by dilating the cervix and initiating uterine wall contractions. Vacuum aspiration is used for abortions in the first trimester.

5. The multiple marker test is used to assess the fetus for which condition? a. Down syndrome c. Congenital cardiac abnormality b. Diaphragmatic hernia d. Anencephaly

ANS: A The maternal serum level of alpha-fetoprotein is used to screen for Down syndrome, neural tube defects, and other chromosome anomalies. The multiple marker test would not detect diaphragmatic hernia, congenital cardiac abnormality, or anencephaly. Additional testing, such as ultrasonography and amniocentesis, would be required to diagnose these conditions

Which laboratory testing is used to detect the human immunodeficiency virus (HIV)? a.HIV screening b.HIV antibody testing c.Cluster of differentiation 4 (CD4) counts d.Cluster of differentiation 8 (CD8) counts

ANS: B The screening tool used to detect HIV is the enzyme immunoassay, which tests for the presence of antibodies to the HIV. HIV-1 and HIV-2 antibody tests are used to confirm the diagnosis. To determine whether the HIV is present, the test performed must be able to detect antibodies to the virus, not the virus itself. CD4 counts are associated with the incidence of acquired immunodeficiency syndrome (AIDS) in HIV-infected individuals. CD8 counts are not performed to detect HIV.

Which menopausal discomfort would the nurse anticipate when evaluating a woman for signs and symptoms of the climacteric? a. Headaches b. Hot flashes c. Mood swings d. Vaginal dryness with dyspareunia

ANS: B Vasomotor instability, in the form of hot flashes or flushing, is a result of fluctuating estrogen levels and is the most common disturbance of the perimenopausal woman. Headaches may be associated with a decline in hormonal levels; however, headaches are not the most frequently reported discomfort for menopausal women. Mood swings may also be associated with a decline in hormonal levels; however, mood swings are not the most frequently reported discomfort for menopausal women. Vaginal dryness and dyspareunia may be associated with a decline in hormonal levels; however, both are not the most frequently reported discomforts for menopausal women.

16. What type of cultural concern is the most likely deterrent to many women seeking prenatal care? a. Religion c. Ignorance b. Modesty d. Belief that physicians are evil

ANS: B A concern for modesty is a deterrent to many women seeking prenatal care. For some women, exposing body parts, especially to a man, is considered a major violation of their modesty. Many cultural variations are found in prenatal care. Even if the prenatal care described is familiar to a woman, some practices may conflict with the beliefs and practices of a subculture group to which she belongs.

Which contraceptive method should the nurse identify as protecting against sexually transmitted infections (STIs) and the human immunodeficiency virus (HIV)? a. Periodic abstinence b. Barrier methods c. Hormonal methods d. Same protection with all methods

ANS: B Barrier methods, such as condoms, protect against STIs and the HIV the best of all contraceptive methods. Periodic abstinence and hormonal methods, such as birth control pills, offer no protection against STIs or the HIV.

30. Which statement about multifetal pregnancy is inaccurate? a. The expectant mother often develops anemia because the fetuses have a greater demand for iron. b. Twin pregnancies come to term with the same frequency as single pregnancies. c. The mother should be counseled to increase her nutritional intake and gain more weight. d. Backache and varicose veins often are more pronounced.

ANS: B Twin pregnancies often end in prematurity. Serious efforts should be made to bring the pregnancy to term. A woman with a multifetal pregnancy often develops anemia, suffers more or worse backache, and needs to gain more weight. Counseling is needed to help her adjust to these conditions.

oligohydramnios

Abnormally small amount of amniotic fluid, less than about 500 mL at term.

Probable signs of pregnancy are: a.Determined by ultrasound b.Observed by the health care provider c.Reported by the client d.Diagnostic tests

b.Observed by the health care provider

Which minerals and vitamins usually are recommended to supplement a pregnant woman's diet? a.Fat-soluble vitamins A and D b.Water-soluble vitamins C and B6 c.Iron and folate d.Calcium and zinc

c. Iron and folate

While taking a diet history the nurse might be told that the expectant mother has cravings for ice chips, cornstarch, and baking soda. This represents a nutritional problem known as: a.Preeclampsia b.Pyrosis c.Pica d.Purging

c. Pica

In order to reassure and educate pregnant clients about changes in their breasts, nurses should be aware that: a.The visibility of blood vessels that form an intertwining blue network indicates full function of Montgomery's tubercles and possibly infection of the tubercles b.The mammary glands do not develop until 2 weeks before labor c.Lactation is inhibited until the estrogen level declines after birth d.Colostrum is the yellowish oily substance used to lubricate the nipples for breastfeeding

c.Lactation is inhibited until the estrogen level declines after birth

A nurse providing care to a pregnant woman should know that all are normal gastrointestinal changes in pregnancy except: a.Ptyalism b.Pyrosis c.Pica d.Decreased peristalsis

c.Pica

27. The Valsalva maneuver can be described as the process of making a forceful bearing-down attempt while holding ones breath with a closed glottis and a tightening of the abdominal muscles. When is it appropriate to instruct the client to use this maneuver? a. During the second stage to enhance the movement of the fetus b. During the third stage to help expel the placenta c. During the fourth stage to expel blood clots d. Not at all

d (The client should not be instructed to use this maneuver. This process stimulates the parasympathetic division of the autonomic nervous system and produces a vagal response (decrease in heart rate and blood pressure.) An alternative method includes instructing the client to perform open-mouth and open-glottis breathing and pushing.)

Which vitamins or minerals can lead to congenital malformations of the fetus if taken in excess by the mother? a.Zinc b.Vitamin D c.Folic acid d.Vitamin A

d. Vitamin A

While completing an assessment of a homeless woman, the nurse should be aware of which of the following ailments this client is at a higher risk to develop? (Select all that apply.) a. Infectious diseases b. Chronic illness c. Anemia d. Hyperthermia e. Substance abuse

A B C E

A 25-year-old woman is seeing her gynecologist for apreconception visit. She has been married for 3 years.Her brother has hemophilia. Which of these genetic tests would be recommended for this woman? A. Carrier testing B. Predictive testing C. Preimplantation testing D. Prenatal testing

A. Carrier testing

True or false: Meiosis is a process of two successive cell divisions that produces two haploid cells. A. True B. False

A. True

2. The volume of amniotic fluid is an important factor in assessing fetal well-being. Oligohydramnios (an amniotic fluid volume of less than 300 ml) is associated with what kind of fetal anomalies? a. Renal b. Cardiac c. Gastrointestinal d. Neurologic

ANS: A An amniotic fluid volume of less than 300 ml (oligohydramnios) is associated with fetal renal anomalies. The amniotic fluid volume has no bearing on the fetal cardiovascular system. Gastrointestinal anomalies are associated with hydramnios, or an amniotic fluid volume greater than 2 L. The amniotic fluid volume has no bearing on the fetal neurologic system.

9. A nurse is discussing the storage of breast milk with a mother whose infant is preterm and in the special care nursery. Which statement indicates that the mother requires additional teaching? a. I can store my breast milk in the refrigerator for 3 months. b. I can store my breast milk in the freezer for 3 months. c. I can store my breast milk at room temperature for 4 hours. d. I can store my breast milk in the refrigerator for 3 to 5 days.

ANS: A Breast milk for the hospitalized infant can be stored in the refrigerator for only 8 days, not for 3 months. Breast milk can be stored in the freezer for 3 months, in a deep freezer for 6 months, or at room temperature for 4 hours. Human milk for the healthy or preterm hospitalized infant can be kept in the refrigerator for up to 8 days or in the freezer for up to 3 months, but only for 4 hours or less at room temperature.

A client exhibits a thick, white, lumpy, cottage cheese-like discharge, along with white patches on her labia and in her vagina. She complains of intense pruritus. Which medication should the nurse practitioner order to treat this condition? a.Fluconazole b.Tetracycline c.Clindamycin d.Acyclovir

ANS: A The client is experiencing a candidiasis infection. Fluconazole, metronidazole, and clotrimazole are the drugs of choice to treat this condition. Tetracycline is used to treat syphilis. Clindamycin is used to treat bacterial vaginosis. Acyclovir is used to treat genital herpes.

Which risk factors are associated with NEC (Necrotizing enterocolitis)? (Select all that apply.) a.Polycythemia b.Anemia c.Congenital heart disease d.Bronchopulmonary dysphasia e.Retinopathy

ANS: A, B, C Risk factors for NEC include asphyxia, RDS, umbilical artery catheterization, exchange transfusion, early enteral feedings, patent ductus arteriosus (PDA), congenital heart disease, polycythemia, anemia, shock, and gastrointestinal infection. Bronchopulmonary dysphasia and retinopathy are not associated with NEC

Counseling and education are critical components of the nursing care of women with herpes infections. Clients should be taught to identify triggers that might result in a herpes attack. Which factors are possible triggers for a recurrence? (Select all that apply.) a.Menstruation b.Trauma c.Febrile illness d.Soap e.Ultraviolet light

ANS: A, B, C, E Stress, menstruation, trauma, febrile illness, chronic illnesses, and ultraviolet light have all been found to trigger genital herpes. Women might elect to keep a diary of symptoms to help identify stressors. Lesions should be cleansed with saline or simple soap and water. Lesions can be kept dry by using a blow dryer, wearing cotton underwear, and wearing loose clothing. Tea bags or hydrogen peroxide might also be helpful.

19. The nurse is explaining the benefits associated with breastfeeding to a new mother. Which statement by the nurse would provide conflicting information to the client? a. Women who breastfeed have a decreased risk of breast cancer. b. Breastfeeding is an effective method of birth control. c. Breastfeeding increases bone density. d. Breastfeeding may enhance postpartum weight loss.

ANS: B Although breastfeeding delays the return of fertility, it is not an effective birth control method. Women who breastfeed have a decreased risk of breast cancer, an increase in bone density, and a possibility of faster postpartum weight loss.

12. Which procedure falls into the category of micromanipulation techniques of the follicle? (Select all that apply.) a. Intrauterine insemination b. Preimplantation genetic diagnosis c. Intracytoplasmic sperm injection (ISCI) d. Assisted hatching e. IVF-ET

ANS: C, D ISCI makes it possible to achieve fertilization even with a few or poor-quality sperm by introducing sperm beneath the zone pellucid into the egg. Another micromanipulation technique is assisted hatching. An infrared laser breaks through the thick or tough zone pellucid, enabling the blastocyst to hatch.

3. A postpartum woman telephones the provider regarding her 5-day-old infant. The client is not scheduled for another weight check until the infant is 14 days old. The new mother is worried about whether breastfeeding is going well. Which statement indicates that breastfeeding is effective for meeting the infants nutritional needs? a. Sleeps for 6 hours at a time between feedings b. Has at least one breast milk stool every 24 hours c. Gains 1 to 2 ounces per week d. Has at least six to eight wet diapers per day

ANS: D After day 4, when the mothers milk comes in, the infant should have six to eight wet diapers every 24 hours. Typically, infants sleep 2 to 4 hours between feedings, depending on whether they are being fed on a 2- to 3-hour schedule or cluster-fed. The infants sleep pattern is not an indication whether the infant is breastfeeding well. The infant should have a minimum of three bowel movements in a 24-hour period. Breastfed infants typically gain 15 to 30 g/day.

12. The most basic information a maternity nurse should have concerning conception is: a. Ova are considered fertile 48 to 72 hours after ovulation b. Sperm remain viable in the woman's reproductive system for an average of 12 to 24 hours c. Conception is achieved when a sperm successfully penetrates the membrane surrounding the ovum d. Implantation in the endometrium occurs 6 to 10 days after conception

ANS: D After implantation, the endometrium is called the decidua. Ova are considered fertile for about 24 hours after ovulation. Sperm remain viable in the woman's reproductive system for an average of 2 to 3 days. Penetration of the ovum by the sperm is called fertilization. Conception occurs when the zygote, the first cell of the new individual, is formed.

Which clinical findings would alert the nurse that the neonate is expressing pain? a.Low-pitched crying; tachycardia; eyelids open wide b.Cry face; flaccid limbs; closed mouth c.High-pitched, shrill cry; withdrawal; change in heart rate d.Cry face; eyes squeezed; increase in blood pressure

ANS: D Crying and an increased heart rate are manifestations indicative of pain in the neonate. Typically, infants tightly close their eyes when in pain, not open them wide. In addition, infants may display a rigid posture with the mouth open and may also withdraw limbs and become tachycardic with pain. A high-pitched, shrill cry is associated with genetic or neurologic anomalies.

Which statement about the development of cultural competence is inaccurate? a. Local health care workers and community advocates can help extend health care to underserved populations. b. Nursing care is delivered in the context of the clients culture but not in the context of the nurses culture. c. Nurses must develop an awareness of and a sensitivity to various cultures. d. Cultures economic, religious, and political structures influence practices that affect childbearing.

B

A nurse is explaining the process of the development of sperm to a couple who are having trouble conceiving and who are being tested for possible infertility. Which of the following should the nurse mention as the site of spermatogenesis, or the production of sperm? 1. Seminiferous tubules 2. Seminal vesicles 3. Prostate gland 4. Bulbourethral glands

1. Seminiferous tubules Spermatogenesis takes place in the seminiferous tubules within the testes. Seminal vesicles produce secretions that contain fructose, which is an energy source for sperm, but not the sperm themselves. The prostate gland secretes an alkaline fluid that enhances sperm mobility, but does not produce the sperm themselves. The bulbourethral glands, also referred to as Cowper's glands, secrete an alkaline solution that coats the interior of the urethra to neutralize the acidic urine that is present. They do not produce the sperm.

The nurse asks a patient when her last menstrual period began. The patient replies that it began about 10 days ago and ended about 5 days ago. The nurse should recognize that this patient is currently in which phase of her ovarian cycle? 1. Ovulatory 2. Luteal 3. Follicular 4. Secretory

3. Follicular The ovulatory phase of the ovarian cycle follows the follicular phase and begins about 12 to 14 days after the first day of menstruation, when estrogen levels peak, and ends with the release of the oocyte from the mature graafian follicle. The luteal phase of the ovarian cycle begins after ovulation and lasts approximately 14 days. The follicular phase of the ovarian cycle begins on the first day of menstruation and lasts 12 to 14 days. Thus, this patient is currently in the follicular phase. The secretory phase occurs in the endometrial cycle, not the ovarian cycle, and begins after ovulation and ends with the onset of menstruation.

An Apgar score of 10 at 1 minute after birth indicates what? 1. An infant having no difficulty adjusting to extrauterine life and needing no further testing 2. An infant in severe distress that needs resuscitation 3. A prediction of a future free of neurologic problems 4. An infant having no difficulty adjusting to extrauterine life but who should be assessed again at 5 minutes after birth

4. An infant having no difficulty adjusting to extrauterine life but who should be assessed again at 5 minutes after birth

28. What is the correct placement of the tocotransducer for effective EFM? a. Over the uterine fundus b. On the fetal scalp c. Inside the uterus d. Over the mothers lower abdomen

A (The tocotransducer monitors uterine activity and should be placed over the fundus, where the most intensive uterine contractions occur. The tocotransducer is for external use.)

6. A nurse caring for a laboring woman should know that meconium is produced by: a. Fetal intestines b. Fetal kidneys c. Amniotic fluid d. The placenta

A As the fetus nears term, fetal waste products accumulate in the intestines as dark green to black, tarry meconium. Meconium is not produced by the fetal kidneys. Meconium should not be present in amniotic fluid. This may be an indication of fetal compromise. The placenta does not produce meconium.

10. After delivery, excess hypertrophied tissue in the uterus undergoes a period of self-destruction. What is the correct term for this process? a. Autolysis b. Subinvolution c. Afterpains d. Diastasis

A (Autolysis is caused by a decrease in hormone levels. Subinvolution is failure of the uterus to return to a nonpregnant state. Afterpains are caused by uterine cramps 2 to 3 days after birth. Diastasis refers to the separation of muscles.)

a

A perinatal nurse is caring for a woman in the immediate postbirth period. Assessment reveals that the client is experiencing profuse bleeding. What is the most likely cause for this bleeding? a. Uterine atony b. Uterine inversion c. Vaginal hematoma d. Vaginal laceration

Dysfunctional uterine bleeding (DUB) is defined as excessive uterine bleeding without a demonstrable cause. Which statement regarding this condition is most accurate? a. DUB is most commonly caused by anovulation. b. DUB most often occurs in middle age. c. The diagnosis of DUB should be the first consideration for abnormal menstrual bleeding. d. Steroids are the most effective medical treatment for DUB.

ANS: A Anovulation may occur because of hypothalamic dysfunction or polycystic ovary syndrome. DUB most often occurs when the menstrual cycle is being established or when it draws to a close at menopause. A diagnosis of DUB is made only after all other causes of abnormal menstrual bleeding have been ruled out. The most effective medical treatment is oral or intravenous estrogen.

Clients treated for syphilis with penicillin may experience a Jarisch-Herxheimer reaction. Which clinical presentation would be unlikely if a client is experiencing this reaction? a.Vomiting and diarrhea b.Headache, myalgias, and arthralgia c.Preterm labor d.Jarisch-Herxheimer in the first 24 hours after treatment

ANS: A The Jarisch-Herxheimer reaction is an acute febrile reaction that occurs within the first 24 hours of treatment and is accompanied by headache, myalgias, and arthralgia. Vomiting and diarrhea are not anticipated. If the client is pregnant, then she is at risk for preterm labor and birth.

3. Which physiologic factors are reliable indicators of impending shock from postpartum hemorrhage? (Select all that apply.) a. Respirations b. Skin condition c. Blood pressure d. Level of consciousness e. Urinary output

ANS: A, B, D, E Blood pressure is not a reliable indicator; several more sensitive signs are available. Blood pressure does not drop until 30% to 40% of blood volume is lost. Respirations, pulse, skin condition, urinary output, and level of consciousness are more sensitive means of identifying hypovolemic shock.

Infants born between 34 0/7 and 36 6/7 weeks of gestation are called late-preterm infants because they have many needs similar to those of preterm infants. Because they are more stable than early-preterm infants, they may receive care that is similar to that of a full-term baby. These infants are at increased risk for which conditions? (Select all that apply.) a.Problems with thermoregulation b.Cardiac distress c.Hyperbilirubinemia d.Sepsis e.Hyperglycemia

ANS: A, C, D Thermoregulation problems, hyperbilirubinemia, and sepsis are all conditions related to immaturity and warrant close observation. After discharge, the infant is at risk for rehospitalization related to these problems. Association of Women's Health, Obstetric and Neonatal Nurses (AWHONN) launched the Near-Term Infant Initiative to study the problem and ways to ensure that these infants receive adequate care. The nurse should ensure that this infant is adequately feeding before discharge and that parents are taught the signs and symptoms of these complications. Late-preterm infants are also at increased risk for respiratory distress and hypoglycemia.

4. Which statements concerning the benefits or limitations of breastfeeding are accurate? (Select all that apply.) a. Breast milk changes over time to meet the changing needs as infants grow. b. Breastfeeding increases the risk of childhood obesity. c. Breast milk and breastfeeding may enhance cognitive development. d. Long-term studies have shown that the benefits of breast milk continue after the infant is weaned. e. Benefits to the infant include a reduced incidence of SIDS

ANS: A, C, D, E Breastfeeding actually decreases the risk of childhood obesity. Human milk is the perfect food for human infants. Breast milk changes over time to meet the demands of the growing infant. Scientific evidence is clear that human milk provides the best nutrients for infants with continued benefits long after weaning. Fatty acids in breast milk promote brain growth and development and may lead to enhanced cognition. Infants who are breastfed experience a reduced incidence of SIDS.

Which medications can be taken by postmenopausal women to treat and/or prevent osteoporosis? (Select all that apply.) a. Calcium b. NSAIDs c. Fosamax d. Actonel e. Calcitonin

ANS: A, C, D, E Calcium, Evista, Fosamax, Actonel, and Calcitonin can be used by postmenopausal women to treat or prevent osteoporosis. Parathyroid hormone and estrogen may also be of value. NSAIDs may provide pain relief; however, these medications neither prevent nor treat osteoporosis.

10. A probable cause for increasing infertility is the societal delay in pregnancy until later in life. What are the natural reasons for the decrease in female fertility? (Select all that apply.) a. Ovulation dysfunction b. Endocrine dysfunction c. Organ damage from toxins d. Endometriosis e. Tubal infections

ANS: A, C, D, E All of these factors may result in a cumulative effect, decreasing fertility in women. Male infertility is more often caused by unfavorable sperm production attributable to endocrine dysfunction or cumulative metabolic disease.

3. The nurse should be aware that a statistic widely used to compare the health status of different populations is the: a. Incidence of specific infections, such as acquired immunodeficiency syndrome (AIDS) and tuberculosis b. Infant mortality rate c. Maternal morbidity rate d. Incidence of low-birth-weight (LBW) infants

ANS: B City, county, and state health departments provide annual reports of births and deaths. Maternal and infant death rates are particularly important because they reflect health outcomes that may be preventable. Infant mortality continues to be a concern in all populations. AIDS and tuberculosis may be the target of research studies; however, maternal and infant mortality rates are particularly important in the evaluation of the health of a population. The number of maternal deaths in the United States is small; however, worldwide many women die each year from problems related to pregnancy and childbirth. The incidence of LBW infants is monitored in order to determine risk factors such as racial disparity. It is not as widely used as infant mortality.

Five different viruses (A, B, C, D, and E) account for almost all cases of hepatitis infections. Which statement regarding the various forms of hepatitis is most accurate? a.Vaccine exists for hepatitis C virus (HCV) but not for HBV. b.HAV is acquired by eating contaminated food or drinking polluted water. c.HBV is less contagious than HIV. d.Incidence of HCV is decreasing.

ANS: B Contaminated milk and shellfish are common sources of infection for HAV. A vaccine exists for HBV but not for HCV. HBV is more contagious than HIV. The incidence of HCV is on the rise.

6. Contemporary maternity nursing is exemplified by: a. The use of midwives for all vaginal deliveries b. Family-centered care c. Free-standing birth clinics d. Physician-driven care

ANS: B Contemporary maternity nursing focuses on the family's needs and desires. Midwives and physicians both perform vaginal deliveries. Free-standing clinics are an example of alternative birth options. Contemporary maternity nursing is driven by the relationship between nurses and their clients.

Which treatment regime would be most appropriate for a client who has been recently diagnosed with acute pelvic inflammatory disease (PID)? a.Oral antiviral therapy b.Bed rest in a semi-Fowler position c.Antibiotic regimen continued until symptoms subside d.Frequent pelvic examination to monitor the healing progress

ANS: B The woman with acute PID should be on bed rest in a semi-Fowler position. Broad-spectrum antibiotics are used; antiviral therapy is ineffective. Antibiotics must be taken as prescribed, even if symptoms subside. Few pelvic examinations should be conducted during the acute phase of the disease.

26. The nurse should have knowledge of the purpose of the pinch test. It is used to: a. Check the sensitivity of the nipples. b. Determine whether the nipple is everted or inverted. c. Calculate the adipose buildup in the abdomen. d. See whether the fetus has become inactive.

ANS: B The pinch test is used to determine whether the nipple is everted or inverted. Nipples must be everted to allow breastfeeding.

Which statements indicate that the nurse is practicing appropriate family-centered care techniques? (Select all that apply.) a.The nurse commands the pregnant woman to do as she is told. b.The nurse allows time for the partner to ask questions. c.The nurse allows the mother and father to make choices when possible. d.The nurse informs the family about what is going to happen. e.The nurse tells the client's sister, who is a nurse, that she cannot be in the room during the delivery.

ANS: B, C Including the partner in the care process and allowing the couple to make choices are important elements of family-centered care. The nurse should never tell the client what to do. Family-centered care involves collaboration between the health care team and the client. Unless an institutional policy limits the number of attendants at a delivery, the client should be allowed to have whomever she wants present (except when the situation is an emergency and guests are asked to leave).

Management of primary dysmenorrhea often requires a multifaceted approach. Which pharmacologic therapy provides optimal pain relief for this condition? a. Acetaminophen b. Oral contraceptive pills (OCPs) c. Nonsteroidal antiinflammatory drugs (NSAIDs) d. Aspirin

ANS: C NSAIDs have the strongest research results for pain relief. If one NSAID is not effective, then another one may provide relief. Approximately 80% of women find relief from these prostaglandin inhibitors. Preparations containing acetaminophen are less effective for dysmenorrhea because they lack the antiprostaglandin properties of NSAIDs. OCPs are a reasonable choice for women who also want birth control. The benefit of OCPs is the reduction of menstrual flow and irregularities. OCPs may be contraindicated for some women and have a number of potential side effects. NSAIDs are the drug of choice. However, if a woman is taking an NSAID, she should avoid taking aspirin as well.

8. The National Quality Forum has issued a list of "never events" pertaining specifically to maternal and child health. These include all except: a. Infant discharged to the wrong person b. Kernicterus associated with failure to identify and treat hyperbilirubinemia c. Artificial insemination with wrong donor sperm or egg d. Foreign object retained after surgery

ANS: D Although a foreign object retained after surgery is a never event, this does not pertain specifically to obstetric clients. A client undergoing any type of surgery may be at risk for this event. An infant discharged to the wrong person pertains specifically to postpartum care. Death or serious disability as a result of kernicterus pertains to newborn assessment and care. Artificial insemination affects families seeking care for infertility.

12. Parents who have not already done so need to make time for newborn follow-up of the discharge. According to the American Academy of Pediatrics (AAP), when should a breastfeeding infant first need to be seen for a follow-up examination? a. 2 weeks of age b. 7 to 10 days after childbirth c. 4 to 5 days after hospital discharge d. 48 to 72 hours after hospital discharge

ANS: D Breastfeeding infants are routinely seen by the pediatric health care provider clinic within 3 to 5 days after birth or 48 to 72 hours after hospital discharge and again at 2 weeks of age. Formula-feeding infants may be seen for the first time at 2 weeks of age.

18. A woman at 26 weeks of gestation is being assessed to determine whether she is experiencing preterm labor. Which finding indicates that preterm labor is occurring? a. Estriol is not found in maternal saliva. b. Irregular, mild uterine contractions are occurring every 12 to 15 minutes. c. Fetal fibronectin is present in vaginal secretions. d. The cervix is effacing and dilated to 2 cm.

ANS: D Cervical changes such as shortened endocervical length, effacement, and dilation are predictors of imminent preterm labor. Changes in the cervix accompanied by regular contractions indicate labor at any gestation. Estriol is a form of estrogen produced by the fetus that is present in plasma at 9 weeks of gestation. Levels of salivary estriol have been shown to increase before preterm birth. Irregular, mild contractions that do not cause cervical change are not considered a threat. The presence of fetal fibronectin in vaginal secretions between 24 and 36 weeks of gestation could predict preterm labor, but it has only a 20% to 40% positive predictive value. Of more importance are other physiologic clues of preterm labor such as cervical changes. DIF: Cognitive Level: Apply REF: p. 759 TOP: Nursing Process: Assessment | Nursing Process: Planning MSC: Client Needs: Health Promotion and Maintenance

On day 3 of life, a newborn continues to require 100% oxygen by nasal cannula. The parents ask if they may hold their infant during his next gavage feeding. Considering that this newborn is physiologically stable, what response should the nurse provide? a."Parents are not allowed to hold their infants who are dependent on oxygen." b."You may only hold your baby's hand during the feeding." c."Feedings cause more physiologic stress; therefore, the baby must be closely monitored. I don't think you should hold the baby." d."You may hold your baby during the feeding."

ANS: D Physical contact with the infant is important to establish early bonding. The nurse as the support person and teacher is responsible for shaping the environment and making the caregiving responsive to the needs of both the parents and the infant. Allowing the parents to hold their baby is the most appropriate response by the nurse. Parental interaction by holding should be encouraged during gavage feedings; nasal cannula oxygen therapy allows for easy feedings and psychosocial interactions. The parent can swaddle the infant or provide kangaroo care while gavage feeding their infant. Both swaddling and kangaroo care during feedings provide positive interactions for the infant and help the infant associate feedings with positive interactions.

19. Which assessment is least likely to be associated with a breech presentation? a. Meconium-stained amniotic fluid b. Fetal heart tones heard at or above the maternal umbilicus c. Preterm labor and birth d. Postterm gestation

ANS: D Postterm gestation is not likely to occur with a breech presentation. The presence of meconium in a breech presentation may be a result of pressure on the fetal wall as it traverses the birth canal. Fetal heart tones heard at the level of the umbilical level of the mother are a typical finding in a breech presentation because the fetal back would be located in the upper abdominal area. Breech presentations often occur in preterm births. DIF: Cognitive Level: Analyze REF: pp. 775-776 TOP: Nursing Process: Assessment MSC: Client Needs: Health Promotion and Maintenance

When evaluating the preterm infant, the nurse understands that compared with the term infant, what information is important for the nurse to understand? a.Few blood vessels visible through the skin b.More subcutaneous fat c.Well-developed flexor muscles d.Greater surface area in proportion to weight

ANS: D Preterm infants have greater surface area in proportion to their weight. More subcutaneous fat and well-developed muscles are indications of a more mature infant.

9. The obstetric provider has informed the nurse that she will be performing an amniotomy on the client to induce labor. What is the nurse's highest priority intervention after the amniotomy is performed? a. Applying clean linens under the woman b. Taking the client's vital signs c. Performing a vaginal examination d. Assessing the fetal heart rate (FHR)

ANS: D The FHR is assessed before and immediately after the amniotomy to detect any changes that might indicate cord compression or prolapse. Providing comfort measures, such as clean linens, for the client is important but not the priority immediately after an amniotomy. The woman's temperature should be checked every 2 hours after the rupture of membranes but not the priority immediately after an amniotomy. The woman would have had a vaginal examination during the procedure. Unless cord prolapse is suspected, another vaginal examination is not warranted. Additionally, FHR assessment provides clinical cues to a prolapsed cord. DIF: Cognitive Level: Analyze REF: p. 783 TOP: Nursing Process: Implementation MSC: Client Needs: Physiologic Integrity

22. Nurses should be able to teach breastfeeding mothers the signs that the infant has correctly latched on. Which client statement indicates a poor latch? a. I feel a firm tugging sensation on my nipples but not pinching or pain. b. My baby sucks with cheeks rounded, not dimpled. c. My babys jaw glides smoothly with sucking. d. I hear a clicking or smacking sound.

ANS: D The clicking or smacking sound may indicate that the baby is having difficulty keeping the tongue out over the lower gum ridge. The mother should hope to hear the sound of swallowing. The tugging sensation without pinching is a good sign. Rounded cheeks are a positive indicator of a good latch. A smoothly gliding jaw also is a good sign.

15. While obtaining a detailed history from a woman who has recently immigrated from Somalia, the nurse realizes that the client has undergone female genital mutilation. The nurse's best response to this client is: a. "This is a very abnormal practice and rarely seen in the United States." b. "Are you aware of who performed this so that it can be reported to the authorities?" c. "We will be able to fully restore your circumcision after delivery." d. "The extent of your circumcision will affect the potential for complications."

ANS: D The extent of the circumcision is important. The client may experience pain, bleeding, scarring, or infection and may require surgery prior to childbirth. Although this practice is not prevalent in the United States, it is very common in many African and Middle Eastern countries for religious reasons. Mentioning that the practice is abnormal and rarely seen in the United States is culturally insensitive. The infibulation may have occurred during infancy or childhood. The client will have little to no recollection of the event. She would have considered this to be a normal milestone during her growth and development. The International Council of Nurses has spoken out against this procedure as harmful to a woman's health.

16. A newly delivered mother who intends to breastfeed tells her nurse, I am so relieved that this pregnancy is over so that I can start smoking again. The nurse encourages the client to refrain from smoking. However, this new mother is insistent that she will resume smoking. How will the nurse adapt her health teaching with this new information? a. Smoking has little-to-no effect on milk production. b. No relationship exists between smoking and the time of feedings. c. The effects of secondhand smoke on infants are less significant than for adults. d. The mother should always smoke in another room.

ANS: D The new mother should be encouraged not to smoke. If she continues to smoke, she should be encouraged to always smoke in another room, removed from the baby. Smoking may impair milk production. When the products of tobacco are broken down, they cross over into the breast milk. Tobacco also results in a reduction of the antiinfective properties of breast milk. Research supports the conclusion that mothers should not smoke within 2 hours before a feeding (AAP Committee on Drugs, 2001). The effects of secondhand smoke on infants include excessive crying, colic, upper respiratory infections, and an increased risk of sudden infant death syndrome (SIDS).

Because of the premature infant's decreased immune functioning, what nursing diagnosis should the nurse include in a plan of care for a premature infant? a.Delayed growth and development b.Ineffective thermoregulation c.Ineffective infant feeding pattern d.Risk for infection

ANS: D The nurse needs to understand that decreased immune functioning increases the risk for infection. Growth and development, thermoregulation, and feeding may be affected, although only indirectly.

13. On examining a woman who gave birth 5 hours ago, the nurse finds that the woman has completely saturated a perineal pad within 15 minutes. What is the nurses highest priority at this time? a. Beginning an intravenous (IV) infusion of Ringers lactate solution b. Assessing the womans vital signs c. Calling the womans primary health care provider d. Massaging the womans fundus

ANS: D The nurse should first assess the uterus for atony by massaging the womans fundus. Uterine tone must be established to prevent excessive blood loss. The nurse may begin an IV infusion to restore circulatory volume, but this would not be the first action. Blood pressure is not a reliable indicator of impending shock from impending hemorrhage; assessing vital signs should not be the nurses first action. The physician would be notified after the nurse completes the assessment of the woman.

3. In evaluating the effectiveness of magnesium sulfate for the treatment of preterm labor, which finding alerts the nurse to possible side effects? a. Urine output of 160 ml in 4 hours b. DTRs 2+ and no clonus c. Respiratory rate (RR) of 16 breaths per minute d. Serum magnesium level of 10 mg/dl

ANS: D The therapeutic range for magnesium sulfate management is 4 to 7.5 mg/dl. A serum magnesium level of 10 mg/dl could lead to signs and symptoms of magnesium toxicity, including oliguria and respiratory distress. Urine output of 160 ml in 4 hours, DTRs of 2+, and a RR of 16 breaths per minute are all normal findings. DIF: Cognitive Level: Apply REF: p. 767 TOP: Nursing Process: Evaluation MSC: Client Needs: Physiologic Integrity

11. The high cost of health care in the United States is most likely a result of: a. Early postpartum discharge policies b. Midwifery care c. The involvement of nurses in the politics of cost containment d. An emphasis on the use of advanced technology in care

ANS: D The use of advanced technology in care increases costs. Caring for the increased number of low-birth-weight infants in neonatal intensive care unit (NICU) settings contributes significantly to increased health care costs. Early discharges reduce costs. Midwifery care reduces costs. Involvement of nurses should ameliorate costs.

The nurse providing care in a women's health care setting must be knowledgeable about STIs. Which STIs can be successfully treated? a.HSV b.AIDS c.Venereal warts d.Chlamydia

ANS: D The usual treatment for Chlamydia bacterial infection is doxycycline or azithromycin. Concurrent treatment of all sexual partners is needed to prevent recurrence. No known cure is available for HSV; therefore, the treatment focuses on pain relief and preventing secondary infections. Because no cure is known for AIDS, prevention and early detection are the primary focus of care management. HPV causes condylomata acuminata (venereal warts); no available treatment eradicates the virus.

1. A woman gave birth vaginally to a 9-pound, 12-ounce girl yesterday. Her primary health care provider has written orders for perineal ice packs, use of a sitz bath three times daily, and a stool softener. Which information regarding the clients condition is most closely correlated with these orders? a. Woman is a gravida 2, para 2. b.Woman had a vacuum-assisted birth. c.Woman received epidural anesthesia. d.Woman has an episiotomy.

ANS: D These orders are typical interventions for a woman who has had an episiotomy, lacerations, and hemorrhoids. A multiparous classification is not an indication for these orders. A vacuum-assisted birth may be used in conjunction with an episiotomy, which would indicate these interventions. The use of an epidural anesthesia has no correlation with these orders.

18. Which nursing action is most appropriate to correct a boggy uterus that is displaced above and to the right of the umbilicus? a. Notify the physician of an impending hemorrhage. b. Assess the blood pressure and pulse. c. Evaluate the lochia. d. Assist the client in emptying her bladder.

ANS: D Urinary retention may cause overdistention of the urinary bladder, which lifts and displaces the uterus. Nursing actions need to be implemented before notifying the physician. Evaluating blood pressure, pulse, and lochia is important if the bleeding continues; however, the focus at this point is to assist the client in emptying her bladder.

31. The phenomenon of someone other than the mother-to-be experiencing pregnancy-like symptoms such as nausea and weight gain applies to the: a. Mother of the pregnant woman. c. Sister of the pregnant woman. b. Couple's teenage daughter. d. Expectant father.

ANS: D An expectant father's experiencing pregnancy-like symptoms is called the couvade syndrome.

What is the importance of obtaining informed consent when educating a client regarding contraceptive methods? a. Contraception is an invasive procedure that requires hospitalization. b. The method may require a surgical procedure to insert a device. c. The contraception method chosen may be unreliable. d. The method chosen has potentially dangerous side effects.

ANS: D Being aware of the potential side effects is important for couples who are making an informed decision about the use of contraceptives. The only contraceptive method that is a surgical procedure and requires hospitalization is sterilization. Some methods have greater efficacy than others, and this efficacy should be included in the teaching.

33. While you are assessing the vital signs of a pregnant woman in her third trimester, the patient complains of feeling faint, dizzy, and agitated. Which nursing intervention is appropriate? a. Have the patient stand up and retake her blood pressure. b. Have the patient sit down and hold her arm in a dependent position. c. Have the patient lie supine for 5 minutes and recheck her blood pressure on both arms. d. Have the patient turn to her left side and recheck her blood pressure in 5 minutes.

ANS: D Blood pressure is affected by maternal position during pregnancy. The supine position may cause occlusion of the vena cava and descending aorta. Turning the pregnant woman to a lateral recumbent position alleviates pressure on the blood vessels and quickly corrects supine hypotension. Pressures are significantly higher when the patient is standing. This option causes an increase in systolic and diastolic pressures. The arm should be supported at the same level of the heart. The supine position may cause occlusion of the vena cava and descending aorta, creating hypotension.

Which statement regarding the term contraceptive failure rate is the most accurate? a. The contraceptive failure rate refers to the percentage of users expected to have an accidental pregnancy over a 5-year span. b. It refers to the minimum rate that must be achieved to receive a government license. c. The contraceptive failure rate increases over time as couples become more careless. d. It varies from couple to couple, depending on the method and the users.

ANS: D Contraceptive effectiveness varies from couple to couple, depending on how well a contraceptive method is used and how well it suits the couple. The contraceptive failure rate measures the likelihood of accidental pregnancy in the first year only. Failure rates decline over time because users gain experience.

28. When discussing work and travel during pregnancy with a pregnant patient, nurses should instruct them that: a. Women should sit for as long as possible and cross their legs at the knees from time to time for exercise. b. Women should avoid seat belts and shoulder restraints in the car because they press on the fetus. c. Metal detectors at airport security checkpoints can harm the fetus if the woman passes through them a number of times. d. While working or traveling in a car or on a plane, women should arrange to walk around at least every hour or so

ANS: D Periodic walking helps prevent thrombophlebitis. Pregnant women should avoid sitting or standing for long periods and crossing the legs at the knees. Pregnant women must wear lap belts and shoulder restraints. The most common injury to the fetus comes from injury to the mother. Metal detectors at airport security checkpoints do not harm fetuses.

3. Which symptom is considered a first-trimester warning sign and should be reported immediately by the pregnant woman to her health care provider? a. Nausea with occasional vomiting c. Urinary frequency b. Fatigue d. Vaginal bleeding

ANS: D Signs and symptoms that must be reported include severe vomiting, fever and chills, burning on urination, diarrhea, abdominal cramping, and vaginal bleeding. These symptoms may be signs of potential complications of the pregnancy. Nausea with occasional vomiting, fatigue, and urinary frequency are normal first-trimester complaints. Although they may be worrisome or annoying to the mother, they usually are not indications of pregnancy problems.

A woman is using the basal body temperature (BBT) method of contraception. She calls the clinic and tells the nurse, "My period is due in a few days, and my temperature has not gone up." What is the nurse's most appropriate response? a. "This probably means that you're pregnant." b. "Don't worry; it's probably nothing." c. "Have you been sick this month?" d. "You probably didn't ovulate during this cycle."

ANS: D The absence of a temperature decrease most likely is the result of a lack of ovulation. Pregnancy cannot occur without ovulation, which is being measured using the BBT method. A comment such as, "Don't worry; it's probably nothing," discredits the client's concerns. Illness is most likely the cause of an increase in BBT.

15. In her work with pregnant women of various cultures, a nurse practitioner has observed various practices that seemed strange or unusual. She has learned that cultural rituals and practices during pregnancy seem to have one purpose in common. Which statement best describes that purpose? a. To promote family unity b. To ward off the "evil eye" c. To appease the gods of fertility d. To protect the mother and fetus during pregnancy

ANS: D The purpose of all cultural practices is to protect the mother and fetus during pregnancy. Although many cultures consider pregnancy normal, certain practices are expected of women of all cultures to ensure a good outcome. Cultural prescriptions tell women what to do, and cultural proscriptions establish taboos. The purposes of these practices are to prevent maternal illness resulting from a pregnancy-induced imbalanced state and to protect the vulnerable fetus.

20. What represents a typical progression through the phases of a woman's establishing a relationship with the fetus? a. Accepts the fetus as distinct from herself—accepts the biologic fact of pregnancy—has a feeling of caring and responsibility b. Fantasizes about the child's gender and personality—views the child as part of herself—becomes introspective c. Views the child as part of herself—has feelings of well-being—accepts the biologic fact of pregnancy d. "I am pregnant."—"I am going to have a baby."—"I am going to be a mother.

ANS: D The woman first centers on herself as pregnant, then on the baby as an entity separate from herself, and then on her responsibilities as a mother. The expressions, "I am pregnant," "I am going to have a baby," and "I am going to be a mother" sum up the progression through the three phases.

7. Which statement regarding gamete intrafallopian transfer (GIFT) is most accurate? a. Semen is collected after laparoscopy. b. Women must have two normal fallopian tubes. c. Ovulation spontaneously occurs. d. Ova and sperm are transferred to one tube.

ANS: D Similar to in vitro fertilization (IVF), GIFT requires the woman to have at least one normal tube. Ovulation is induced, and the oocytes are aspirated during laparoscopy. Semen is collected before laparoscopy. The ova and sperm are then transferred to one uterine tube, permitting natural fertilization and cleavage.

____________________ is the process by which the parent and infant come to love and accept each other.

Attachment p. 612

The secondary level of prevention is best illustrated by which example? a. Approved infant car seats b. BSE c. Immunizations d. Support groups for parents of children with Down syndrome

B

What is the primary difference between hospital care and home health care? a. Home care is routinely and continuously delivered by professional staff. b. Home care is delivered on an intermittent basis by professional staff. c. Home care is delivered for emergency conditions. d. Home care is not available 24 hours a day.

B

Which key factors play the most powerful role in the behaviors of individuals and families? a. Rituals and customs b. Beliefs and values c. Boundaries and channels d. Socialization processes

B

Other early sensual contacts between infant and mother involve sound and smell. Nurses should be aware that despite what folk wisdom might say: A. High-pitched voices irritate newborns. B. Infants can learn to distinguish their mother's voice from others soon after birth. C. All babies in the hospital smell alike. D. A mother's breast milk has no distinctive odor.

B A. Incorrect: Infants respond positively to high-pitched voices. B. Correct: Infants know the sound of their mother's voice early. C. Incorrect: Each infant has a unique odor. D. Incorrect: Infants quickly learn to distinguish the odor of their mother's breast milk. p. 617

The nurse observes several interactions between a postpartum woman and her new son. What behavior, if exhibited by this woman, would the nurse identify as a possible maladaptive behavior regarding parent-infant attachment? A. Talks and coos to her son B. Seldom makes eye contact with her son C. Cuddles her son close to her D. Tells visitors how well her son is feeding

B A. Incorrect: This is a normal infant-parent interaction. B. Correct: The woman should be encouraged to hold her infant in the en face position and make eye contact with the infant. C. Incorrect: This is a normal infant-parent interaction. D. Incorrect: This is a normal infant-parent interaction. p. 625

When caring for a newborn, the nurse must be alert for signs of cold stress, including: A. decreased activity level. B. increased respiratory rate. C. hyperglycemia. D. shivering.

B. increased respiratory rate. An increased respiratory rate is a sign of cold stress in the newborn. Infants experiencing cold stress have an increased activity level. Hypoglycemia would occur with cold stress. Newborns are unable to shiver as a means of increasing heat production; they increase their activity level instead.

When would the best timeframe be to establish gestational age based on ultrasound? At term 8 weeks Between 14 and 22 weeks 36 weeks

Between 14 and 22 weeks Rationale: Ultrasound determination of gestational age dating is best done between 14 and 22 weeks. It is less reliable after that period because of variability in fetal size. Standard sets of measurements relative to gestational age are noted around 10 to after 12 weeks and include crown-rump length (after 10), biparietal diameter (after 12), femur length, and head and abdominal circumferences.

In which culture is the father more likely to be expected to participate in the labor and delivery? a. Asian-American b. African-American c. European-American d. Hispanic

C

Using the family stress theory as an interventional approach for working with families experiencing parenting challenges, the nurse can assist the family in selecting and altering internal context factors. Which statement best describes the components of an internal context? a. Biologic and genetic makeup b. Maturation of family members c. Familys perception of the event d. Prevailing cultural beliefs of society

C

When weighing the advantages and disadvantages of planning home care for perinatal services, what information should the nurse use in making the decision? a. Home care for perinatal services is more dangerous for vulnerable neonates at risk of acquiring an infection from the nurse. b. Home care for perinatal services is more cost-effective for the nurse than office visits. c. Home care for perinatal services allows the nurse to interact with and include family members in teaching. d. Home care for perinatal services is made possible by the ready supply of nurses with expertise in maternity care.

C

Which statement accurately describes the walking survey as a data collection tool? a. The walking survey determines how much exercise an expectant mother has been getting, to help her make health care decisions. b. The walking survey usually takes place on the maternity ward but can be expanded to other areas of the hospital. c. The walking survey is a method of observing the resources and health-related environment of the community. d. The walking survey is performed by government census takers as part of their canvas.

C

27. The nurse observes a sudden increase in variability on the ERM tracing. Which class of medications may cause this finding? a. Narcotics b. Barbiturates c. Methamphetamines d. Tranquilizers

C (Narcotics, barbiturates, and tranquilizers may be causes of decreased variability; whereas methamphetamines may cause increased variability.)

During a phone follow-up conversation with a woman who is 4 days postpartum, the woman tells the nurse, "I don't know what's wrong. I love my son, but I feel so let down. I seem to cry for no reason!" The nurse would recognize that the woman is experiencing: A. Taking-in B. Postpartum depression (PPD) C. Postpartum blues D. Attachment difficulty

C A. Incorrect: The taking-in phase is the period after birth when the mother focuses on her own psychologic needs. Typically, this period lasts 24 hours. B. Incorrect: PPD is an intense, pervasive sadness marked by severe, labile mood swings; it is more serious and persistent than the PP blues. C. Correct: During the PP blues, women are emotionally labile, often crying easily and for no apparent reason. This lability seems to peak around the fifth PP day. D. Incorrect: Crying is not a maladaptive attachment response; it indicates PP blues. p. 620

A pregnant woman's diet consists almost entirely of whole grain breads and cereals, fruits, and vegetables. The nurse is most concerned about this woman's intake of: a.Calcium b.Protein c.Vitamin B12 d.Folic acid

C. Vitamin B12

A Native-American woman gave birth to a baby girl 12 hours ago. The nurse notes that the woman keeps her baby in the bassinet except for bottle feeding and states that she will wait until she gets home to begin breastfeeding. The nurse recognizes that this behavior is most likely a reflection of what? a. Delayed attachment b. Embarrassment c. Disappointment in the sex of the baby d. Belief that babies should not be fed colostrum

D

In what form do families tend to be the most socially vulnerable? a. Married-blended family b. Extended family c. Nuclear family d. Single-parent family

D

A 40-year-old woman with a high body mass index (BMI) is 10 weeks pregnant. Which diagnostic tool is appropriate to suggest to her at this time? Biophysical profile Incorrect Amniocentesis Maternal serum alpha-fetoprotein (MSAFP) Transvaginal ultrasound

D. Transvaginal ultrasound Rationale: An ultrasound is the method of biophysical assessment of the infant that is performed at this gestational age. Transvaginal ultrasound is especially useful for obese women, whose thick abdominal layers cannot be penetrated adequately with the abdominal approach. A biophysical profile is a method of biophysical assessment of fetal well-being in the third trimester. An amniocentesis is performed after the fourteenth week of pregnancy. A MSAFP test is performed from week 15 to week 22 of the gestation (weeks 16 to 18 are ideal).

Which placental hormone is responsible for regulating glucose availability to the fetus? A. Progesterone B. Estrogen C. hCG D. hPL

D. hPL

d

If nonsurgical treatment for late PPH is ineffective, which surgical procedure would be appropriate to correct the cause of this condition? a. Hysterectomy b. Laparoscopy c. Laparotomy d. Dilation and curettage (D&C)

c

The most effective and least expensive treatment of puerperal infection is prevention. What is the most important strategy for the nurse to adopt? a. Large doses of vitamin C during pregnancy b. Prophylactic antibiotics c. Strict aseptic technique, including hand washing, by all health care personnel d. Limited protein and fat intake

d

The nurse suspects that her postpartum client is experiencing hemorrhagic shock. Which observation indicates or would confirm this diagnosis? a. Absence of cyanosis in the buccal mucosa b. Cool, dry skin c. Calm mental status d. Urinary output of at least 30 ml/hr

d

What is one of the initial signs and symptoms of puerperal infection in the postpartum client? a. Fatigue continuing for longer than 1 week b. Pain with voiding c. Profuse vaginal lochia with ambulation d. Temperature of 38° C (100.4° F) or higher on 2 successive days

a

What is the most common reason for late postpartum hemorrhage (PPH)? a. Subinvolution of the uterus b. Defective vascularity of the decidua c. Cervical lacerations d. Coagulation disorders

b

What is the primary nursing responsibility when caring for a client who is experiencing an obstetric hemorrhage associated with uterine atony? a. Establishing venous access b. Performing fundal massage c. Preparing the woman for surgical intervention d. Catheterizing the bladder

b

What would a steady trickle of bright red blood from the vagina in the presence of a firm fundus suggest to the nurse? a. Uterine atony b. Lacerations of the genital tract c. Perineal hematoma d. Infection of the uterus

d

Which classification of placental separation is not recognized as an abnormal adherence pattern? a. Placenta accreta b. Placenta increta c. Placenta percreta d. Placenta abruptio

b

Which client is at greatest risk for early PPH? a. Primiparous woman (G 2, P 1-0-0-1) being prepared for an emergency cesarean birth for fetal distress b. Woman with severe preeclampsia on magnesium sulfate whose labor is being induced c. Multiparous woman (G 3, P 2-0-0-2) with an 8-hour labor d. Primigravida in spontaneous labor with preterm twins

a

Which condition is considered a medical emergency that requires immediate treatment? a. Inversion of the uterus b. Hypotonic uterus c. ITP d. Uterine atony

c

Which is the initial treatment for the client with vWD who experiences a PPH? a. Cryoprecipitate b. Factor VIII and von Willebrand factor (vWf) c. Desmopressin d. Hemabate

abd

Which medications are used to manage PPH? (Select all that apply.) a. Oxytocin b. Methergine c. Terbutaline d. Hemabate e. Magnesium sulfate

29. When managing the care of a woman in the second stage of labor, the nurse uses various measures to enhance the progress of fetal descent. Which instruction best describes these measures? a. Encouraging the woman to try various upright positions, including squatting and standing b. Telling the woman to start pushing as soon as her cervix is fully dilated c. Continuing an epidural anesthetic so pain is reduced and the woman can relax d. Coaching the woman to use sustained, 10- to 15-second, closed-glottis bearing-down efforts with each contraction

a (Both upright and squatting positions may enhance the progress of fetal descent. Many factors dictate when a woman should begin pushing. Complete cervical dilation is necessary, but complete dilation is only one factor. If the fetal head is still in a higher pelvic station, then the physician or midwife may allow the woman to labor down if the woman is able (allowing more time for fetal descent and thereby reducing the amount of pushing needed). The epidural may mask the sensations and muscle control needed for the woman to push effectively. Closed glottic breathing may trigger the Valsalva maneuver, which increases intrathoracic and cardiovascular pressures, reducing cardiac output and inhibiting perfusion of the uterus and placenta. In addition, holding her breath for longer than 5 to 7 seconds diminishes the perfusion of oxygen across the placenta and results in fetal hypoxia.)

Some pregnant clients may complain of changes in their voice and impaired hearing. The nurse can tell these clients that these are common reactions to: a.A decreased estrogen level b.Displacement of the diaphragm, resulting in thoracic breathing c.Congestion and swelling, which occur because the upper respiratory tract has become more vascular d.Increased blood volume

c. Congestion and swelling, which occur because the upper respiratory tract has become more vascular

A patient who has just learned that she is pregnant asks the nurse at what point the baby's heart will begin beating. Which of the following should the nurse say? 1. 4th gestational week 2. 8th gestational week 3. 12th gestational week 4. 16th gestational week

1. 4th gestational week The heart forms during the 3rd gestational week and begins to beat and circulate blood during the 4th gestational week.

The nurse observes generalized petechiae while assessing the skin of a neonate. What further intervention would the primary health care provider most likely request from the nurse? 1. Wrap the neonate in a warm blanket. 2. Administer vitamin K intramuscularly. 3. Provide ventilator support to the neonate. 4. Clean the neonate skin with lukewarm water.

2. Administer vitamin K intramuscularly.

The nurse is teaching the parents of an infant about prevention and care of diaper rash. Which intervention is appropriate when caring for an infant with diaper rashes? 1. Clean the diaper area with alcohol-based baby wipes. 2. Apply baby powder on the buttocks after cleaning. 3. Change the diaper when the infant voids or stools. 4. Avoid use of soap when cleaning the diaper area.

3. Change the diaper when the infant voids or stools.

A mother expresses fear about changing her infant's diaper after he is circumcised. What does the woman need to be taught to take care of the infant when she gets home? 1. Cleanse the penis with prepackaged diaper wipes every 3 to 4 hours. 2. Apply constant, firm pressure by squeezing the penis with the fingers for at least 5 minutes if bleeding occurs. 3. Cleanse the penis gently with water and put petroleum jelly around the glans after each diaper change. 4. Wash off the yellow exudate that forms on the glans at least once every day to prevent infection.

3. Cleanse the penis gently with water and put petroleum jelly around the glans after each diaper change.

Which health care service represents a primary level of prevention? a. Immunizations b. Breast self-examination (BSE) c. Home care for high-risk pregnancies d. Blood pressure screening

A

16. Which client would not be a suitable candidate for internal EFM? a. Client who still has intact membranes b. Woman whose fetus is well engaged in the pelvis c. Pregnant woman who has a comorbidity of obesity d. Client whose cervix is dilated to 4 to 5 cm

A (For internal EFM, the membranes must have ruptured and the cervix must be dilated at least 2 to 3 cm. The presenting part must be low enough to allow placement of the spiral electrode necessary for internal EFM. The accuracy of EFM is not affected by maternal size. However, evaluating fetal well-being using external EFM may be more difficult on an obese client. The client whose cervix is dilated to 4 to 5 cm is indeed a candidate for internal monitoring.)

6. When assessing a woman in the first stage of labor, which clinical finding will alert the nurse that uterine contractions are effective? a. Dilation of the cervix b. Descent of the fetus to 2 station c. Rupture of the amniotic membranes d. Increase in bloody show

A (The vaginal examination reveals whether the woman is in true labor. Cervical change, especially dilation, in the presence of adequate labor, indicates that the woman is in true labor. Engagement and descent of the fetus are not synonymous and may occur before labor. ROM may occur with or without the presence of labor. Bloody show may indicate a slow, progressive cervical change (e.g., effacement) in both true and false labor.)

19. Conscious relaxation is associated with which method of childbirth preparation? a. Grantly Dick-Read childbirth method b. Lamaze method c. Bradley method d. Psychoprophylactic method

A (With the Grantly Dick-Read method, women are taught to consciously and progressively relax different muscle groups throughout the body until a high degree of skill at relaxation is achieved. The Lamaze method combines controlled muscular relaxation with breathing techniques. The Bradley method advocates natural labor, without any form of anesthesia or analgesia, assisted by a husband-coach and using breathing techniques for labor. The psychoprophylactic method is another name for the Lamaze method.)

1. Emergency conditions during labor that would require immediate nursing intervention can arise with startling speed. Which situations are examples of such an emergency? (Select all that apply.) a. Nonreassuring or abnormal FHR pattern b. Inadequate uterine relaxation c. Vaginal bleeding d. Prolonged second stage e. Prolapse of the cord

A, B, C, E (A nonreassuring or abnormal FHR pattern, inadequate uterine relaxation, vaginal bleeding, infection, and cord prolapse all constitute an emergency during labor that requires immediate nursing intervention. A prolonged second stage of labor after the upper limits for duration is reached. This is 3 hours for nulliparous women and 2 hours for multiparous women.)

5. A couple have arrived for their preprocedural In vitro fertilization-embryo transfer (IVF-ET) interview. Which explanation regarding the procedure is most accurate? a. "The procedure begins with collecting eggs from the woman's ovaries." b. "A donor embryo will be transferred into the woman's uterus." c. "Donor sperm will be used to inseminate the woman." d. "Don't worry about the technical stuff; that's what we are here for."

ANS: A A woman's eggs are collected from her ovaries, fertilized in the laboratory with the partner's sperm, and transferred to her uterus after normal embryonic development has occurred. Transferring a donor embryo to the woman's uterus describes the procedure for a donor embryo. Inseminating the woman with donor sperm describes therapeutic donor insemination. Telling the client not to worry discredits the client's need for teaching and is not the most appropriate response.

Which sexual behaviors are associated with exposure to an STI? (Select all that apply.) a.Fellatio b.Unprotected anal intercourse c.Multiple sex partners d.Dry kissing e.Abstinence

ANS: A, B, C Engaging in fellatio, unprotected anal intercourse, or having multiple sex partners increases the exposure risk and the possibility of acquiring an STI. Dry kissing and abstinence are considered safe sexual practices.

9. Many factors, male and female, contribute to normal fertility. Which factors are possible causes for female infertility? (Select all that apply.) a. Congenital or developmental b. Hormonal or ovulatory c. Tubal or peritoneal d. Uterine e. Emotional or psychologic

ANS: A, B, C, D Female infertility can be attributed to alterations in any one of these systems along with possible vaginal-cervical factors. Although the diagnosis and treatment of infertility require considerable emotional investment and may cause psychologic stress, these are not considered factors associated with infertility. Feelings connected with infertility are many and complex. Resolve is an organization that provides support, advocacy, and education for both clients and health care providers.

SATA 1. Examples of alternative healing modalities include (choose all that apply): a. Acupuncture b. Meditation c. Yoga d. Antibiotics e. Chelation therapy

ANS: A, B, C, E Acupuncture, meditation, yoga, and chelation therapy are examples of alternative healing modalities. Western medicine uses antibiotics. Macrobiotics are commonly used as an alternative therapy.

2. A nurse is discussing the signs and symptoms of mastitis with a mother who is breastfeeding. Which findings should the nurse include in the discussion? (Select all that apply.) a. Breast tenderness b. Warmth in the breast c. Area of redness on the breast often resembling the shape of a pie wedge d. Small white blister on the tip of the nipple e. Fever and flulike symptoms

ANS: A, B, C, E Breast tenderness, warmth in the breast, redness on the breast, and fever and flulike symptoms are commonly associated with mastitis and should be included in the nurses discussion of mastitis. A small white blister on the tip of the nipple generally is not associated with mastitis but is commonly seen in women who have a plugged milk duct.

Nurses are in an ideal position to educate clients who experience PMDD. What self-help activities have been documented as helpful in alleviating the symptoms of PMDD? (Select all that apply.) a. Regular exercise b. Improved nutrition c. Daily glass of wine d. Smoking cessation e. Oil of evening primrose

ANS: A, B, D, E Regular exercise, improved nutrition, smoking cessation, and oil of evening primrose are accurate modalities that may provide significant symptom relief in 1 to 2 months. If no improvement is realized after these changes have been made, then the client may need to begin pharmacologic therapy. Women should decrease their alcohol and caffeinated beverage consumption if they suffer from PMDD.

The nurse is reviewing the educational packet provided to a client about tubal ligation. Which information regarding this procedure is important for the nurse to share? (Select all that apply.) a. "It is highly unlikely that you will become pregnant after the procedure." b. "Tubal ligation is an effective form of 100% permanent sterilization. You won't be able to get pregnant." c. "Sterilization offers some form of protection against sexually transmitted infections (STIs)." d. "Sterilization offers no protection against sexually transmitted infections (STIs)." e. "Your menstrual cycle will greatly increase after your sterilization."

ANS: A, D A woman is unlikely to become pregnant after tubal ligation. However, sterilization offers no protection against STIs and is not 100% effective. Typically, the menstrual cycle remains the same after a tubal ligation.

Which STI does not respond well to antibiotic therapy? a.Chlamydia b.Gonorrhea c.Genital herpes d.Syphilis

ANS: C Genital herpes is a chronic and recurring disease for which no known cure is available; therefore, it does not respond to antibiotics. Chlamydia is a bacterial infection that is treated with doxycycline or azithromycin. Gonorrhea is a bacterial infection that is treated with any of several antibiotics. Syphilis is a bacterial infection that is treated with penicillin.

12. Which statement regarding the nutrient needs of breastfed infants is correct? a. Breastfed infants need extra water in hot climates. b. During the first 3 months, breastfed infants consume more energy than formula-fed infants. c. Breastfeeding infants should receive oral vitamin D drops daily during at least the first 2 months. d. Vitamin K injections at birth are not necessary for breastfed infants.

ANS: C Human milk contains only small amounts of vitamin D. All infants who are breastfed should receive 400 International Units of vitamin D each day. Neither breastfed nor formula-fed infants need to be fed water, not even in very hot climates. During the first 3 months, formula-fed infants consume more energy than breastfed infants and therefore tend to grow more rapidly. Vitamin K shots are required for all infants because the bacteria that produce it are absent from the babys stomach at birth.

Which statement related to the condition of endometriosis is most accurate? a. Endometriosis is characterized by the presence and growth of endometrial tissue inside the uterus. b. It is found more often in African-American women than in Caucasian or Asian women. c. Endometriosis may worsen with repeated cycles or remain asymptomatic and disappear after menopause. d. It is unlikely to affect sexual intercourse or fertility.

ANS: C With endometriosis, the endometrial tissue is outside the uterus. Endometriosis is found equally in Caucasian and African-American women and is slightly more prevalent in Asian women. Symptoms vary among women, ranging from nonexistent to incapacitating. The condition is seven times more prevalent in women who have a firstdegree relative with endometriosis. Women can experience painful intercourse and impaired fertility with endometriosis.

Which condition might premature infants who exhibit 5 to 10 seconds of respiratory pauses, followed by 10 to 15 seconds of compensatory rapid respiration, be experiencing? a.Suffering from sleep or wakeful apnea b. Experiencing severe swings in blood pressure c.Trying to maintain a neutral thermal environment d.Breathing in a respiratory pattern common to premature infants

ANS: D Breathing in a respiratory pattern is called periodic breathing and is common to premature infants. This pattern may still require nursing intervention of oxygen and/or ventilation. Apnea is the cessation of respirations for 20 seconds or longer and should not be confused with periodic breathing.

When providing an infant with a gavage feeding, which infant assessment should be documented each time? a.Abdominal circumference after the feeding b.Heart rate and respirations before feeding c.Suck and swallow coordination d.Response to the feeding

ANS: D Documentation of a gavage feeding should include the size of the feeding tube, the amount and quality of the residual from the previous feeding, the type and quantity of the fluid instilled, and the infant's response to the procedure. Abdominal circumference is not measured after a gavage feeding. Although vital signs may be obtained before feeding, the infant's response to the feeding is more important. Similarly, some older infants may be learning to suck; the most important factor to document would still be the infant's response to the feeding, including the attempts to suck.

6. The trend in the United States is for women to remain hospitalized no longer than 1 or 2 days after giving birth. Which scenario is not a contributor to this model of care? a. Wellness orientation model of care rather than a sick-care model b. Desire to reduce health care costs c. Consumer demand for fewer medical interventions and more family-focused experiences d. Less need for nursing time as a result of more medical and technologic advances and devices available at home that can provide information

ANS: D Nursing time and care are in demand as much as ever; the nurse simply has to do things more quickly. A wellness orientation model of care seems to focus on getting clients out the door sooner. In most cases, less hospitalization results in lower costs. People believe that the family gives more nurturing care than the institution.

A clients household consists of her husband, his mother, and another child. To which family configuration does this client belong? a. Multigenerational family b. Single-parent family c. Married-blended family d. Nuclear family

A

A married couple lives in a single-family house with their newborn son and the husbands daughter from a previous marriage. Based on this information, what family form best describes this family? a. Married-blended family b. Extended family c. Nuclear family d. Same-sex family

A

Which intervention should the nurse perform to determine the baseline measurements of a newborn's physical growth? 1. Place and hold the naked newborn on the scale to obtain weight. 2. Allow the caregiver to hold the infant while measuring its length. 3. Measure the circumference of the head just above the eyebrows. 4. Check for plantar reflex by placing a finger in the newborn's palm.

3. Measure the circumference of the head just above the eyebrows.

6. A woman who is 32 weeks' pregnant is informed by the nurse that a danger sign of pregnancy could be: a. Constipation. b. Alteration in the pattern of fetal movement. c. Heart palpitations. d. Edema in the ankles and feet at the end of the day.

ANS: B An alteration in the pattern or amount of fetal movement may indicate fetal jeopardy. Constipation, heart palpitations, and ankle and foot edema are normal discomforts of pregnancy that occur in the second and third trimesters.

5. Postpartum fatigue (PPF) is more than just feeling tired. It is a complex phenomenon affected by physiologic, psychologic, and situational variables. Which factors contribute to this phenomenon? (Select all that apply.) a. Precipitous labor b. Hospital routines c. Bottle feeding d. Anemia e. Excitement

ANS: B, D, E Physical fatigue and exhaustion are often associated with a long labor or cesarean birth, hospital routines, breastfeeding, and infant care. PPF is also attributed to anemia, infection, or thyroid dysfunction. The excitement and exhilaration of delivering a new infant along with well-intentioned visitors may make rest difficult.

7. A woman who is 14 weeks pregnant tells the nurse that she always had a glass of wine with dinner before she became pregnant. She has abstained during her first trimester and would like to know if it is safe for her to have a drink with dinner now. The nurse would tell her: "Since you're in your second trimester, there's no problem with having one drink with dinner." b. "One drink every night is too much. One drink three times a week should be fine." c. "Since you're in your second trimester, you can drink as much as you like." d. "Because no one knows how much or how little alcohol it takes to cause fetal problems, the best course is to abstain throughout your pregnancy."

ANS: D The statement "Because no one knows how much or how little alcohol it takes to cause fetal problems, the best course is to abstain throughout your pregnancy" is accurate. A safe level of alcohol consumption during pregnancy has not yet been established. Although the consumption of occasional alcoholic beverages may not be harmful to the mother or her developing fetus, complete abstinence is strongly advised.

10. A woman is 3 months pregnant. At her prenatal visit, she tells the nurse that she doesn't know what is happening; one minute she's happy that she is pregnant, and the next minute she cries for no reason. Which response by the nurse is most appropriate? a. "Don't worry about it; you'll feel better in a month or so." b. "Have you talked to your husband about how you feel?" c. "Perhaps you really don't want to be pregnant." d. "Hormonal changes during pregnancy commonly result in mood swings."

ANS: D The statement "Hormonal changes during pregnancy commonly result in mood swings" is accurate and the most appropriate response by the nurse. The statement "Don't worry about it; you'll feel better in a month or so" dismisses the client's concerns and is not the most appropriate response. Although women should be encouraged to share their feelings, "Have you talked to your husband about how you feel" is not the most appropriate response and does not provide the client with a rationale for the psychosocial dynamics of her pregnancy. "Perhaps you really don't want to be pregnant" is completely inappropriate and deleterious to the psychologic well-being of the woman. Hormonal and metabolic adaptations often cause mood swings in pregnancy. The woman's responses are normal. She should be reassured about her feelings.

9. For what reason would breastfeeding be contraindicated? a. Hepatitis B b. Everted nipples c. History of breast cancer 3 years ago d. Human immunodeficiency virus (HIV) positive

ANS: D Women who are HIV positive are discouraged from breastfeeding. Although hepatitis B antigen has not been shown to be transmitted through breast milk, as an added precaution infants born to HBsAg-positive women should receive the hepatitis B vaccine and immune globulin immediately after birth. Everted nipples are functional for breastfeeding. Newly diagnosed breast cancer would be a contraindication to breastfeeding

When the services of an interpreter are needed, which is the most important factor for the nurse to consider? a. Using a family member who is fluent in both languages b. Using an interpreter who is certified, and documenting the persons name in the nursing notes c. Directing questions only to the interpreter d. Using an interpreter only in an emergency

B

Which key point is important for the nurse to understand regarding the perinatal continuum of care? a. Begins with conception and ends with the birth b. Begins with family planning and continues until the infant is 1 year old c. Begins with prenatal care and continues until the newborn is 24 weeks old d. Refers to home care only

B

Which statement regarding the Family Systems Theory is inaccurate? a. Family system is part of a larger suprasystem. b. Family, as a whole, is equal to the sum of the individual members. c. Changes in one family member affect all family members. d. Family is able to create a balance between change and stability.

B

The nurse observes that a 15-year-old mother seems to ignore her newborn. A strategy that the nurse can use to facilitate mother-infant attachment in this mother is: A. Tell the mother she must pay attention to her infant B. Show the mother how the infant initiates interaction and attends to her C. Demonstrate for the mother different positions for holding her infant while feeding D. Arrange for the mother to watch a video on parent-infant interaction

B A. Incorrect: A statement of this kind may be perceived as derogatory and is not appropriate. B. Correct: Pointing out the responsiveness of the infant is a positive strategy for facilitating parent-infant attachment. C. Incorrect: Educating the young mother in infant care is important, but pointing out the responsiveness of her baby is a better tool for facilitating mother-infant attachment. D. Incorrect: Videos are an educational tool that can demonstrate parent-infant attachment, but encouraging the mother to recognize the infant's responsiveness is more appropriate. p. 624

The nurse notes that, when placed on the scale, the newborn immediately abducts and extends the arms, and the fingers fan out with the thumb and forefinger forming a "C." This response is known as what? A. Tonic neck reflex. B. Moro reflex. C. Cremasteric reflex. D. Babinski reflex.

B. Moro reflex. These actions show the Moro reflex. Tonic neck reflex refers to the "fencing posture" a newborn assumes when supine and turns the head to the side. The cremasteric reflex refers to retraction of testes when chilled. The Babinski reflex refers to the flaring of the toes when the sole is stroked.

A woman has come to the clinic for preconception counseling because she wants to start trying to get pregnant in 3 months. She can expect the following advice a."Discontinue all contraception now." b."Lose weight so that you can gain more during pregnancy." c."You may take any medications you have been taking regularly." d."Make sure you include adequate folic acid in your diet."

D. "Make sure you include adequate folic acid in your diet."

A pregnant woman's diet history indicates that she likes the following. The nurse encourages this woman to consume more of which food in order to increase her calcium intake? a.Fresh apricots b.Canned clams c.Spaghetti with meat sauce d.Canned sardines

D. Canned sardines

The nurse sees a woman for the first time when she is 30 weeks pregnant. The woman has smoked throughout the pregnancy, and fundal height measurements now are suggestive of growth restriction in the fetus. In addition to ultrasound to measure fetal size, what would be another tool useful in confirming the diagnosis? Doppler blood flow analysis Contraction stress test (CST) Amniocentesis Daily fetal movement counts

Doppler blood flow analysis Rationale: Doppler blood flow analysis allows the examiner to study the blood flow noninvasively in the fetus and the placenta. It is a helpful tool in the management of high risk pregnancy due to intrauterine growth restriction (IUGR), diabetes mellitus, multiple fetuses, or preterm labor. Because of the potential risk of inducing labor and causing fetal distress, a CST is not performed in a woman whose fetus is preterm. Indications for an amniocentesis include diagnosis of genetic disorders or congenital anomalies, assessment of pulmonary maturity, and the diagnosis of fetal hemolytic disease, not IUGR. Fetal kick count monitoring is performed to monitor the fetus in pregnancies complicated by conditions that may affect fetal oxygenation. Although it may be a useful tool at some point later in this woman's pregnancy, it is not used to diagnose IUGR.

Dietary Reference Intakes (DRIs) have been established for the people of the United States and Canada. These recommendations for daily nutritional intakes meet the needs of approximately 75% to 80% of the healthy population. Is this statement true or false?

False

acde

Lacerations of the cervix, vagina, or perineum are also causes of PPH. Which factors influence the causes and incidence of obstetric lacerations of the lower genital tract? (Select all that apply.) a. Operative and precipitate births b. Adherent retained placenta c. Abnormal presentation of the fetus d. Congenital abnormalities of the maternal soft tissue e. Previous scarring from infection

____________________ is when the fetus begins to descend and drop into the pelvis.

Lightening

Nurses should be aware of the strengths and limitations of various biochemical assessments during pregnancy, including that: Chorionic villus sampling (CVS) is becoming more popular because it provides early diagnosis. Screening for maternal serum alpha-fetoprotein (MSAFP) levels is recommended only for women at risk for neural tube defects. Percutaneous umbilical blood sampling (PUBS) is one of the quad-screen tests for Down syndrome. MSAFP is a screening tool only; it identifies candidates for more definitive procedures.

MSAFP is a screening tool only; it identifies candidates for more definitive procedures. Rationale: CVS does provide a rapid result, but it is declining in popularity because of advances in noninvasive screening techniques. MSAFP screening is recommended for all pregnant women. MSAFP, not PUBS, is part of the quad-screen tests for Down syndrome. MSAFP is a screening tool, not a diagnostic tool. Further diagnostic testing is indicated after an abnormal result.

A nurse is providing instruction for an obstetrical patient to perform a daily fetal movement count (DFMC). Which instructions could be included in the plan of care? (Select all that apply.) The fetal alarm signal is reached when there are no fetal movements noted for 5 hours. The patient can monitor fetal activity once daily for a 60-minute period and note activity. Monitor fetal activity two times a day either after meals or before bed for a period of 2 hours or until 10 fetal movements are noted. Count all fetal movements in a 12-hour period daily until 10 fetal movements are noted.

The patient can monitor fetal activity once daily for a 60-minute period and note activity. Monitor fetal activity two times a day either after meals or before bed for a period of 2 hours or until 10 fetal movements are noted. Count all fetal movements in a 12-hour period daily until 10 fetal movements are noted. Rationale: The fetal alarm signal is reached when no fetal movements are noted for a period of 12 hours.

polyhydramnios

Too much amniotic fluid >1.5 -2 L of amniotic fluid

In nonpregnant women, blood glucose levels must be 160 to 180 mg/dl before glucose is "spilled" into the urine. During pregnancy glucosuria occurs when maternal glucose levels are lower than 160 mg/dl. Is this statement true or false?

True

16. Which information regarding the procedures and criteria for admitting a woman to the hospital labor unit is important for the nurse to understand? a. Client is considered to be in active labor when she arrives at the facility with contractions. b. Client can have only her male partner or predesignated doula with her at assessment. c. Children are not allowed on the labor unit. d. NonEnglish speaking client must bring someone to translate.

a (According to the Emergency Medical Treatment and Active Labor Act (EMTALA), a woman is entitled to active labor care and is presumed to be in true labor until a qualified health care provider certifies otherwise. A woman may have anyone she wishes present for her support. An interpreter must be provided by the hospital, either in person or by a telephonic service. Siblings of the new infant may be allowed at the delivery, depending on hospital policy and adequate preparation and supervision.)

18. Where is the point of maximal intensity (PMI) of the FHR located? a. Usually directly over the fetal abdomen b. In a vertex position, heard above the mothers umbilicus c. Heard lower and closer to the midline of the mothers abdomen as the fetus descends and internally rotates d. In a breech position, heard below the mothers umbilicus

c (Nurses should be prepared for the shift. The PMI of the FHR is usually directly over the fetal back. In a vertex position, the PMI of the FHR is heard below the mothers umbilicus. In a breech position, it is heard above the mothers umbilicus.)

A woman who is at 36 weeks of gestation is having a nonstress test. Which statement indicates her correct understanding of the test? "I will need to have a full bladder for the test to be done accurately." "I should have my husband drive me home after the test because I may be nauseated." "This test will help to determine whether the baby has Down syndrome or a neural tube defect." "This test observes for fetal activity and an acceleration of the fetal heart rate to determine the well-being of the baby."

"This test observes for fetal activity and an acceleration of the fetal heart rate to determine the well-being of the baby." Rationale: The nonstress test is one of the most widely used techniques to determine fetal well-being and is accomplished by monitoring fetal heart rate in conjunction with fetal activity and movements. An ultrasound requires a full bladder. An amniocentesis is the test after which a pregnant woman should be driven home. A maternal serum alpha-fetoprotein test is used in conjunction with unconjugated estriol levels and human chorionic gonadotropin helps to detect Down syndrome.

The nurse is educating the parents of a newborn about the use of the bulb syringe. Which statement from the parents indicates effective learning about the bulb syringe? 1. "It is used in the baby to prevent defecation from the anal opening." 2. "It is used in the baby to reduce the temperature during hypothermia." 3. "It is used in the baby to prevent suffocation and clear airway obstruction." 4. "It is used in the baby to avoid heat loss due to evaporation and convection."

3. "It is used in the baby to prevent suffocation and clear airway obstruction."

18. A nurse caring for a woman in labor should understand that absent or minimal variability is classified as either abnormal or indeterminate. Which condition related to decreased variability is considered benign? a. Periodic fetal sleep state b. Extreme prematurity c. Fetal hypoxemia d. Preexisting neurologic injury

A (When the fetus is temporarily in a sleep state, minimal variability is present. Periodic fetal sleep states usually last no longer than 30 minutes. A woman in labor with extreme prematurity may display a FHR pattern of minimal or absent variability. Abnormal variability may also be related to fetal hypoxemia and metabolic acidemia. Congenital anomalies or a preexisting neurologic injury may also result in absent or minimal variability. Other possible causes might be central nervous system (CNS) depressant medications, narcotics, or general anesthesia.)

A male patient who is undergoing fertility testing has just been informed by his urologist that he will need to provide a masturbated semen sample at his next office visit. Which of the following instructions should the nurse give the patient regarding this procedure? 1. Bring the sample to the office within 3 hours of collection 2. Engage in sexual activity at least daily the week before collecting the sample 3. Take your temperature immediately before collecting the sample 4. Abstain from any sexual activity for 2 to 3 days before producing the sample

4. Abstain from any sexual activity for 2 to 3 days before producing the sample Semen samples, when collected outside of the office, should be brought to the office within an hour of collection, not 3 hours. The patient should abstain from sexual activity for 2 to 3 days before providing the sample. The patient need not take his temperature before collecting the sample; this is a requirement for female patients who are assessing themselves for ovulatory dysfunction. The patient should abstain from sexual activity for 2 to 3 days before providing the sample.

The student nurse is asked to distinguish cutaneous jaundice from normal skin color of a neonate. What will the student nurse do to differentiate them? 1. Evaluate the size of the nipples. 2. Measure the circumference of the head. 3. Observe the symmetry of lip movement. 4. Apply pressure on the forehead with a finger.

4. Apply pressure on the forehead with a finger.

The nurse auscultates a neonate in resting position and hears a murmur. What further assessments should the nurse make to know if the infant has any cardiac defects? 1. Measure the circumference of the head. 2. Assess movements of the lower extremities. 3. Monitor blood pressure (BP) in upper extremities. 4. Assess blood pressure (BP) in all four extremities.

4. Assess blood pressure (BP) in all four extremities.

22. A client is experiencing back labor and complains of intense pain in her lower back. Which measure would best support this woman in labor? a. Counterpressure against the sacrum b. Pant-blow (breaths and puffs) breathing techniques c. Effleurage d. Conscious relaxation or guided imagery

A (Counterpressure is steady pressure applied by a support person to the sacral area with the fist or heel of the hand. This technique helps the woman cope with the sensations of internal pressure and pain in the lower back. The pain management techniques of pant-blow, effleurage, and conscious relaxation or guided imagery are usually helpful for contractions per the gate-control theory.)

The corrected age of an infant who was born at 25 1/7 weeks and is preparing for discharge 124 days past delivery is ______________.

ANS: 42 6/7 weeks The age of a preterm newborn is corrected by adding the gestational age and the postnatal age. For example, an infant born at 32 weeks of gestation 4 weeks ago would now be considered 36 weeks of age. (32 + 4 = 36).

1. What is the most likely cause for early decelerations in the fetal heart rate (FHR) pattern? a. Altered fetal cerebral blood flow b. Umbilical cord compression c. Uteroplacental insufficiency d. Spontaneous rupture of membranes

A (Early decelerations are the fetus response to fetal head compression; these are considered benign, and interventions are not necessary. Variable decelerations are associated with umbilical cord compression. Late decelerations are associated with uteroplacental insufficiency. Spontaneous rupture of membranes has no bearing on the FHR unless the umbilical cord prolapses, which would result in variable or prolonged bradycardia.)

2. Which clinical finding or intervention might be considered the rationale for fetal tachycardia to occur? a. Maternal fever b. Umbilical cord prolapse c. Regional anesthesia d. Magnesium sulfate administration

A (Fetal tachycardia can be considered an early sign of fetal hypoxemia and may also result from maternal or fetal infection. Umbilical cord prolapse, regional anesthesia, and the administration of magnesium sulfate will each more likely result in fetal bradycardia, not tachycardia.)

The nurse hands over a newborn to the mother after phototherapy. After some time the mother reports that the child has loose stools. What would account for the infant's loose stools? 1. Bilirubin-induced gastric motility. 2. Decreased body fluids in the body. 3. Administration of glucose water. 4. Administration of infant formula.

1. Bilirubin-induced gastric motility.

The practice of the calendar rhythm method is based on the number of days in each menstrual cycle. The fertile period is determined after monitoring each cycle for 6 months. The beginning of the fertile period is estimated by subtracting 18 days from the longest cycle and 11 days from the shortest. If the woman's cycles vary in length from 24 to 30 days, then her fertile period would be day _____ through day ______.

ANS: 6; 19 To avoid pregnancy, the couple must abstain from intercourse on days 6 through 19. Ovulation occurs on day 12 (plus or minus 2 days either way).

A nurse is explaining to a young woman who is trying to become pregnant about the process of conception. Which organ should the nurse mention as the site at which fertilization takes place? 1. Ovaries 2. Fallopian tubes 3. Vagina 4. Uterus

2. Fallopian tubes Ovaries produce the ovum, or egg. From there, the ovum travels to the fallopian tubes, where fertilization takes place. Fertilization occurs in the fallopian tubes. The vagina is the muscular tube about 4 inches in length that extends from the cervix to the perineum. It receives the sperm during sexual intercourse, but fertilization occurs in the fallopian tubes. After fertilization of the ovum in the fallopian tubes, the resulting zygote travels into the uterus and implants on the uterine wall.

A couple has decided to terminate their pregnancy based on the results of genetic testing, which showed that the fetus has the genetic disorder trisomy 21. Which of the following are appropriate nursing actions? 1. Explain the stages of grief the couple will experience 2. Persuade the couple to reconsider their decision, as a human life is at stake 3. Encourage the couple to communicate with each other and share emotions 4. Refer the couple to a support group available in their community 5. Praise the couple for their decision and explain how difficult it is to raise a child with Down syndrome

1. Explain the stages of grief the couple will experience 3. Encourage the couple to communicate with each other and share emotions 4. Refer the couple to a support group available in their community

What should nurses be aware of with regard to umbilical cord care? 1. The stump can easily become infected. 2. A nurse noting bleeding from the vessels of the cord should immediately call for assistance. 3. The cord clamp is removed at cord separation. 4. The average cord separation time is 5 to 7 days.

1. The stump can easily become infected.

The nurse is assessing the vital signs of a neonate 12 hours after birth. Which method should the nurse use to check the infant's temperature? 1. Rectal route 2. Axillary route 3. Temporal artery 4. Tymphanic route

2. Axillary route

A nurse must administer erythromycin ophthalmic ointment to a newborn after birth. How should the nurse do this? 1. Instill within 15 minutes of birth for maximum effectiveness 2. Cleanse eyes from inner to outer canthus before administration if necessary 3. Apply directly over the cornea 4. Flush eyes 10 minutes after instillation to reduce irritation

2. Cleanse eyes from inner to outer canthus before administration if necessary

The nurse must administer erythromycin ophthalmic ointment to a newborn after birth. What should the nurse do? 1. Instill within 15 minutes of birth for maximum effectiveness. 2. Cleanse eyes from inner to outer canthus before administration. 3. Apply directly over the cornea. 4. Flush eyes 10 minutes after instillation to reduce irritation.

2. Cleanse eyes from inner to outer canthus before administration.

The nurse is caring for a newborn with a high bilirubin level. What intervention does the nurse perform while using a fiberoptic blanket and phototherapy light for the newborn? 1. Provide intermittent feedings of glucose water. 2. Cover the newborn's eyes with an opaque mask. 3. Place the fully unclothed newborn under the light. 4. Wrap the naked newborn with a fiberoptic blanket.

2. Cover the newborn's eyes with an opaque mask.

A patient who is pregnant comments to the nurse that her breasts have begun to enlarge recently. She asks the nurse what causes this. Which of the following hormones should the nurse mention as being responsible for stimulating this change? 1. Progesterone 2. Estrogen 3. Human chorionic gonadotropin 4. Human placental lactogen

2. Estrogen Progesterone facilitates implantation and decreases uterine contractility. Estrogen stimulates the enlargement of the breasts and uterus. Human chorionic gonadotropin stimulates the corpus luteum so that it will continue to secrete estrogen and progesterone until the placenta is mature enough to secrete these hormones. Human placental lactogen promotes fetal growth by regulating glucose available to the developing human and stimulates breast development in preparation for lactation.

Following circumcision of a newborn, the nurse provides instructions to his parents regarding postcircumcision care. The nurse should tell the parents to do what? 1. Apply topical anesthetics with each diaper change. 2. Expect a yellowish exudate to cover the glans after the first 24 hours. 3. Change the diaper every 2 hours and cleanse the site with soap and water or baby wipes. 4. Apply constant pressure to the site if bleeding occurs and call the physician.

2. Expect a yellowish exudate to cover the glans after the first 24 hours.

The nurse observes increased bilirubin levels in the laboratory reports of a newborn. Which complication does the nurse expect in the newborn if this condition is poorly monitored? 1. Syndactyly 2. Kernicterus 3. Rectal fistula 4. Down syndrome

2. Kernicterus

A nurse is counseling a patient who has just learned that she is pregnant and who admits to drinking several beers a day about the characteristics of fetal alcohol syndrome. Which of the following should the nurse mention? 1. Increased risk of Down syndrome 2. Low birth weight 3. Microcephaly 4. Mental retardation 5. Increased risk of type I diabetes

2. Low birth weight 3. Microcephaly 4. Mental retardation

The nurse is assessing a preterm baby and observes dark red skin color with harlequin signs on the skin. What does the nurse infer from these findings? The baby has what? 1. Hypotension. 2. Polycythemia. 3. Hyperthermia. 4. A neurologic disorder.

2. Polycythemia.

A nurse is explaining to a pregnant client the functions of amniotic fluid. Which of the following should the nurse mention? 1. Delivers oxygen to the fetus 2. Stimulates enlargement of the breasts and uterus 3. Acts as a cushion for the fetus when there are sudden maternal movements 4. Prevents adherence of the developing human to the amniotic membranes 5. Provides a consistent thermal environment

3. Acts as a cushion for the fetus when there are sudden maternal movements 4. Prevents adherence of the developing human to the amniotic membranes 5. Provides a consistent thermal environment

The nurse is assessing a neonate during the first hour of birth. Which signs of birth trauma does the nurse relate to a breech presentation? 1. Marked bruising over the entire face 2. Ecchymotic skin over the entire head 3. Bruising and swelling over the genitalia 4. Linear mark across both sides of the face

3. Bruising and swelling over the genitalia

23. Part of the nurses role is assisting with pushing and positioning. Which guidance should the nurse provide to her client in active labor? a. Encourage the womans cooperation in avoiding the supine position. b. Advise the woman to avoid the semi-Fowler position. c. Encourage the woman to hold her breath and tighten her abdominal muscles to produce a vaginal response. d. Instruct the woman to open her mouth and close her glottis, letting air escape after the push.

A (The woman should maintain a side-lying position. The semi-Fowler position is the recommended side-lying position with a lateral tilt to the uterus. Encouraging the woman to hold her breath and tighten her abdominal muscles is the Valsalva maneuver, which should be avoided. Both the mouth and glottis should be open, allowing air to escape during the push.)

Vitamin K is given to the newborn to do what? 1. Reduce bilirubin levels 2. Increase the production of red blood cells 3. Enhance the ability of blood to clot 4. Stimulate the formation of surfactant

3. Enhance the ability of blood to clot

A nurse observes a patient, who has recently given birth, interacting with her mother, who is visiting her and the baby. The patient and her mother both share the same eye and hair color and are about the same height. The nurse knows that these traits are examples of which of the following? 1. Genotypes 2. Genomes 3. Phenotypes 4. Genetics

3. Phenotypes Genotype refers to a person's genetic makeup, not to the outward expressions of this genetic makeup. A genome is an organism's complete set of DNA. A phenotype refers to how the genes are outwardly expressed (i.e., eye color, hair color, height). Genetics is the study of heredity.

30. The client has delivered by urgent caesarean birth for fetal compromise. Umbilical cord gases were obtained for acid-base determination. The pH is 6.9, partial pressure of carbon dioxide (PCO2) is elevated, and the base deficit is 11 mmol/L. What type of acidemia is displayed by the infant? a. Respiratory b. Metabolic c. Mixed d. Turbulent

A (These findings are evidence of respiratory acidemia. Metabolic acidemia is expressed by a pH <7.20, normal carbon dioxide pressure, and a base excess of 12 mmol/L. Mixed acidemia is evidenced by a pH <7.20, elevated carbon dioxide pressure, and a base excess of 12 mmol/L. There is no such finding as turbulent acidemia.)

The primary healthcare provider instructs the nurse to give a hepatitis B (HepB) vaccine to a newborn. How should the nurse administer the vaccine? Select all that apply. 1. Through the deltoid muscle 2. Via the dorsogluteal muscle 3. Using the vastuslateralis muscle 4. By inserting the needle at a 60-degree angle 5. By inserting the needle at a 90-degree angle

3. Using the vastuslateralis muscle 5. By inserting the needle at a 90-degree angle

The nurse is assessing a neonate with hydrocephaly. What observation reported by the nurse would be consistent with the neonate's condition? 1. A body weight of 7 pounds 2. A heart rate 120 beats/minute 3. A head-to-heel length of 55 cm 4. A head circumference greater than chest circumference

4. A head circumference greater than chest circumference

The nurse is assessing a breast-fed newborn 1 hour after birth. The nurse identified that the glucose levels are less than 25 mg/dl and immediately reports it to the primary health care provider (PHP). What medication administration does the nurse expect the PHP to advise? 1. Cow's milk orally 2. Infant formula orally 3. Intravenous (IV) saline infusion 4. Intravenous (IV) dextrose infusion

4. Intravenous (IV) dextrose infusion

When placing a newborn under a radiant heat warmer to stabilize temperature after birth, what should the nurse do? 1. Place the thermistor probe on the left side of the chest 2. Cover the probe with a nonreflective material 3. Recheck temperature by periodically taking a rectal temperature 4. Perform all examinations and activities under the warmer

4. Perform all examinations and activities under the warmer

A nurse is working with a woman in labor who has sickle cell anemia. The nurse understands that this patient must have which of the following combinations of genes related to this disorder? 1. Two dominant genes 2. A recessive gene and a dominant gene 3. Only one dominant gene, with the other gene missing 4. Two recessive genes

4. Two recessive genes Genetic diseases or disorders, such as sickle cell anemia, are usually related to a defective recessive gene and present in the developing human when both pairs of the gene have the same defect.

The womans family members are present when the nurse arrives for a postpartum and newborn visit. What should the nurse do? a. Observe the family members interactions with the newborn and one another. b. Ask the woman to meet with her and the baby alone. c. Perform a brief assessment on all family members who are present. d. Reschedule the visit for another time so that the mother and infant can be privately assessed.

A

To provide culturally competent care to an Asian-American family, which question should the nurse include during the assessment interview? a. Do you prefer hot or cold beverages? b. Do you want some milk to drink? c. Do you want music playing while you are in labor? d. Do you have a name selected for the baby?

A

Which pictorial tool can assist the nurse in assessing the aspects of family life related to health care? a. Genogram b. Ecomap c. Life-cycle model d. Human development wheel

A

Which term is an accurate description of the process by which people retain some of their own culture while adopting the practices of the dominant society? a. Acculturation b. Assimilation c. Ethnocentrism d. Cultural relativism

A

The nurse can help a father in his transition to parenthood by: A. Pointing out that the infant turned at the sound of his voice B. Encouraging him to go home to get some sleep C. Telling him to tape the infant's diaper a different way D. Suggesting that he let the infant sleep in the bassinet

A A. Correct: Infants respond to the sound of voices. Because attachment involves a reciprocal interchange, observing the interaction between parent and infant is very important. B. Incorrect: Separation of the parent and infant does not encourage parent-infant attachment. C. Incorrect: Educating the parent in infant care techniques is important, but the manner in which a diaper is taped is not relevant and does not enhance parent-infant interactions. D. Incorrect: Parent-infant attachment involves touching, holding, and cuddling. It is appropriate for a father to want to hold the infant as the baby sleeps. p. 623

When the infant's behaviors and characteristics call forth a corresponding set of maternal behaviors and characteristics, this is called: A. Mutuality B. Bonding C. Claiming D. Acquaintance

A A. Correct: Mutuality extends the concept of attachment to include this shared set of behaviors. B. Incorrect: Bonding is the process over time of parents forming an emotional attachment to their infant. Mutuality refers to a shared set of behaviors that is a part of the bonding process. C. Incorrect: Claiming is the process by which parents identify their new baby in terms of likeness to other family members, their differences and uniqueness. Mutuality refers to a shared set of behaviors that is part of the bonding process. D. Incorrect: Like mutuality, acquaintance is part of attachment. It describes how parents get to know their baby during the immediate postpartum period through eye contact, touching, and talking. p. 613

The nurse notes that a Vietnamese woman does not cuddle or interact with her newborn other than to feed him, change his diapers or soiled clothes, and put him to bed. In evaluating the woman's behavior with her infant, the nurse realizes that: A. What appears to be a lack of interest in the newborn is in fact the Vietnamese way of demonstrating intense love by attempting to ward off evil spirits. B. The woman is inexperienced in caring for newborns. C. The woman needs a referral to a social worker for further evaluation of her parenting behaviors once she goes home with the newborn. D. Extra time needs to be planned for assisting the woman in bonding with her newborn.

A A. Correct: The nurse may observe a Vietnamese woman who gives minimal care to her infant and refuses to cuddle or interact with her infant. The apparent lack of interest in the newborn is this cultural group's attempt to ward off evil spirits and actually reflects an intense love and concern for the infant. B. Incorrect: Cultural beliefs are important determinates of parenting behaviors. The woman's "lack of interest" is a Vietnamese cultural behavior. C. Incorrect: Cultural beliefs are important determinates of parenting behaviors. The woman's "lack of interest" is a Vietnamese cultural behavior. The lack of infant interaction is not a form of infant neglect, but rather a demonstration of love and concern for the infant. D. Incorrect: The nurse may observe the woman and may be concerned by the apparent lack of interest in the newborn when, in fact, this is a cultural display of love and concern for the infant. It is important to educate the woman in infant care, but it is equally important to acknowledge her cultural beliefs and practices. p. 627

New parents express concern that because of the mother's emergency cesarean birth under general anesthesia, they did not have the opportunity to hold and bond with their daughter immediately after her birth. The nurse's response should convey to the parents that: A. Attachment, or bonding, is a process that occurs over time and does not require early contact. B. The time immediately after birth is a critical period for humans. C. Early contact is essential for optimum parent-infant relationships. D. They should just be happy that the infant is healthy.

A A. Correct: This statement is accurate. B. Incorrect: The formerly accepted definition of bonding held that the period immediately after birth was a critical time for bonding to occur. Research since has indicated that parent-infant attachment occurs over time. A delay does not inhibit the process. C. Incorrect: Parent-infant attachment involves activities such as touching, holding, and gazing; it is not exclusively eye contact. D. Incorrect: This response is inappropriate because it is derogatory and belittling. pp. 612-613

In the United States, the en face position is preferred immediately after birth. Nurses can facilitate this process by all of these actions except: A. Washing both the infant's face and the mother's face B. Placing the infant on the mother's abdomen or breast with their heads on the same plane C. Dimming the lights D. Delaying the instillation of prophylactic antibiotic ointment in the infant's eyes

A A. Correct: To facilitate the position in which the parent's and infant's faces are approximately 8 inches apart on the same plane, allowing them to make eye contact, the nurse can place the infant at the proper height on the mother's body, dim the light so that the infant's eyes open, and delay putting ointment in the infant's eyes. B. Incorrect: To facilitate the position in which the parent's and infant's faces are approximately 8 inches apart on the same plane, allowing them to make eye contact, the nurse can place the infant at the proper height on the mother's body, dim the light so that the infant's eyes open, and delay putting ointment in the infant's eyes. C. Incorrect: To facilitate the position in which the parent's and infant's faces are approximately 8 inches apart on the same plane, allowing them to make eye contact, the nurse can place the infant at the proper height on the mother's body, dim the light so that the infant's eyes open, and delay putting ointment in the infant's eyes. D. Incorrect: To facilitate the position in which the parent's and infant's faces are approximately 8 inches apart on the same plane, allowing them to make eye contact, the nurse can place the infant at the proper height on the mother's body, dim the light so that the infant's eyes open, and delay putting ointment in the infant's eyes. p. 617

3. Nursing care measures are commonly offered to women in labor. Which nursing measure reflects the application of the gate-control theory? a. Massage the womans back. b. Change the womans position. c. Give the prescribed medication. d. Encourage the woman to rest between contractions.

A (According to the gate-control theory, pain sensations travel along sensory nerve pathways to the brain, but only a limited number of sensations, or messages, can travel through these nerve pathways at one time. Distraction techniques, such as massage or stroking, music, focal points, and imagery, reduce or completely block the capacity of the nerve pathways to transmit pain. These distractions are thought to work by closing down a hypothetic gate in the spinal cord, thus preventing pain signals from reaching the brain. The perception of pain is thereby diminished. Changing the womans position, administering pain medication, and resting between contractions do not reduce or block the capacity of the nerve pathways to transmit pain using the gate-control theory.)

6. Which alteration in the FHR pattern would indicate the potential need for an amnioinfusion? a. Variable decelerations b. Late decelerations c. Fetal bradycardia d. Fetal tachycardia

A (Amnioinfusion is used during labor to either dilute meconium-stained amniotic fluid or supplement the amount of amniotic fluid to reduce the severity of variable FHR decelerations caused by cord compression. Late decelerations are unresponsive to amnioinfusion. Amnioinfusion is not appropriate for the treatment of fetal bradycardia and has no bearing on fetal tachycardia.)

16. The nurse should be cognizant of which important information regarding nerve block analgesia and anesthesia? a. Most local agents are chemically related to cocaine and end in the suffix caine. b. Local perineal infiltration anesthesia is effective when epinephrine is added, but it can be injected only once. c. Pudendal nerve block is designed to relieve the pain from uterine contractions. d. Pudendal nerve block, if performed correctly, does not significantly lessen the bearing-down reflex.

A (Common agents include lidocaine and chloroprocaine. Injections can be repeated to prolong the anesthesia. A pudendal nerve block relieves pain in the vagina, vulva, and perineum but not the pain from uterine contractions. A pudendal nerve block lessens or shuts down the bearing-down reflex.)

3. While evaluating an external monitor tracing of a woman in active labor, the nurse notes that the FHR for five sequential contractions begins to decelerate late in the contraction, with the nadir of the decelerations occurring after the peak of the contraction. What is the nurses first priority? a. Change the womans position. b. Notify the health care provider. c. Assist with amnioinfusion d. Insert a scalp electrode.

A (Late FHR decelerations may be caused by maternal supine hypotension syndrome. These decelerations are usually corrected when the woman turns onto her side to displace the weight of the gravid uterus from the vena cava. If the fetus does not respond to primary nursing interventions for late decelerations, then the nurse should continue with subsequent intrauterine resuscitation measures and notify the health care provider. An amnioinfusion may be used to relieve pressure on an umbilical cord that has not prolapsed. The FHR pattern associated with this situation most likely will reveal variable decelerations. Although a fetal scalp electrode will provide accurate data for evaluating the well-being of the fetus, it is not a nursing intervention that will alleviate late decelerations nor is it the nurses first priority.)

25. Which nursing intervention would result in an increase in maternal cardiac output? a. Change in position b. Oxytocin administration c. Regional anesthesia d. IV analgesic

A (Maternal supine hypotension syndrome is caused by the weight and pressure of the gravid uterus on the ascending vena cava when the woman is in a supine position. This position reduces venous return to the womans heart, as well as cardiac output, and subsequently reduces her blood pressure. The nurse can encourage the woman to change positions and to avoid the supine position. Oxytocin administration, regional anesthesia, and IV analgesic may reduce maternal cardiac output.)

10. What are the legal responsibilities of the perinatal nurses? a. Correctly interpreting FHR patterns, initiating appropriate nursing interventions, and documenting the outcomes b. Greeting the client on arrival, assessing her status, and starting an IV line c. Applying the external fetal monitor and notifying the health care provider d. Ensuring that the woman is comfortable

A (Nurses who care for women during childbirth are legally responsible for correctly interpreting FHR patterns, initiating appropriate nursing interventions based on those patterns, and documenting the outcomes of those interventions. Greeting the client on arrival, assessing her, and starting an IV line are activities that should be performed when any client arrives to the maternity unit. The nurse is not the only one legally responsible for performing these functions. Applying the external fetal monitor and notifying the health care provider is a nursing function that is part of the standard of care for all obstetric clients and falls within the registered nurses scope of practice. Everyone caring for the pregnant woman should ensure that both she and her support partner are comfortable.)

13. Which statement best describes a normal uterine activity pattern in labor? a. Contractions every 2 to 5 minutes b. Contractions lasting approximately 2 minutes c. Contractions approximately 1 minute apart d. Contraction intensity of approximately 500 mm Hg with relaxation at 50 mm Hg

A (Overall contraction frequency generally ranges from two to five contractions per 10 minutes of labor, with lower frequencies during the first stage and higher frequencies observed during the second stage. Contraction duration remains fairly stable throughout the first and second stages, ranging from 45 to 80 seconds, generally not exceeding 90 seconds. Contractions 1 minute apart are occurring too often and would be considered an abnormal labor pattern. The intensity of uterine contractions generally ranges from 25 to 50 mm Hg in the first stage of labor and may rise to more than 80 mm Hg in the second stage.)

10. The nurse should be cognizant of which physiologic effect of pain? a. Predominant pain of the first stage of labor is visceral pain that is located in the lower portion of the abdomen. b. Referred pain is the extreme discomfort experienced between contractions. c. Somatic pain of the second stage of labor is more generalized and related to fatigue. d. Pain during the third stage is a somewhat milder version of the pain experienced during the second stage.

A (Predominant pain comes from cervical changes, the distention of the lower uterine segment, and uterine ischemia. Referred pain occurs when the pain that originates in the uterus radiates to the abdominal wall, lumbosacral area of the back, iliac crests, and gluteal area. Second-stage labor pain is intense, sharp, burning, and localized. Third-stage labor pain is similar to that of the first stage.)

24. Which alterations in the perception of pain by a laboring client should the nurse understand? a. Sensory pain for nulliparous women is often greater than for multiparous women during early labor. b. Affective pain for nulliparous women is usually less than for multiparous women throughout the first stage of labor. c. Women with a history of substance abuse experience more pain during labor. d. Multiparous women have more fatigue from labor and therefore experience more pain.

A (Sensory pain is greater for nulliparous women because their reproductive tract structures are less supple. Affective pain is greater for nulliparous women during the first stage but decreases for both nulliparous and multiparous during the second stage. Women with a history of substance abuse experience the same amount of pain as those without such a history. Nulliparous women have longer labors and therefore experience more fatigue.)

4. Breathing patterns are taught to laboring women. Which breathing pattern should the nurse support for the woman and her coach during the latent phase of the first stage of labor if the couple has attended childbirth preparation classes? a. Slow-paced breathing b. Deep abdominal breathing c. Modified-paced breathing d. Patterned-paced breathing

A (Slow-paced breathing is approximately one half the womans normal breathing rate and is used during the early stages of labor when a woman can no longer walk or talk through her contractions. No such pattern called deep abdominal breathing exists in childbirth preparation. Modified-paced breathing is shallow breathing that is twice the womans normal breathing rate. It is used when labor progresses and the woman can no longer maintain relaxation through paced breathing. Patterned-pace breathing is a fast, 4:1 breathe, breathe, breathe, blow pattern that is used during the transitional phase of labor just before pushing and delivery.)

3. The nurse is caring for a client in early labor. Membranes ruptured approximately 2 hours earlier. This client is at increased risk for which complication? a. Intrauterine infection b. Hemorrhage c. Precipitous labor d. Supine hypotension

A (When the membranes rupture, microorganisms from the vagina can ascend into the amniotic sac, causing chorioamnionitis and placentitis. ROM is not associated with fetal or maternal bleeding. Although ROM may increase the intensity of the contractions and facilitate active labor, it does not result in precipitous labor. ROM has no correlation with supine hypotension.)

8. A woman gave birth to a 7-pound, 6-ounce infant girl 1 hour ago. The birth was vaginal and the estimated blood loss (EBL) was 1500 ml. When evaluating the womans vital signs, which finding would be of greatest concern to the nurse? a. Temperature 37.9 C, heart rate 120 beats per minute (bpm), respirations 20 breaths per minute, and blood pressure 90/50 mm Hg b. Temperature 37.4 C, heart rate 88 bpm, respirations 36 breaths per minute, and blood pressure 126/68 mm Hg c. Temperature 38 C, heart rate 80 bpm, respirations 16 breaths per minute, and blood pressure 110/80 mm Hg d. Temperature 36.8 C, heart rate 60 bpm, respirations 18 breaths per minute, and blood pressure 140/90 mm Hg

A (An EBL of 1500 ml with tachycardia and hypotension suggests hypovolemia caused by excessive blood loss. Temperature 37.4 C, heart rate 88 bpm, respirations 36 breaths per minute, and blood pressure 126/68 mm Hg are normal vital signs except for an increased respiratory rate, which may be secondary to pain from the birth. Temperature 38 C, heart rate 80 bpm, respirations 16 breaths per minute, and blood pressure 110/80 mm Hg are normal vital signs except for the temperature, which may increase to 38 C during the first 24 hours as a result of the dehydrating effects of labor. Temperature 36.8 C, heart rate 60 bpm, respirations 18 breaths per minute, and blood pressure 140/90 mm Hg are normal vital signs, although the blood pressure is slightly elevated, which may be attributable to the use of oxytocic medications.)

1. A woman gave birth to an infant boy 10 hours ago. Where does the nurse expect to locate this womans fundus? a. 1 centimeter above the umbilicus b. 2 centimeters below the umbilicus c. Midway between the umbilicus and the symphysis pubis d. Nonpalpable abdominally

A (The fundus descends approximately 1 to 2 cm every 24 hours. Within 12 hours after delivery the fundus may be approximately 1 cm above the umbilicus. By the sixth postpartum week the fundus is normally halfway between the symphysis pubis and the umbilicus. The fundus should be easily palpated using the maternal umbilicus as a reference point.)

15. Several delivery changes in the integumentary system that appear during pregnancy disappear after birth, although not always completely. What change is almost certain to be completely reversed? a. Nail brittleness b. Darker pigmentation of the areolae and linea nigra c. Striae gravidarum on the breasts, abdomen, and thighs d. Spider nevi

A (The nails return to their prepregnancy consistency and strength. Some women have permanent darker pigmentation of the areolae and linea nigra. Striae gravidarum (stretch marks) usually do not completely disappear. For some women, spider nevi persist indefinitely.)

c

A woman who has recently given birth complains of pain and tenderness in her leg. On physical examination, the nurse notices warmth and redness over an enlarged, hardened area. Which condition should the nurse suspect, and how will it be confirmed? a. Disseminated intravascular coagulation (DIC); asking for laboratory tests b. von Willebrand disease (vWD); noting whether bleeding times have been extended c. Thrombophlebitis; using real-time and color Doppler ultrasound d. Idiopathic or immune thrombocytopenic purpura (ITP); drawing blood for laboratory analysis

1. In assessing the immediate condition of the newborn after birth, a sample of cord blood may be a useful adjunct to the Apgar score. Cord blood is then tested for pH, carbon dioxide, oxygen, and base deficit or excess. Which clinical situation warrants this additional testing? (Select all that apply.) a. Low 5-minute Apgar score b. Intrauterine growth restriction (IUGR) c. Maternal thyroid disease d. Intrapartum fever e. Vacuum extraction

A, B, C, D (The American College of Obstetricians and Gynecologists (ACOG) suggests obtaining cord blood values in all of these clinical situations except for vacuum extractions deliveries. Cord blood gases should also be performed for multifetal pregnancies or abnormal FHR tracings. Samples can be drawn from both the umbilical artery and the umbilical vein. Results may indicate that fetal compromise has occurred.)

2. Women who have participated in childbirth education classes often bring a birth plan with them to the hospital. Which items might this plan include? (Select all that apply.) a. Presence of companions b. Clothing to be worn c. Care and handling of the newborn d. Medical interventions e. Date of delivery

A, B, C, D (The presence of companions, clothing to be worn, care and handling of the newborn, medical interventions, and environmental modifications all might be included in the couples birth plan. Other items include the presence of nonessential medical personnel (students), labor activities such as the tub or ambulation, preferred comfort and relaxation methods, and any cultural or religious requirements. The expected date of delivery would not be part of a birth plan unless the client is scheduled for an elective cesarean birth.)

5. While developing an intrapartum care plan for the client in early labor, which psychosocial factors would the nurse recognize upon the clients pain experience? (Select all that apply.) a. Culture b. Anxiety and fear c. Previous experiences with pain d. Intervention of caregivers e. Support systems

A, B, C, E (Culture: A womans sociocultural roots influence how she perceives, interprets, and responds to pain during childbirth. Some cultures encourage loud and vigorous expressions of pain, whereas others value self-control. The nurse should avoid praising some behaviors (stoicism) while belittling others (noisy expression). Anxiety and fear: Extreme anxiety and fear magnify the sensitivity to pain and impair a womans ability to tolerate it. Anxiety and fear increase muscle tension in the pelvic area, which counters the expulsive forces of uterine contractions and pushing efforts. Previous experiences with pain: Fear and withdrawal are a natural response to pain during labor. Learning about these normal sensations ahead of time helps a woman suppress her natural reactions of fear regarding the impending birth. If a woman previously had a long and difficult labor, she is likely to be anxious. She may also have learned ways to cope and may use these skills to adapt to the present labor experience. Support systems: An anxious partner is less able to provide help and support to a woman during labor. A womans family and friends can be an important source of support if they convey realistic and positive information about labor and delivery. Although the intervention of caregivers may be necessary for the well-being of the woman and her fetus, some interventions add discomfort to the natural pain of labor (i.e., fetal monitor straps, IV lines).)

2. Which alternative approaches to relaxation have proven successful when working with the client in labor? (Select all that apply.) a. Aromatherapy b. Massage c. Hypnosis d. Cesarean birth e. Biofeedback

A, B, C, E (Approaches to relaxation can include neuromuscular relaxation, aromatherapy, music, massage, imagery, hypnosis, or touch relaxation. Cesarean birth is a method of delivery, not a method of relaxation.)

Which are sexually transmitted bacterial infections? Select all that apply. a. Syphilis b. Gonorrhea c. Chlamydia trachomatis d. Vulvovaginal candidiasis e.Pelvic inflammatory disease

A, B, C, E Syphilis is caused by Treponema pallidum, which enters the subcutaneous tissue through microscopic abrasions that can occur during sexual intercourse. Gonorrhea is caused by the aerobic, gram-negative diplococcus N. gonorrhoeae and is transmitted by sexual contact. Chlamydia trachomatis is the most commonly reported STI seen in sexually active women aged 15 to 24. Pelvic inflammatory disease (PID) is caused by N. gonorrhoeae, chlamydia trachomatis, and a wide variety of anaerobic and aerobic bacteria. Women who have multiple sex partners and a history of STIs are at risk for PID. Vulvovaginal candidiasis is a yeast infection caused by Candida albicans and is not sexually transmitted or bacterial. p. 148-152

3. A woman has requested an epidural block for her pain. She is 5 cm dilated and 100% effaced. The baby is in a vertex position and is engaged. The nurse increases the womans IV fluid for a preprocedural bolus. Before the initiation of the epidural, the woman should be informed regarding the disadvantages of an epidural block. Which concerns should the nurse share with this client? (Select all that apply.) a. Ability to move freely is limited. b. Orthostatic hypotension and dizziness may occur. c. Gastric emptying is not delayed. d. Higher body temperature may occur. e. Blood loss is not excessive.

A, B, D (The womans ability to move freely and to maintain control of her labor is limited, related to the use of numerous medical interventions (IV lines and electronic fetal monitoring [EFM]). Significant disadvantages of an epidural block include the occurrence of orthostatic hypotension, dizziness, sedation, and leg weakness. Women who receive an epidural block have a higher body temperature (38 C or higher), especially when labor lasts longer than 12 hours, and may result in an unnecessary neonatal workup for sepsis. An advantage of an epidural block is that blood loss is not excessive. Other advantages include the following: the woman remains alert and able to participate, good relaxation is achieved, airway reflexes remain intact, and only partial motor paralysis develops.)

What concerns about parenthood are often expressed by visually impaired mothers? Choose all that apply. A. Infant safety B. Transportation C. The ability to care for the infant D. Missing out visually E. Needing extra time for parenting activities to accommodate the visual limitations

A, B, D, E Correct: Concerns expressed by visually impaired mothers include infant safety, extra time needed for parenting activities, transportation, handling other people's reactions, providing proper discipline, and missing out visually. Incorrect: Blind people sense reluctance on the part of others to acknowledge that they have a right to be parents. However, blind parents are fully capable of caring for their infants. p. 628

Which of these are the lowest-risk sexual practices? Select all that apply. a. Abstinence b. Dry kissing c. Wet kissing d. Mutual masturbation e. Hugging, massaging, touching (assuming no break in skin) f. Monogamous (both partners and no high-risk activities), but not tested for HIV or other STIs

A, B, D, E The lowest-risk sexual practices include abstinence; dry kissing; mutual masturbation; and hugging, massaging, and touching (assuming no break in skin). Wet kissing and having partners who are both monogamous and who have used no high-risk activities but have not been tested for HIV or other STIs are low-risk practices, but there are potential risks. P. 147

4. The class of drugs known as opioid analgesics (butorphanol, nalbuphine) is not suitable for administration to women with known opioid dependence. The antagonistic activity could precipitate withdrawal symptoms (abstinence syndrome) in both mothers and newborns. Which signs would indicate opioid or narcotic withdrawal in the mother? (Select all that apply.) a. Yawning, runny nose b. Increase in appetite c. Chills or hot flashes d. Constipation e. Irritability, restlessness

A, C, E (The woman experiencing maternal opioid withdrawal syndrome will exhibit yawning, runny nose, sneezing, anorexia, chills or hot flashes, vomiting, diarrhea, abdominal pain, irritability, restlessness, muscle spasms, weakness, and drowsiness. Assessing both the mother and the newborn and planning the care accordingly are important steps for the nurse to take.)

The nurse is assisting a client during delivery. What measures does the nurse take to protect the infant from heat loss? A. Ensure the infant is dried immediately after birth. B. Place the naked infant on bare scales for accuracy. C. Place the naked infant on the mother's bare chest and cover with a blanket. D. Ensure the nursery temperature is 27° C (80.6° F). E. Wrap the infant and cover the head with a cap.

A, C, E Heat loss by evaporation is intensified if the newborn is not dried immediately after birth. It is important to dry the infant quickly after birth to prevent hypothermia. The naked infant is placed on the mother's bare chest and covered with a warm blanket to reduce heat loss. The infant must be wrapped in a warm blanket; the head may be covered with a cap to conserve heat. The naked infant is weighed on scales with a protective cover to minimize conductive heat loss. The ambient temperature in the nursery is generally maintained at 24° C (75.2° F) and the infant lies in an open bassinet with a warm blanket and a cap.

Which symptoms indicate syphilis? Select all that apply. a. Generalized lymphadenopathy b. Abdominal pain and irregular bleeding c. Wartlike infectious lesions on the vulva d. A painful papule at the site of inoculation e. The presence of rash on the palms and soles

A, C, E Generalized lymphadenopathy indicates secondary syphilis, which occurs 6 weeks to 6 months after the appearance of the chancre. Wartlike infectious lesions, also known as condylomata lata, may develop on the vulva, the perineum, or the anus. The presence of rash on the palms and soles also indicates secondary syphilis. Abdominal pain and irregular bleeding are symptoms of pelvic inflammatory disease (PID). Primary syphilis is a lesion that begins as a painless papule at the site of inoculation and becomes a nontender, shallow, indurated, clean ulcer. p. 151

A female client is receiving treatment for gonorrhea. What does the nurse tell the client while providing care? Select all that apply. a. Recommend that the client be tested for human immunodeficiency virus (HIV) b. Inform the client that the treatment has a poor success rate. c. Reinforce teaching of correct condom use during sexual activity. d. Inform the client that the disease is not reportable. e. Encourage the client to notify her partners about the infection.

A, C, E There is an increased risk for HIV infection in clients who have gonorrhea. Therefore the nurse needs to counsel the client to seek HIV testing. The nurse needs to instruct the client to use a condom during sexual activities, because there may be a chance of reinfection if preventive measures are not taken. Gonorrhea is a highly communicable disease, so the nurse encourages the client to notify her partners about their exposure to the disease so that they can seek appropriate assessment and treatment. The treatment does not have a poor success rate, but reinfection occurs if the client does not take proper preventive measures. Gonorrhea is a reportable communicable disease, and the nurse needs to inform the client that the case will be reported to the health authorities. p. 151

What are some known triggers that cause an outbreak of genital herpes? Select all that apply. a. Stress b. Safety issues c. Menstruation d. Acute illnesses e. Family illnesses f. Ultraviolet light

A, C, F Some known triggers that can cause an outbreak of genital herpes include stress, menstruation, and ultraviolet light. Safety issues and acute or family illnesses are not known triggers for an outbreak of genital herpes. P. 157

After you complete your nutritional counseling for a pregnant woman, you ask her to repeat your instructions so that you can assess her understanding of the instructions given. Which statement indicates that she understands the role of protein in her pregnancy? a."Protein will help my baby grow." b."Eating protein will prevent me from becoming anemic." c."Eating protein will make my baby have strong teeth after he is born." d."Eating protein will prevent me from being diabetic."

A. "Protein will help my baby grow."

The nurse is assessing digestion and elimination in a newborn. Which enzyme helps the newborn convert starch into maltose? A. Amylase in colostrum B. Mammary lipase in breast milk C. Amylase in the salivary glands D. Lactase in the digestive system

A. Amylase in colostrum The enzyme amylase is necessary to convert starch into maltose and occurs in high amounts in colostrum. Mammary lipase in breast milk aids in the digestion of fats. The salivary glands produce amylase starting only at 3 months of age, so the newborn depends on the amylase available in colostrum. The newborn's digestive system produces a high level of lactase, which aids in the digestion of lactose, a carbohydrate present in milk.

Factors that place a man at risk for infertility are: (select all of the correct answers) A. Cigarette smoking B. Eating disorders C. Excessive exercising D. Use of marijuana

A. Cigarette smoking D. Use of marijuana

With regard to the respiratory development of the newborn, of what should nurses be aware? A. Crying increases the distribution of air in the lungs B. Newborns must expel the fluid in utero from the respiratory system within a few minutes of birth C. Newborns are instinctive mouth breathers D. Seesaw respirations are no cause for concern in the first hour after birth

A. Crying increases the distribution of air in the lungs Respirations in the newborn can be stimulated by mechanical factors such as changes in intrathoracic pressure resulting from the compression of the chest during vaginal birth. With birth, the pressure on the chest is released, which helps draw air into the lungs. The positive pressure created by crying helps to keep the alveoli open and increases distribution of air throughout the lungs. Newborns continue to expel fluid for the first hour of life. Newborns are natural nose breathers; they may not have the mouth breathing response to nasal blockage for 3 weeks. Seesaw respirations instead of normal abdominal respirations are not normal and should be reported.

Which nutrient's recommended dietary allowance (RDA) is higher during lactation than during pregnancy? a.Energy (kcal) b.Iron c.Vitamin A d.Folic acid

A. Energy (kcal)

17. Rho immune globulin will be ordered postpartum if which situation occurs? a. Mother Rh, baby Rh+ b. Mother Rh, baby Rh c. Mother Rh+, baby Rh+ d. Mother Rh+, baby Rh

ANS: A An Rh mother delivering an Rh+ baby may develop antibodies to fetal cells that entered her bloodstream when the placenta separated. The Rho immune globulin works to destroy the fetal cells in the maternal circulation before sensitization occurs. If mother and baby are both Rh+ or Rh the blood types are alike, so no antibody formation would be anticipated. If the Rh+ blood of the mother comes in contact with the Rh blood of the infant, no antibodies would develop because the antigens are in the mothers blood, not in the infants.

Your patient delivered a full-term baby 12 hours ago.The woman and her husband, who are in their early 20s,have just been informed by their pediatrician that their baby has trisomy 21 (Down syndrome). This is their first child and they did not have prenatal genetic testing. Your nursing care will include (select all of the correct nursing actions): A. Explaining that they will go through a grieving process over the loss of their "dream" child. B. Explaining the importance of talking openly to each other about their feelings and concerns regarding their child. C. Explaining they will benefit from seeing a genetic counselor. D. Explaining that they should direct questions regard-ing the baby's diagnosis to their pediatrician.

A. Explaining that they will go through a grieving process over the loss of their "dream" child. B. Explaining the importance of talking openly to each other about their feelings and concerns regarding their child. C. Explaining they will benefit from seeing a genetic counselor.

The nurse is assessing a neonate born by vacuum extraction. What assessment does the nurse perform to detect possible subgaleal hemorrhage? A. Measure serial head circumference. B. Monitor the neonate for bradycardia. C. Inspect the frontal aspect of the head. D. Look for backward positioning of the ears.

A. Measure serial head circumference. The nurse should obtain serial head circumference measurements for early detection of possible hemorrhage. Increasing head circumference may be an early sign of a subgaleal hemorrhage. The neonate with subgaleal hemorrhage will have tachycardia, not bradycardia. The nurse must inspect the back of the neck for increasing edema and a firm mass. If hematoma is present, it extends posteriorly, leading to a forward and lateral positioning of the neonate's ears.

The nurse is assessing a neonate immediately after birth. How does the nurse document the presence of bluish-black pigmentation on the neonate's buttocks? A. Mongolian spots B. Nevus simplex C. Nevus flammeus D. Erythema toxicum

A. Mongolian spots Mongolian spots are bluish-black areas of pigmentation on the neonate's back. This information must be documented because they can be mistaken for bruises after discharge, raising the suspicion of physical abuse. Nevi simplex are usually small, flat, pink lesions that are easily blanched. The most common sites are the upper eyelids, nose, upper lip, and nape of the neck. Nevus flammeus, or a port-wine stain, is usually pink and flat at birth, but darkens with time, turning red or purple and becoming pebbly in consistency. Erythema toxicum is a transient rash that first appears in term neonates during the first 24 to 72 hours after birth and can last up to 3 weeks.

The nurse is caring for a full-term neonate born by cesarean. What is the effect of cesarean birth on the respiratory function of the neonate? A. Retention of fluid in the lungs B. Incidence of transient bradypnea C. Exhaustion from the effort of breathing D. Episodes of periodic breathing

A. Retention of fluid in the lungs Before the onset of labor, and during labor, a catecholamine surge promotes fluid clearance from the lungs. This is absent during birth by cesarean when the mother does not go into labor. The full-term neonate born by cesarean is likely to experience some retention of fluid in the lungs, which generally clears without any deleterious effects. The neonate is more likely to develop transient tachypnea of the newborn (TTNB), not bradypnea, due to lower levels of catecholamines. Preterm or sick term infants may experience exhaustion from breathing due to absent or decreased surfactant in the lungs, which causes more pressure on the lungs. It is normal for all infants to experience periodic breathing, with pauses in respirations lasting less than 20 seconds during the active sleep cycle.

You are a nurse in a family planning clinic. Your patienthas been married for 5 years. She used an IUD, whichwas removed 12 months ago. She informs you that sheand her husband have been trying to get pregnant forthe past 12 months. Initial screening to determine the cause of infertility includes (select all of the correct answers): A. Sperm analysis B. Testicular biopsy C. Assessing for ovulation D. Hysterosalpingogram

A. Sperm analysis C. Assessing for ovulation D. Hysterosalpingogram

The nurse notices that a newborn has difficulty breathing. What infant behavior might have led to the nurse to this conclusion? A. The infant did not cry after birth. B. The infant had improper bowel sounds. C. The infant moved its head from side to side. D. The infant had increased blood pressure (BP).

A. The infant did not cry after birth. The nurse concludes that the newborn has difficulty breathing because the infant did not cry after birth. Crying creates positive intrathoracic pressure, which helps draw air into the alveoli of lungs and promotes respiration. Increased BP is a normal finding after the birth and does not cause any breathing difficulties. Improper bowel sounds may indicate a gastrointestinal disorder, but is not related to respiration. Side to side head movement is common after the birth of an infant and it is not associated with breathing difficulties.

The nurse performs nasal and oral suctioning of a newborn immediately after birth. What is the reason for this nursing intervention? A. To stimulate respiration B. Assist in stimulating cardiac activity C. Removal of fluid from the lungs D. To increase pulmonary blood flow

A. To stimulate respiration Respiration in a newborn is stimulated by several chemical, mechanical, thermal, and sensory factors working together. Suctioning of the mouth and nose of the newborn stimulates the respiratory center. Thoracic squeezing in the newborn helps to remove fluid from the lungs; however, suction helps to remove the secretions from the upper respiratory tract. If cardiac activity is absent in the newborn, it can be stimulated by cardiopulmonary resuscitation. The pulmonary blood flow increases spontaneously once the newborn starts breathing.

What findings might the nurse expect in a neonate within 30 minutes of birth? A. Tremors B. Nasal flaring C. Audible grunting D. Pinkish skin color E. Quick respiration

A. Tremors, B. Nasal flaring, C. Audible grunting The first 30 minutes after birth is referred to as the first period of reactivity. Tremors, nasal flaring, and grunting are the signs seen in this phase. These signs disappear within the first hour of birth. Pinkish skin color and quick, shallow respirations are not observed immediately after the birth; they are observed in the period of decreased responsiveness, which occurs around 60 to 100 minutes after the first period of reactivity.

A nurse caring for a newborn should be aware that the sensory system least mature at the time of birth is what? A. vision. B. hearing. C. smell. D. taste.

A. Vision The visual system continues to develop for the first 6 months. As soon as the amniotic fluid drains from the ear (minutes), the infant's hearing is similar to that of an adult. Newborns have a highly developed sense of smell. The newborn can distinguish and react to various tastes.

4. A woman in preterm labor at 30 weeks of gestation receives two 12-mg intramuscular (IM) doses of betamethasone. What is the purpose of this pharmacologic intervention? a. To stimulate fetal surfactant production b. To reduce maternal and fetal tachycardia associated with ritodrine administration c. To suppress uterine contractions d. To maintain adequate maternal respiratory effort and ventilation during magnesium sulfate therapy

ANS: A Antenatal glucocorticoids administered as IM injections to the mother accelerate fetal lung maturity. Propranolol (Inderal) is given to reduce the effects of ritodrine administration. Betamethasone has no effect on uterine contractions. Calcium gluconate is given to reverse the respiratory depressive effects of magnesium sulfate therapy. DIF: Cognitive Level: Understand REF: p. 769 TOP: Nursing Process: Planning MSC: Client Needs: Physiologic Integrity

15. Which description most accurately describes the augmentation of labor? a. Is part of the active management of labor that is instituted when the labor process is unsatisfactory b. Relies on more invasive methods when oxytocin and amniotomy have failed c. Is a modern management term to cover up the negative connotations of forceps-assisted birth d. Uses vacuum cups

ANS: A Augmentation is part of the active management of labor that stimulates uterine contractions after labor has started but is not progressing satisfactorily. Augmentation uses amniotomy and oxytocin infusion, as well as some more gentle, noninvasive methods. Forceps-assisted births are less common than in the past and not considered a method of augmentation. A vacuum-assisted delivery occurs during childbirth if the mother is too exhausted to push. Vacuum extraction is not considered an augmentation methodology. DIF: Cognitive Level: Understand REF: pp. 785-786 TOP: Nursing Process: Planning MSC: Client Needs: Health Promotion and Maintenance

10. A new mother asks the nurse what the experts say about the best way to feed her infant. Which recommendation of the American Academy of Pediatrics (AAP) regarding infant nutrition should be shared with this client? a. Infants should be given only human milk for the first 6 months of life. b. Infants fed on formula should be started on solid food sooner than breastfed infants. c. If infants are weaned from breast milk before 12 months, then they should receive cows milk, not formula. d. After 6 months, mothers should shift from breast milk to cows milk.

ANS: A Breastfeeding and human milk should also be the sole source of milk for the first 12 months, not for only the first 6 months. Infants should be started on solids when they are ready, usually at 6 months, whether they start on formula or breast milk. If infants are weaned from breast milk before 12 months, then they should receive iron-fortified formula, not cows milk.

11. Which statement is the best rationale for recommending formula over breastfeeding? a. Mother has a medical condition or is taking drugs that could be passed along to the infant via breast milk. b. Mother lacks confidence in her ability to breastfeed. c. Other family members or care providers also need to feed the baby. d. Mother sees bottle feeding as more convenient.

ANS: A Breastfeeding is contraindicated when mothers have certain viruses, tuberculosis, are undergoing chemotherapy, or are using or abusing drugs. Some women lack confidence in their ability to produce breast milk of adequate quantity or quality. The key to encouraging these mothers to breastfeed is anticipatory guidance beginning as early as possible during the pregnancy. A major barrier for many women is the influence of family and friends. She may view formula feeding as a way to ensure that the father and other family members can participate. Each encounter with the family is an opportunity for the nurse to educate, dispel myths, and clarify information regarding the benefits of breastfeeding. Many women see bottle feeding as more convenient and less embarrassing than breastfeeding. They may also see breastfeeding as incompatible with an active social life. Although modesty issues related to feeding the infant in public may exist, these concerns are not legitimate reasons to formula-feed an infant. Often, the decision to formula feed rather than breastfeed is made without complete information regarding the benefits of breastfeeding.

29. A new mother asks whether she should feed her newborn colostrum, because it is not real milk. What is the nurses most appropriate answer? a. Colostrum is high in antibodies, protein, vitamins, and minerals. b. Colostrum is lower in calories than milk and should be supplemented by formula. c. Giving colostrum is important in helping the mother learn how to breastfeed before she goes home. d. Colostrum is unnecessary for newborns.

ANS: A Colostrum is important because it has high levels of the nutrients needed by the neonate and helps protect against infection. Supplementation is not necessary and will decrease stimulation to the breast and decrease the production of milk. It is important for the mother to feel comfortable in this role before discharge; however, the importance of the colostrum to the infant is the top priority. Colostrum provides immunities and enzymes necessary to cleanse the gastrointestinal system, among other things.

15. When should discharge instruction, or the teaching plan that tells the woman what she needs to know to care for herself and her newborn, officially begin? a. At the time of admission to the nurses unit b. When the infant is presented to the mother at birth c. During the first visit with the physician in the unit d. When the take-home information packet is given to the couple

ANS: A Discharge planning, the teaching of maternal and newborn care, begins on the womans admission to the unit, continues throughout her stay, and actually never ends as long as she has contact with medical personnel.

7. A woman asks the nurse, "What protects my baby's umbilical cord from being squashed while the baby's inside of me?" The nurse's best response is: a. "Your baby's umbilical cord is surrounded by connective tissue called Wharton jelly, which prevents compression of the blood vessels and ensures continued nourishment of your baby." b. "Your baby's umbilical floats around in blood anyway." c. "You don't need to be worrying about things like that." d. "The umbilical cord is a group of blood vessels that are very well protected by the placenta."

ANS: A Explaining the structure and function of the umbilical cord is the most appropriate response. The umbilical cord does not float around in blood. Telling the client not to worry negates her need for teaching and discounts her feelings. The placenta does not protect the umbilical cord. The cord is protected by the surrounding Wharton jelly.

2. A pregnant woman wants to breastfeed her infant; however, her husband is not convinced that there are any scientific reasons to do so. The nurse can give the couple printed information comparing breastfeeding and bottle feeding. Which statement regarding bottle feeding using commercially prepared infant formulas might influence their choice? a. Bottle feeding using a commercially prepared formula increases the risk that the infant will develop allergies. b. Bottle feeding helps the infant sleep through the night. c. Commercially prepared formula ensures that the infant is getting iron in a form that is easily absorbed. d. Bottle feeding requires that multivitamin supplements be given to the infant.

ANS: A Exposure to cows milk poses a risk of developing allergies, eczema, and asthma. Newborns should be fed during the night, regardless of the feeding method. Iron is better absorbed from breast milk than from formula. Commercial formulas are designed to meet the nutritional needs of the infant and to resemble breast milk. No supplements are necessary.

Chapter 01: 21st Century Maternity and Women's Health 1. To assess a mother's risk of having a low-birth-weight (LBW) infant, what is the most important factor for the nurse to consider? a. African-American race b. Cigarette smoking c. Poor nutritional status d. Limited maternal education

ANS: A For African-American births, the incidence of LBW infants is twice that of Caucasian births. Race is a nonmodifiable risk factor. Cigarette smoking is an important factor in potential infant mortality rates, but it is not the most important. Additionally, smoking is a modifiable risk factor. Poor nutrition is an important factor in potential infant mortality rates, but it is not the most important. Additionally, nutritional status is a modifiable risk factor. Maternal education is an important factor in potential infant mortality rates, but it is not the most important. Additionally, maternal education is a modifiable risk factor.

13. The nurse should be cognizant of which statement regarding the unique qualities of human breast milk? a. Frequent feedings during predictable growth spurts stimulate increased milk production. b. Milk of preterm mothers is the same as the milk of mothers who gave birth at term. c. Milk at the beginning of the feeding is the same as the milk at the end of the feeding. d. Colostrum is an early, less concentrated, less rich version of mature milk.

ANS: A Growth spurts (at 10 days, 3 weeks, 6 weeks, and 3 months) usually last 24 to 48 hours, after which the infants resume normal feeding. The milk of mothers of preterm infants is different from that of mothers of full-term infants to meet the needs of these newborns. Milk changes composition during feeding. The fat content of the milk increases as the infant feeds. Colostrum precedes mature milk and is more concentrated and richer in proteins and minerals (but not fat).

13. The level of practice a reasonably prudent nurse provides is called: a. The standard of care b. Risk management c. A sentinel event d. Failure to rescue

ANS: A Guidelines for standards of care are published by various professional nursing organizations. Risk management identifies risks and establishes preventive practices, but it does not define the standard of care. Sentinel events are unexpected negative occurrences. They do not establish the standard of care. Failure to rescue is an evaluative process for nursing, but it does not define the standard of care.

What is the minimum level of practice that a reasonably prudent nurse is expected to provide? a.Standard of care b.Risk management c.Sentinel event d.Failure to rescue

ANS: A Guidelines for standards of care are published by various professional nursing organizations. Risk management identifies risks and establishes preventive practices, but it does not define the standard of care. Sentinel events are unexpected negative occurrences. They do not establish the standard of care. Failure to rescue is an evaluative process for nursing, but it does not define the standard of care.

A recently graduated nurse is attempting to understand the reason for increasing health care spending in the United States. Which information gathered from her research best explains the rationale for these higher costs compared with other developed countries? a.Higher rate of obesity among pregnant women b.Limited access to technology c.Increased use of health care services along with lower prices d.Homogeneity of the population

ANS: A Health care is one of the fastest growing sectors of the U.S. economy. Currently, 17.4% of the gross domestic product is spent on health care. Higher spending in the United States, as compared with 12 other industrialized countries, is related to higher prices and readily accessible technology along with greater obesity rates among women. More than one third of women in the United States are obese. In the population in the United States, 16% are uninsured and have limited access to health care. Maternal morbidity and mortality are directly related to racial disparities.

7. Parents have been asked by the neonatologist to provide breast milk for their newborn son, who was born prematurely at 32 weeks of gestation. The nurse who instructs them regarding pumping, storing, and transporting the milk needs to assess their knowledge of lactation. Which statement is valid? a. Premature infants more easily digest breast milk than formula. b. A glass of wine just before pumping will help reduce stress and anxiety. c. The mother should only pump as much milk as the infant can drink. d. The mother should pump every 2 to 3 hours, including during the night.

ANS: A Human milk is the ideal food for preterm infants, with benefits that are unique, in addition to those benefits received by full-term, healthy infants. Greater physiologic stability occurs with breastfeeding, compared with formula feeding. Consumption of alcohol during lactation is approached with caution. Excessive amounts can have serious effects on the infant and can adversely affect the mothers milk ejection reflex. To establish an optimal milk supply, the most appropriate instruction for the mother should be to pump 8 to 10 times a day for 10 to 15 minutes on each breast.

14. In many hospitals, new mothers are routinely presented with gift bags containing samples of infant formula. This practice is inconsistent with what? a. Baby Friendly Hospital Initiative b. Promotion of longer periods of breastfeeding c. Perception of being supportive to both bottle feeding and breastfeeding mothers d. Association with earlier cessation of breastfeeding

ANS: A Infant formula should not be given to mothers who are breastfeeding. Such gifts are associated with early cessation of breastfeeding. Baby Friendly USA prohibits the distribution of any gift bags or formula to new mothers.

16. The exact cause of preterm labor is unknown but believed to be multifactorial. Infection is thought to be a major factor in many preterm labors. Which type of infection has not been linked to preterm birth? a. Viral b. Periodontal c. Cervical d. Urinary tract

ANS: A Infections that increase the risk of preterm labor and birth are bacterial and include cervical, urinary tract, periodontal, and other bacterial infections. Therefore, early, continual, and comprehensive participation by the client in her prenatal care is important. Recent evidence has shown a link between periodontal infections and preterm labor. Researchers recommend regular dental care before and during pregnancy, oral assessment as a routine part of prenatal care, and scrupulous oral hygiene to prevent periodontal infections. DIF: Cognitive Level: Remember REF: p. 760 TOP: Nursing Process: Assessment MSC: Client Needs: Physiologic Integrity

A nurse caring for a pregnant client should be aware that the U.S. birth rate shows what trend? a.Births to unmarried women are more likely to have less favorable outcomes. b.Birth rates for women 40 to 44 years of age are declining. c.Cigarette smoking among pregnant women continues to increase. d.Rates of pregnancy and abortion among teenagers are lower in the United States than in any other industrialized country.

ANS: A LBW infants and preterm births are more likely because of the large number of teenagers in the unmarried group. Birth rates for women in their early 40s continue to increase. Fewer pregnant women smoke. Teen pregnancy and abortion rates are higher in the United States than in any other industrial country.

An infant at 26 weeks of gestation arrives intubated from the delivery room. The nurse weighs the infant, places him under the radiant warmer, and attaches him to the ventilator at the prescribed settings. A pulse oximeter and cardiorespiratory monitor are placed. The pulse oximeter is recording oxygen saturations of 80%. The prescribed saturations are 92%. What is the nurse's most appropriate action at this time? a.Listening to breath sounds, and ensuring the patency of the endotracheal tube, increasing oxygen, and notifying a physician b.Continuing to observe and making no changes until the saturations are 75% c.Continuing with the admission process to ensure that a thorough assessment is completed d.Notifying the parents that their infant is not doing well

ANS: A Listening to breath sounds and ensuring the patency of the endotracheal tube, increasing oxygen, and notifying a physician are appropriate nursing interventions to assist in optimal oxygen saturation of the infant. Oxygen saturation should be maintained above 92%, and oxygenation status of the infant is crucial. The nurse should delay other tasks to stabilize the infant. Notifying the parents that the infant is not doing well is not an appropriate action. Further assessment and intervention are warranted before determining fetal status.

10. A nurse caring for a pregnant client should be aware that the U.S. birth rate shows what trend? a. Births to unmarried women are more likely to have less favorable outcomes. b. Birth rates for women 40 to 44 years of age are declining. c. Cigarette smoking among pregnant women continues to increase. d. The rates of pregnancy and abortion among teens are lower in the United States than in any other industrialized country.

ANS: A Low-birth-weight infants and preterm birth are more likely because of the large number of teenagers in the unmarried group. Birth rates for women in their early 40s continue to increase. Fewer pregnant women smoke. Teen pregnancy and abortion rates are higher in the United States than in any other industrial country.

A pregnant woman was admitted for induction of labor at 43 weeks of gestation with sure dates. A nonstress test (NST) in the obstetrician's office revealed a nonreactive tracing. On artificial rupture of membranes, thick meconium-stained fluid was noted. What should the nurse caring for the infant after birth anticipate? a.Meconium aspiration, hypoglycemia, and dry, cracked skin b.Excessive vernix caseosa covering the skin, lethargy, and RDS c.Golden yellow to green-stained skin and nails, absence of scalp hair, and an increased amount of subcutaneous fat d.Hyperglycemia, hyperthermia, and an alert, wide-eyed appearance

ANS: A Meconium aspiration, hypoglycemia, and dry, cracked skin are consistent with a postmature infant. Excessive vernix caseosa, lethargy, and RDS are consistent with a very premature infant. The skin may be meconium stained, but the infant will most likely have long hair and decreased amounts of subcutaneous fat. Postmaturity with a nonreactive NST is indicative of hypoxia. Signs and symptoms associated with fetal hypoxia are hypoglycemia, temperature instability, and lethargy.

On vaginal examination of a 30-year-old woman, the nurse documents the following findings: profuse, thin, grayish-white vaginal discharge with a "fishy" odor and complaints of pruritus. Based upon these findings, which condition would the nurse suspect? a.Bacterial vaginosis b.Candidiasis c.Trichomoniasis d.Gonorrhea

ANS: A Most women with bacterial vaginosis complain of a characteristic "fishy" odor. The discharge is usually profuse, thin, and has a white, gray, or milky color. Some women may also experience mild irritation or pruritus. The discharge associated with candidiasis is thick, white, and lumpy and resembles cottage cheese. Trichomoniasis may be asymptomatic, but women commonly have a characteristic yellow-to-green, frothy, mucopurulent, copious, and malodorous discharge. Women with gonorrhea are often asymptomatic. Although they may have a purulent endocervical discharge, the discharge is usually minimal or absent.

14. A nurse providing couplet care should understand the issue of nipple confusion. In which situation might this condition occur? a. Breastfeeding babies receive supplementary bottle feedings. b. Baby is too abruptly weaned. c. Pacifiers are used before breastfeeding is established. d. Twins are breastfed together.

ANS: A Nipple confusion can result when babies go back and forth between bottles and breasts, especially before breastfeeding is established in 3 to 4 weeks; bottle feeding and breastfeeding require different skills. Abrupt weaning can be distressing to the mother and/or baby but should not lead to nipple confusion. Pacifiers used before breastfeeding is established can be disruptive but do not lead to nipple confusion. Breastfeeding twins require some logistical adaptations but should not lead to nipple confusion.

24. What is a maternal indication for the use of vacuum-assisted birth? a. Wide pelvic outlet b. Maternal exhaustion c. History of rapid deliveries d. Failure to progress past station 0

ANS: B A mother who is exhausted may be unable to assist with the expulsion of the fetus. The client with a wide pelvic outlet will likely not require vacuum extraction. With a rapid delivery, vacuum extraction is not necessary. A station of 0 is too high for a vacuum-assisted birth. DIF: Cognitive Level: Understand REF: p. 786 TOP: Nursing Process: Assessment MSC: Client Needs: Physiologic Integrity

During the assessment of a preterm infant, the nurse notices continued respiratory distress even though oxygen and ventilation have been provided. In this situation, which condition should the nurse suspect? a.Hypovolemia and/or shock b.Excessively cool environment c.Central nervous system (CNS) injury d.Pending renal failure

ANS: A Other symptoms might include hypotension, prolonged capillary refill, and tachycardia, followed by bradycardia. Intervention is necessary. Preterm infants are susceptible to temperature instability. The goal of thermoregulation is to provide a neutral thermal environment. Hypoglycemia is likely to occur if the infant is attempting to conserve heat. CNS injury is manifested by hyperirritability, seizures, and abnormal movements of the extremities. Urine output and testing of specific gravity are appropriate interventions for the infant with suspected renal failure. This neonate is unlikely to be delivered with respiratory distress.

Which symptom described by a client is characteristic of premenstrual syndrome (PMS)? a. I feel irritable and moody a week before my period is supposed to start. b. I have lower abdominal pain beginning on the third day of my menstrual period. c. I have nausea and headaches after my period starts, and they last 2 to 3 days. d. I have abdominal bloating and breast pain after a couple days of my period.

ANS: A PMS is a cluster of physical, psychologic, and behavioral symptoms that begin in the luteal phase of the menstrual cycle and resolve within a couple of days of the onset of menses. Complaints of lower abdominal pain, nausea and headaches, and abdominal bloating all are associated with PMS; however, the timing reflected is inaccurate.

A 21yearold client complains of severe pain immediately after the commencement of her menses. Which gynecologic condition is the most likely cause of this clients presenting complaint? a. Primary dysmenorrhea b. Secondary dysmenorrhea c. Dyspareunia d. Endometriosis

ANS: A Primary dysmenorrhea, or pain during or shortly before menstruation, has a biochemical basis and arises from the release of prostaglandins with menses. Secondary dysmenorrhea develops after the age of 25 years and is usually associated with a pelvic pathologic condition. Dyspareunia, or painful intercourse, is commonly associated with endometriosis. Endometriosis is characterized by endometrial glands and stoma outside of the uterus.

17. Maternity nurses can enhance communication among health care providers by utilizing the SBAR technique. This acronym stands for: a. Situation, Background, Assessment, Recommendation b. Situation, Baseline, Assessment, Recommendation c. Subjective, Background, Analysis, Recommendation d. Subjective, Background, Analysis, Review

ANS: A SBAR is an easy to remember, useful, concrete mechanism for communicating important information that requires a clinician's immediate attention. Baseline is not discussed as part of SBAR. Subjective and analysis are not specific to the SBAR acronym. Subjective, analysis, and review are not specific to the SBAR acronym.

8. A hospital has a number of different perineal pads available for use. A nurse is observed soaking several of them and writing down what she sees. What goal is the nurse attempting to achieve by performing this practice? a. To improve the accuracy of blood loss estimation, which usually is a subjective assessment b. To determine which pad is best c. To demonstrate that other nurses usually underestimate blood loss d. To reveal to the nurse supervisor that one of them needs some time off

ANS: A Saturation of perineal pads is a critical indicator of excessive blood loss; anything done to help in the assessment is valuable. The nurse is noting the saturation volumes and soaking appearances. Instead of determining which pad is best, the nurse is more likely noting saturation volumes and soaking appearances to improve the accuracy of estimated blood loss. Nurses usually overestimate blood loss. Soaking perineal pads and writing down the results does not indicate the need for time off of work.

The health history and physical examination cannot reliably identify all persons infected with HIV or other blood-borne pathogens. Which infection control practice should the nurse use when providing eye prophylaxis to a term newborn? a.Wear gloves. b.Wear mouth, nose, and eye protection. c.Wear a mask. d.Wash the hands after medication administration.

ANS: A Standard Precautions should be consistently used in the care of all persons. Personal protective equipment in the form of gloves should be worn during infant eye prophylaxis, care of the umbilical cord, circumcision site care, diaper changes, handling of colostrum, and parenteral procedures. Masks are worn during respiratory isolation or if the health care practitioner has a cough. Mouth, eye, and nose protection are used to protect the mucous membranes if client-care activities are likely to generate splashes or sprays of body fluids. The hands should be washed both before having contact with the client and after administering medications.

A premature infant with respiratory distress syndrome (RDS) receives artificial surfactant. How does the nurse explain surfactant therapy to the parents? a."Surfactant improves the ability of your baby's lungs to exchange oxygen and carbon dioxide." b."The drug keeps your baby from requiring too much sedation." c."Surfactant is used to reduce episodes of periodic apnea." d."Your baby needs this medication to fight a possible respiratory tract infection."

ANS: A Surfactant can be administered as an adjunct to oxygen and ventilation therapy. With the administration of an artificial surfactant, respiratory compliance is improved until the infant can generate enough surfactant on his or her own. Surfactant has no bearing on the sedation needs of the infant. Surfactant is used to improve respiratory compliance, including the exchange of oxygen and carbon dioxide. The goal of surfactant therapy in an infant with RDS is to stimulate the production of surfactant in the type 2 cells of the alveoli. The clinical presentation of RDS and neonatal pneumonia may be similar. The infant may be started on broad-spectrum antibiotics to treat infection.

A client complains of severe abdominal and pelvic pain around the time of menstruation. This pain has become progressively worse over the last 5 years. She also complains of pain during intercourse and has tried unsuccessfully to become pregnant for the past 18 months. To which condition are these symptoms most likely related? a. Endometriosis b. PMS c. Primary dysmenorrhea d. Secondary dysmenorrhea

ANS: A Symptoms of endometriosis can change over time and may not reflect the extent of the disease. Major symptoms include dysmenorrhea and deep pelvic dyspareunia (painful intercourse). Impaired fertility may result from adhesions caused by endometriosis. Although endometriosis may be associated with secondary dysmenorrhea, it is not a cause of primary dysmenorrhea or PMS. In addition, this woman is complaining of dyspareunia and infertility, which are associated with endometriosis, not with PMS or primary or secondary dysmenorrhea.

5. A primiparous woman is to be discharged from the hospital the following day with her infant girl. Which behavior indicates a need for further intervention by the nurse before the woman can be discharged? a. The woman is disinterested in learning about infant care. b. The woman continues to hold and cuddle her infant after she has fed her. c. The woman reads a magazine while her infant sleeps. d. The woman changes her infants diaper and then shows the nurse the contents of the diaper.

ANS: A The client should be excited, happy, and interested or involved in infant care. A woman who is sad, tearful, or disinterested in caring for her infant may be exhibiting signs of depression or postpartum blues and may require further intervention. Holding and cuddling her infant after feeding is an appropriate parent-infant interaction. Taking time for herself while the infant is sleeping is an appropriate maternal action. Showing the nurse the contents of the diaper is appropriate because the mother is seeking approval from the nurse and notifying the nurse of the infants elimination patterns.

15. The various systems and organs develop at different stages. Which statement is accurate? a. The cardiovascular system is the first organ system to function in the developing human. b. Hematopoiesis originating in the yolk sac begins in the liver at 10 weeks. c. The body changes from straight to C-shaped at 8 weeks. d. The gastrointestinal system is mature at 32 weeks.

ANS: A The heart is developmentally complete by the end of the embryonic stage. Hematopoiesis begins in the liver during the sixth week. The body becomes C-shaped at 21 weeks. The gastrointestinal system is complete at 36 weeks.

25. Which nursing intervention should be immediately performed after the forceps-assisted birth of an infant? a. Assessing the infant for signs of trauma b. Administering prophylactic antibiotic agents to the infant c. Applying a cold pack to the infant's scalp d. Measuring the circumference of the infant's head

ANS: A The infant should be assessed for bruising or abrasions at the site of application, facial palsy, and subdural hematoma. Prophylactic antibiotics are not necessary with a forceps delivery. A cold pack would place the infant at risk for cold stress and is contraindicated. Measuring the circumference of the head is part of the initial nursing assessment. DIF: Cognitive Level: Apply REF: p. 788 TOP: Nursing Process: Implementation MSC: Client Needs: Physiologic Integrity

Which viral sexually transmitted infection is characterized by a primary infection followed by recurrent episodes? a.Herpes simplex virus 2 (HSV-2) b.HPV c.HIV d.CMV

ANS: A The initial HSV genital infection is characterized by multiple painful lesions, fever, chills, malaise, and severe dysuria; it may last 2 to 3 weeks. Recurrent episodes of the HSV infection commonly have only local symptoms that usually are less severe than those of the initial infection. With HPV infection, lesions are a chronic problem. The HIV is a retrovirus. Seroconversion to HIV positivity usually occurs within 6 to 12 weeks after the virus has entered the body. Severe depression of the cellular immune system associated with the HIV infection characterizes AIDS, which has no cure. In most adults, the onset of CMV infection is uncertain and asymptomatic. However, the disease may become a chronic, persistent infection.

The female athlete triad includes which common menstrual disorder? a. Amenorrhea b. Dysmenorrhea c. Menorrhagia d. Metrorrhagia

ANS: A The interrelatedness of disordered eating, amenorrhea, and altered bone mineral density have been described as the female athlete triad. Dysmenorrhea is painful menstruation that begins 2 to 6 months after menarche. Menorrhagia is abnormally profuse or excessive bleeding from the uterus. Metrorrhagia is bleeding between periods and can be caused by progestin injections and implants.

Through the use of social media technology, nurses can link with other nurses who may share similar interests, insights about practice, and advocate for clients. Which factor is the most concerning pitfall for nurses using this technology? a.Violation of client privacy and confidentiality b.Institutions and colleagues who may be cast in an unfavorable light c.Unintended negative consequences for using social media d.Lack of institutional policy governing online contact

ANS: A The most significant pitfall for nurses using this technology is the violation of client privacy and confidentiality. Furthermore, institutions and colleagues can be cast in an unfavorable light with negative consequences for those posting information. Nursing students have been expelled from school and nurses have been fired or reprimanded by their Board of Nursing for injudicious posts. The American Nurses Association has published six principles for social networking and the nurse. All institutions should have policies guiding the use of social media, and the nurse should be familiar with these guidelines.

20. A pregnant woman's amniotic membranes have ruptured. A prolapsed umbilical cord is suspected. What intervention would be the nurse's highest priority? a. Placing the woman in the knee-chest position b. Covering the cord in sterile gauze soaked in saline c. Preparing the woman for a cesarean birth d. Starting oxygen by face mask

ANS: A The woman is assisted into a modified Sims position, Trendelenburg position, or the knee-chest position in which gravity keeps the pressure of the presenting part off the cord. Although covering the cord in sterile gauze soaked saline, preparing the woman for a cesarean, and starting oxygen by face mark are appropriate nursing interventions in the event of a prolapsed cord, the intervention of top priority would be positioning the mother to relieve cord compression. DIF: Cognitive Level: Apply REF: p. 797 TOP: Nursing Process: Implementation MSC: Client Needs: Physiologic Integrity

2. The laboratory results for a postpartum woman are as follows: blood type, A; Rh status, positive; rubella titer, 1:8 (enzyme immunoassay [EIA] 0.8); hematocrit, 30%. How should the nurse best interpret these data? a.Rubella vaccine should be administered. b.Blood transfusion is necessary. c.Rh immune globulin is necessary within 72 hours of childbirth. d. Kleihauer-Betke test should be performed.

ANS: A This clients rubella titer indicates that she is not immune and needs to receive a vaccine. These data do not indicate that the client needs a blood transfusion. Rh immune globulin is indicated only if the client has an Rh-negative status and the infant has an Rh-positive status. A Kleihauer-Betke test should be performed if a large fetomaternal transfusion is suspected, especially if the mother is Rh negative. However, the data provided do not indicate a need for performing this test.

Healthy People 2020 has established national health priorities that focus on a number of maternal-child health indicators. Nurses are assuming greater roles in assessing family health and are providing care across the perinatal continuum. Which of these priorities has made the most significant progress? a.Reduction of fetal deaths and use of prenatal care b.LBW infants and preterm births c.Elimination of health disparities based on race d.Infant mortality and the prevention of birth defects

ANS: A Trends in maternal child health indicate that progress has been made in relation to reduced infant and fetal deaths and increased prenatal care. Notable gaps remain in the rates of LBW infants and preterm births. According to the March of Dimes, persistent disparities still exist between African-Americans and non-Hispanic Caucasians. Many of these negative outcomes are preventable through access to prenatal care and the use of preventive health practices. These preventable negative outcomes demonstrate the need for comprehensive community-based care for all mothers, infants, and families.

The human papillomavirus (HPV), also known as genital warts, affects 79 million Americans, with an estimated number of 14 million new infections each year. The highest rate of infection occurs in young women, ages 20 to 24 years. Prophylactic vaccination to prevent the HPV is now available. Which statement regarding this vaccine is inaccurate? a.Only one vaccine for the HPV is available. b.The vaccine is given in three doses over a 6-month period. c.The vaccine is recommended for both boys and girls. d.Ideally, the vaccine is administered before the first sexual contact.

ANS: A Two vaccines for HPV are available—Cervarix and Gardasil—and other vaccines continue to be investigated. These vaccines protect against HPV types 6, 11, 16, and 18. They are most effective if administered before the first sexual contact. Recommendations are that vaccines be administered to 11- and 12-year-old girls and boys. The vaccine can be given to girls as young as 9 years of age and young women ages 13 to 26 years in three doses over a 6-month period.

What is the most important nursing action in preventing neonatal infection? a.Good handwashing b.Isolation of infected infants c.Separate gown technique d.Standard Precautions

ANS: A Virtually all controlled clinical trials have demonstrated that effective handwashing is responsible for the prevention of nosocomial infection in nursery units. Measures to be taken include Standard Precautions, careful and thorough cleaning, frequent replacement of used equipment, and disposal of excrement and linens in an appropriate manner. Overcrowding must be avoided in nurseries. However, the most important nursing action for preventing neonatal infection is effective handwashing.

An essential component of counseling women regarding safe sex practices includes a discussion regarding avoiding the exchange of body fluids. The most effective physical barrier promoted for the prevention of STIs and HIV is the condom. To educate the client about the use of condoms, which information related to condom use is the most important? a.Strategies to enhance condom use b.Choice of colors and special features c.Leaving the decision up to the male partner d.Places to carry condoms safely

ANS: A When the nurse opens the discussion on safe-sex practices, it gives the woman permission to clear up any concerns or misapprehensions that she may have regarding condom use. The nurse can also suggest ways that the woman can enhance her condom negotiation and communications skills with a sexual partner. These include role-playing, rehearsal, cultural barriers, and situations that place the client at risk. Although women can be taught the differences among condoms, such as size ranges, where to purchase, and price, these features are not as important as negotiating the use of safe sex practices. Although not ideal, women may safely choose to carry condoms in shoes, wallets, or inside their bra. They should be taught to keep the condom away from heat. Although this information is important, it is not relevant if the woman cannot even discuss strategies on how to enhance condom use.

A perimenopausal client has arrived for her annual gynecologic examination. Which preexisting condition would be extremely important for the nurse to identify during a discussion regarding the risks and benefits of hormone therapy? a. Breast cancer b. Vaginal and urinary tract atrophy c. Osteoporosis d. Arteriosclerosis

ANS: A Women with a high risk for breast cancer should be counseled against using estrogen replacement therapy (ERT). Estrogen prevents the atrophy of vaginal and urinary tract tissue and protects against the development of osteoporosis. Estrogen also has a favorable effect on circulating lipids, reducing low-density lipoprotein (LDL) and total cholesterol levels and increasing high-density lipoprotein (HDL) levels. It also has a direct antiatherosclerotic effect on the arteries.

21. As relates to the father's acceptance of the pregnancy and preparation for childbirth, the maternity nurse should know that: a. The father goes through three phases of acceptance of his own. b. The father's attachment to the fetus cannot be as strong as that of the mother because it does not start until after birth. c. In the last 2 months of pregnancy, most expectant fathers suddenly get very protective of their established lifestyle and resist making changes to the home. d. Typically men remain ambivalent about fatherhood right up to the birth of their child.

ANS: A A father typically goes through three phases of development to reach acceptance of fatherhood: the announcement phase, the moratorium phase, and the focusing phase. The father-child attachment can be as strong as the mother-child relationship and can also begin during pregnancy. In the last 2 months of pregnancy, many expectant fathers work hard to improve the environment of the home for the child. Typically the expectant father's ambivalence ends by the first trimester, and he progresses to adjusting to the reality of the situation and then to focusing on his role.

17. Which statement about pregnancy is accurate? a. A normal pregnancy lasts about 10 lunar months. b. A trimester is one third of a year. c. The prenatal period extends from fertilization to conception. d. The estimated date of confinement (EDC) is how long the mother will have to be bedridden after birth.

ANS: A A lunar month lasts 28 days, or 4 weeks. Pregnancy spans 9 calendar months but 10 lunar months. A trimester is one third of a normal pregnancy, or about 13 to 14 weeks. The prenatal period covers the full course of pregnancy (prenatal means before birth). The EDC is now called the EDB, or estimated date of birth. It has nothing to do with the duration of bed rest.

19. When caring for a newly delivered woman, what is the best measure to prevent abdominal distention after a cesarean birth? a. Rectal suppositories b. Early and frequent ambulation c. Tightening and relaxing abdominal muscles d. Carbonated beverages

ANS: B Activity will aid the movement of accumulated gas in the gastrointestinal tract. Rectal suppositories can be helpful after distention occurs; however, they do not prevent it. Ambulation is the best prevention. Carbonated beverages may increase distention.

29. With regard to medications, herbs, shots, and other substances normally encountered by pregnant women, the maternity nurse should be aware that: a. Both prescription and over-the-counter (OTC) drugs that otherwise are harmless can be made hazardous by metabolic deficiencies of the fetus. b. The greatest danger of drug-caused developmental deficits in the fetus is seen in the final trimester. c. Killed-virus vaccines (e.g., tetanus) should not be given during pregnancy, but live-virus vaccines (e.g., measles) are permissible. d. No convincing evidence exists that secondhand smoke is potentially dangerous to the fetus.

ANS: A Both prescription and OTC drugs that otherwise are harmless can be made hazardous by metabolic deficiencies of the fetus. This is especially true for new medications and combinations of drugs. The greatest danger of drug-caused developmental defects exists in the interval from fertilization through the first trimester, when a woman may not realize that she is pregnant. Live-virus vaccines should be part of postpartum care; killed-virus vaccines may be administered during pregnancy. Secondhand smoke is associated with fetal growth restriction and increases in infant mortality.

Which statement regarding emergency contraception is correct? a. Emergency contraception requires that the first dose be taken within 120 hours of unprotected intercourse. b. Emergency contraception may be taken right after ovulation. c. Emergency contraception has an effectiveness rate in preventing pregnancy of approximately 50%. d. Emergency contraception is commonly associated with the side effect of menorrhagia.

ANS: A Emergency contraception should be taken as soon as possible or within 72 hours of unprotected intercourse to prevent pregnancy. If taken before ovulation, follicular development is inhibited, which prevents ovulation. The risk of pregnancy is reduced by as much as 75%. The most common side effect of postcoital contraception is nausea.

Which term best describes the conscious decision concerning deciding when to conceive or avoid pregnancy as opposed to the intentional prevention of pregnancy during intercourse? a. Family planning b. Birth control c. Contraception d. Assisted reproductive therapy

ANS: A Family planning is the process of deciding when and if to have children. Birth control is the device and/or practice used to reduce the risk of conceiving or bearing children. Contraception is the intentional prevention of pregnancy during sexual intercourse. Assisted reproductive therapy is one of several possible treatments for infertility.

2. Prenatal testing for human immunodeficiency virus (HIV) is recommended for: a. All women, regardless of risk factors. b. A woman who has had more than one sexual partner. c. A woman who has had a sexually transmitted infection. d. A woman who is monogamous with her partner.

ANS: A Testing for the antibody to HIV is strongly recommended for all pregnant women. A HIV test is recommended for all women, regardless of risk factors. Women who test positive for HIV can be treated, reducing the risk of transmission to the fetus.

A woman has chosen the calendar method of conception control. Which is the most important action the nurse should perform during the assessment process in preparation to discuss the implementation of this method? a. Obtain a history of the woman's menstrual cycle lengths for the past 6 to 12 months. b. Determine the client's weight gain and loss pattern for the previous year. c. Examine skin pigmentation and hair texture for hormonal changes. d. Explore the client's previous experiences with conception control.

ANS: A The calendar method of conception control is based on the number of days in each cycle, counting from the first day of menses. The fertile period is determined after the lengths of menstrual cycles have been accurately recorded for 6 months. Weight gain or loss may be partly related to hormonal fluctuations, but it has no bearing on the use of the calendar method. Integumentary changes may be related to hormonal changes, but they are not indicators for use of the calendar method. Exploring previous experiences with conception control may demonstrate client understanding and compliancy, but these experiences are not the most important aspect to assess for the discussion of the calendar method.

13. Which behavior indicates that a woman is "seeking safe passage" for herself and her infant? a. She keeps all prenatal appointments. c. She drives her car slowly. b. She "eats for two." d. She wears only low-heeled shoes.

ANS: A The goal of prenatal care is to foster a safe birth for the infant and mother. Although eating properly, driving carefully, and using proper body mechanics all are healthy measures that a mother can take, obtaining prenatal care is the optimal method for providing safety for both herself and her baby

25. While teaching the expectant mother about personal hygiene during pregnancy, maternity nurses should be aware that: a. Tub bathing is permitted even in late pregnancy unless membranes have ruptured. b. The perineum should be wiped from back to front. c. Bubble bath and bath oils are permissible because they add an extra soothing and cleansing action to the bath. d. Expectant mothers should use specially treated soap to cleanse the nipples.

ANS: A The main danger from taking baths is falling in the tub. The perineum should be wiped from front to back. Bubble baths and bath oils should be avoided because they may irritate the urethra. Soap, alcohol, ointments, and tinctures should not be used to cleanse the nipples because they remove protective oils. Warm water is sufficient.

Which comment by the nurse would be most helpful in instruct the woman on how best to reduce her risk of developing toxic shock syndrome (TSS)? (Select all that apply.) a. "You should always leave your diaphragm in place for at 6 hours after intercourse." b. "You should remove your diaphragm right after intercourse to prevent TSS." c. "You can use your diaphragm during your menstrual cycle as long as you clean it well." d. "Make sure you don't leave your diaphragm in for longer than 24 hours" e. Report a fever of 38.4 0 C to your health care provider immediately."

ANS: A The nurse should instruct the client on the proper use and removal of the diaphragm and include the danger signs of TSS. The diaphragm must remain against the cervix for 6 to 8 hours to prevent pregnancy, but it should not remain in place longer than 8 hours to avoid the risk of TSS. The diaphragm should not be used during menses. A fever of 38.4 0 C or higher could be a symptom of TSS and should be reported immediately.

Importantly, the nurse must be aware of which information related to the use of intrauterine devices (IUDs)? a. Return to fertility can take several weeks after the device is removed. b. Copper IUDs can serve as an emergency contraception under certain situations. c. IUDs offer the same protection against STIs as the diaphragm. d. Consent forms are not needed for IUD insertion.

ANS: B The woman has up to 5 days to insert the IUD after unprotected sex. The return to fertility is immediate after the removal of the IUD. IUDs offer no protection against STIs. A consent form is required for insertion, as is a negative pregnancy test.

3. The Baby Friendly Hospital Initiative endorsed by the World Health Organization (WHO) and the United Nations Childrens Fund (UNICEF) was founded to encourage institutions to offer optimal levels of care for lactating mothers. Which actions are included in the Ten Steps to Successful Breastfeeding for Hospitals? (Select all that apply.) a. Give newborns no food or drink other than breast milk. b. Have a written breastfeeding policy that is communicated to all staff members. c. Help mothers initiate breastfeeding within hour of childbirth. d. Give artificial teats or pacifiers as necessary. e. Return infants to the nursery at night.

ANS: A, B, C No artificial teats or pacifiers (also called dummies or soothers) should be given to breastfeeding infants. Although pacifiers have been linked to a reduction in SIDs, they should not be introduced until the infant is 3 to 4 weeks old and breastfeeding is well established. No other food or drink should be given to the newborn unless medically indicated. The breastfeeding policy should be routinely communicated to all health care staff members. All staff should be trained in the skills necessary to maintain this policy. Breastfeeding should be initiated within hour of childbirth, and all mothers need to be shown how to maintain lactation even if separated from their babies. The facility should practice rooming in and keep mothers and babies together 24 hours a day.

1. Which actions are examples of appropriate techniques to wake a sleepy infant for breastfeeding? (Select all that apply.) a. Unwrapping the infant b. Changing the diaper c. Talking to the infant d. Slapping the infants hands and feet e. Applying a cold towel to the infants abdomen

ANS: A, B, C Unwrapping the infant, changing the diaper, and talking to the infant are appropriate techniques to use when trying to wake a sleepy infant. The parent can rub, never slap, the infants hands or feet to wake the infant. Applying a cold towel to the infants abdomen may lead to cold stress in the infant. The parent may want to apply a cool cloth to the infants face to wake the infant.

34. Signs and symptoms that a woman should report immediately to her health care provider include (Select all that apply): a. Vaginal bleeding. b. Rupture of membranes. c. Heartburn accompanied by severe headache. d. Decreased libido. e. Urinary frequency.

ANS: A, B, C Vaginal bleeding, rupture of membranes, and severe headaches all are signs of potential complications in pregnancy. Clients should be advised to report these signs to the health care provider. Decreased libido and urinary frequency are common discomforts of pregnancy that do not require immediate health care interventions.

2. Which methods help alleviate the problems associated with access to health care for the maternity client? Choose all that apply. a. Provide transportation to prenatal visits. b. Provide child care so that a pregnant woman may keep prenatal visits. c. Increase the number of providers that will care for Medicaid clients. d. Provide low-cost or no-cost health care insurance. e. Provide job training.

ANS: A, B, C, D Lack of transportation to visits, lack of child care, access to skilled obstetric providers, and lack of affordable health insurance are prohibitive factors associated with lack of prenatal care. Although job training may result in employment and income, the likelihood of significant changes during the time frame of the pregnancy is remote.

1. Congenital disorders refer to those conditions that are present at birth. These disorders may be inherited and caused by environmental factors or maternal malnutrition. Toxic exposures have the greatest effect on development between 15 and 60 days of gestation. For the nurse to be able to conduct a complete assessment of the newly pregnant client, he or she should be knowledgeable regarding known human teratogens, which include: a. Infections b. Radiation c. Maternal conditions d. Drugs e. Chemicals

ANS: A, B, C, D, E Exposure to radiation and a number of infections may result in profound congenital deformities. These include but are not limited to varicella, rubella, syphilis, parvovirus, cytomegalovirus (CMV), and toxoplasmosis. Certain maternal conditions such as diabetes and phenylketonuria (PKU) may also affect organs and other parts of the embryo during this developmental period. Drugs such as antiseizure medication and some antibiotics as well as chemicals including lead, mercury, tobacco, and alcohol also may result in structural and functional abnormalities.

2. What are the complications and risks associated with cesarean births? (Select all that apply.) a. Pulmonary edema b. Wound dehiscence c. Hemorrhage d. Urinary tract infections e. Fetal injuries

ANS: A, B, C, D, E Pulmonary edema, wound dehiscence, hemorrhage, urinary tract infections, and fetal injuries are possible complications and risks associated with cesarean births. DIF: Cognitive Level: Understand REF: p. 790 TOP: Nursing Process: Evaluation MSC: Client Needs: Physiologic Integrity

A woman who has undergone an induced abortion be instructed to return to the emergency department when what situation exists? (Select all that apply.) a. Fever higher than 38° C b. Chills c. Foul-smelling vaginal discharge d. Bleeding greater than 2 pads in 2 hours e. Abdominal tenderness and or pain

ANS: A, B, C, D, E The client should report to a health care facility for any of the following symptoms: fever higher than 38° C, chills, bleeding more than two saturated pads in 2 hours or heavy bleeding lasting for days, foul-smelling discharge, abdominal tenderness or pain, and cramping or backache.

3. Women who are obese are at risk for several complications during pregnancy and birth. Which of these would the nurse anticipate with an obese client? (Select all that apply.) a. Thromboembolism b. Cesarean birth c. Wound infection d. Breech presentation e. Hypertension

ANS: A, B, C, E A breech presentation is not a complication of pregnancy or birth for the client who is obese. Venous thromboembolism is a known risk for obese women. Therefore, the use of thromboembolism-deterrent (TED) hose and sequential compression devices may help decrease the chance for clot formation. Women should also be encouraged to ambulate as soon as possible. In addition to having an increased risk for complications with a cesarean birth, in general, obese women are also more likely to require an emergency cesarean birth. Many obese women have a pannus (i.e., large roll of abdominal fat) that overlies a lower transverse incision made just above the pubic area. The pannus causes the area to remain moist, which encourages the development of infection. Obese women are more likely to begin pregnancy with comorbidities such as hypertension and type 2 diabetes. DIF: Cognitive Level: Analyze REF: p. 778 TOP: Nursing Process: Assessment MSC: Client Needs: Health Promotion and Maintenance

4. If a woman is at risk for thrombus and is not ready to ambulate, which nursing intervention would the nurse use? (Select all that apply.) a. Putting her in antiembolic stockings (thromboembolic deterrent [TED] hose) and/or sequential compression device (SCD) boots b. Having her flex, extend, and rotate her feet, ankles, and legs c. Having her sit in a chair d. Immediately notifying the physician if a positive Homans sign occurs e. Promoting bed rest

ANS: A, B, D Sitting immobile in a chair does not help; bed exercise and prophylactic footwear might. TED hose and SCD boots are recommended. The client should be encouraged to ambulate with assistance, not remain in bed. Bed exercises are useful. A positive Homans sign (calf muscle pain or warmth, redness, tenderness) requires the physicians immediate attention.

A 23-year-old primiparous client with inconsistent prenatal care is admitted to the hospital's maternity unit in labor. The client states that she has tested positive for the HIV. She has not undergone any treatment during her pregnancy. The nurse understands that the risk of perinatal transmission can be significantly decreased by a number of prophylactic interventions. Which interventions should be included in the plan of care? a.Intrapartum treatment with antiviral medications b.Cesarean birth c.Postpartum treatment with antiviral medications d.Avoidance of breastfeeding e.Pneumococcal, HBV, and Haemophilus influenzae vaccine

ANS: A, B, D The prophylactic measures of prenatal antiviral use, elective cesarean birth, and formula feeding reduce the transmission of the HIV to as low as 1% to 2%. The client who refuses a cesarean birth should be given intravenous antiviral therapy during labor. Ideally, medications should be given prenatally. Administration of antiviral drugs in the postpartum period will not reduce transmission to the infant. All women who are HIV positive should be encouraged to receive these immunizations. They will not reduce the risk of perinatal transmission.

The nurse, responsible for providing health teaching regarding the side effects of combined oral contraceptives (COCs), should discuss what issues? (Select all that apply.) a. Gallbladder disease b. Myocardial infarction c. Hypotension d. Breast tenderness e. Dry skin and scalp

ANS: A, B, D Serious side effects include stroke, myocardial infarction, hypertension, gallbladder disease, and liver tumors. More common side effects include nausea, breast tenderness, fluid retention, increased appetite, and oily skin and scalp

The Patient Protection and Affordable Care Act (ACA) was signed into law by President Obama in early 2010. The Act provides some immediate benefits, and other provisions will take place over the next several years. The practicing nurse should have a thorough understanding of how these changes will benefit his or her clients. Which outcomes are goals of the ACA? (Select all that apply.) a.Insurance affordability b.Improve public health c.Treatment of illness d.Elimination of Medicare and Medicaid e.Cost containment

ANS: A, B, E The ACA goals are to make insurance more affordable, contain costs, and strengthen Medicare and Medicaid. The Act contains provisions that promote the prevention of illness and improve access to public health. The ultimate goal of the Act is to improve the quality of care for all Americans while reducing waste, fraud, and abuse of the current system.

A group of infections known collaboratively as TORCH infections are capable of crossing the placenta and causing serious prenatal effects on the fetus. Which infections are included in this group of organisms? (Select all that apply.) a.Toxoplasmosis b.Other infections c.Roseola d.Clostridium e.Herpes simplex

ANS: A, B, E Toxoplasmosis, other infections, rubella virus, CMV, and HSV are collectively known as TORCH infections. Generally, all TORCH infections produce influenza-like symptoms in the mother; however, fetal effects are generally more serious.

35. A woman has just moved to the United States from Mexico. She is 3 months pregnant and has arrived for her first prenatal visit. During her assessment interview, you discover that she has not had any immunizations. Which immunizations should she receive at this point in her pregnancy (Select all that apply)? a. Tetanus b. Diphtheria c. Chickenpox d. Rubella e. Hepatitis B

ANS: A, B, E Immunization with live or attenuated live viruses is contraindicated during pregnancy because of potential teratogenicity. Vaccines consisting of killed viruses may be used. Immunizations that may be administered during pregnancy include tetanus, diphtheria, recombinant hepatitis B, and rabies vaccines. Live-virus vaccines include those for measles (rubeola and rubella), chickenpox, and mumps.

Which suggestions are appropriate for a client who complains of hot flashes? (Select all that apply.) a. Avoid caffeine. b. Drink a glass of wine to relax. c. Wear layered clothing. d. Drink ice water. e. Drink warm beverages for their calming effect.

ANS: A, C Layered clothing allows the client to remove layers if a hot flash occurs. Ice water may help alleviate the hot flashes. Slow, deep breathing is also beneficial. Avoid triggers such as exercising on hot days, spicy foods, hot beverages, and alcohol.

2. Which practices contribute to the prevention of postpartum infection? (Select all that apply.) a. Not allowing the mother to walk barefoot at the hospital b. Educating the client to wipe from back to front after voiding c. Having staff members with conditions such as strep throat, conjunctivitis, and diarrhea stay home d. Instructing the mother to change her perineal pad from front to back each time she voids or defecates e. Not permitting visitors with cough or colds to enter the postpartum unit

ANS: A, C, D Proper perineal care helps prevent infection and aids in the healing process. Educating the woman to wipe from front to back (urethra to anus) after voiding or defecating is a simple first step. Walking barefoot and getting back into bed can contaminate the linens. Clients should wear shoes or slippers. Staff members with infections need to stay home until they are no longer contagious. The client should also wash her hands before and after these functions. Visitors with any signs of illness should not be allowed entry to the postpartum unit.

The nurse should be familiar with the use of the five Ps as a tool for evaluating risk behaviors for STIs and the HIV. Which components would the nurse include in her use of the five Ps as an assessment tool? (Select all that apply.) a.Number of partners b.Level of physical activity c.Prevention of pregnancy d.Protection from STIs e.Past history

ANS: A, C, D, E Level of physical activity is not a component of this assessment. The five Ps include partners, prevention of pregnancy, protection from STIs, understanding of sexual practices, and past history.

4. The induction of labor is considered an acceptable obstetric procedure if it is in the best interest to deliver the fetus. The charge nurse on the labor and delivery unit is often asked to schedule clients for this procedure and therefore must be cognizant of the specific conditions appropriate for labor induction. What are appropriate indications for induction? (Select all that apply?) a. Rupture of membranes at or near term b. Convenience of the woman or her physician c. Chorioamnionitis (inflammation of the amniotic sac) d. Postterm pregnancy e. Fetal death

ANS: A, C, D, E The conditions listed are all acceptable indications for induction. Other conditions include intrauterine growth restriction (IUGR), maternal-fetal blood incompatibility, hypertension, and placental abruption. Elective inductions for the convenience of the woman or her provider are not recommended; however, they have become commonplace. Factors such as rapid labors and living a long distance from a health care facility may be valid reasons in such a circumstance. Elective delivery should not occur before 39 weeks of completed gestation. DIF: Cognitive Level: Apply REF: p. 790 TOP: Nursing Process: Planning MSC: Client Needs: Physiologic Integrity

5. The AAP recommends pasteurized donor milk for preterm infants if the mothers own milk in not available. Which statements regarding donor milk and milk banking are important for the nurse to understand and communicate to her client? (Select all that apply.) a. All milk bank donors are screened for communicable diseases. b. Internet milk sharing is an acceptable source for donor milk. c. Donor milk may be given to transplant clients. d. Donor milk is used in neonatal intensive care units (NICUs) for severely low-birth-weight infants only. e. Donor milk may be used for children with immunoglobulin A (IgA) deficiencies.

ANS: A, C, E Because of the antiinfective and growth promotion properties for donor milk, donor milk is highly recommended for preterm and sick infants, as well as for term newborns. Human donor milk has also been used for older children with short gut syndrome, immunodeficiencies, metabolic disorders, or congenital anomalies. Human donor milk has also been used in the adult populationposttransplant clients and for those with colitis, ulcers, or cirrhosis of the liver. Some mothers acquire milk through Internet-based or community-based milk sharing. The U.S. Food and Drug Administration (FDA) has issued a warning regarding this practice. Samples of milk from these sources are higher in contaminants and infectious disease. A milk bank that belongs to the Human Milk Banking Association of North America should always be used for donor milk. All donors are scrupulously screened, and the milk is tested to determine its safety for use.

1. Many new mothers experience some type of nipple pain during the first weeks of initiating breastfeeding. Should this pain be severe or persistent, it may discourage or inhibit breastfeeding altogether. Which factors might contribute to this pain? (Select all that apply.) a. Improper feeding position b. Large-for-gestational age infant c. Fair skin d. Progesterone deficiency e. Flat or retracted nipples

ANS: A, C, E Nipple lesions may manifest as chapped, cracked, bleeding, sore, erythematous, edematous, or blistered nipples. Factors that contribute to nipple pain include improper positioning or a failure to break suction before removing the baby from the breast. Flat or retracted nipples along with the use of nipple shields, breast shells, or plastic breast pads also contribute to nipple pain. Women with fair skin are more likely to develop sore and cracked nipples. Preventing nipple soreness is preferable to treating soreness after it appears. Vigorous feeding may be a contributing factor, which may be the case with any size infant, not just infants who are large for gestational age. Estrogen or dietary deficiencies can contribute to nipple soreness.

A client has requested information regarding alternatives to hormonal therapy for menopausal symptoms. Which current information should the nurse provide to the client? (Select all that apply.) a. Soy b. Vitamin C c. Vitamin K d. Vitamin E e. Vitamin A

ANS: A, D Both soy and vitamin E have been reported to help alleviate menopausal symptoms, and both are readily available in food sources. Vitamin E can be also be taken as a supplement. Vitamins C, K, and A have no apparent effect on menopausal symptoms.

6. Nurses play a critical role in educating parents regarding measures to prevent infant abduction. Which instructions contribute to infant safety and security? (Select all that apply.) a. The mother should check the photo identification (ID) of any person who comes to her room. b. The baby should be carried in the parents arms from the room to the nursery. c. Because of infant security systems, the baby can be left unattended in the clients room. d. Parents should use caution when posting photographs of their infant on the Internet. e. The mom should request that a second staff member verify the identity of any questionable person.

ANS: A, D, E Nurses must discuss infant security precautions with the mother and her family because infant abduction continues to be a concern. The mother should be taught to check the identity of any person who comes to remove the baby from her room. Hospital personnel usually wear picture identification patches. On some units, staff members also wear matching scrubs or special badges that are unique to the perinatal unit. As a rule, the baby is never carried in arms between the mothers room and the nursery, but rather the infant is always wheeled in a bassinet. The infant should never be left unattended, even if the facility has an infant security system. Parents should be instructed to use caution when posting photographs of their new baby on the Internet and on other public forums.

A premature infant never seems to sleep longer than an hour at a time. Each time a light is turned on, an incubator closes, or people talk near her crib, she wakes up and inconsolably cries until held. What is the correct nursing diagnosis beginning with "ineffective coping, related to"? a.Severe immaturity b.Environmental stress c.Physiologic distress d.Behavioral responses

ANS: B "Ineffective coping, related to environmental stress" is the most appropriate nursing diagnosis for this infant. Light and sound are known adverse stimuli that add to an already stressed premature infant. The nurse must closely monitor the environment for sources of overstimulation. Although the infant may be severely immature in this case, she is responding to environmental stress. Physiologic distress is the response to environmental stress. The result is stress cues such as increased metabolic rate, increased oxygen and caloric use, and depression of the immune system. The infant's behavioral response to the environmental stress is crying. The appropriate nursing diagnosis reflects the cause of this response.

3. A woman gave birth 48 hours ago to a healthy infant girl. She has decided to bottle feed. During the assessment, the nurse notices that both breasts are swollen, warm, and tender on palpation. Which guidance should the nurse provide to the client at this time? a. Run warm water on her breasts during a shower. b. Apply ice to the breasts for comfort. c. Express small amounts of milk from the breasts to relieve the pressure. d. Wearing a loose-fitting bra to prevent nipple irritation.

ANS: B Applying ice packs and cabbage leaves to the breasts for comfort is an appropriate intervention for treating engorgement in a mother who is bottle feeding. The ice packs should be applied for 15 minutes on and 45 minutes off to avoid rebound engorgement. A bottle-feeding mother should avoid any breast stimulation, including pumping or expressing milk. A bottle-feeding mother should continuously wear a well-fitted support bra or breast binder for at least the first 72 hours after giving birth. A loose-fitting bra will not aid lactation suppression. Furthermore, the shifting of the bra against the breasts may stimulate the nipples and thereby stimulate lactation.

5. It is important for the nurse to understand that the placenta: a. Produces nutrients for fetal nutrition b. Secretes both estrogen and progesterone c. Forms a protective, impenetrable barrier to microorganisms such as bacteria and viruses d. Excretes prolactin and insulin

ANS: B As one of its early functions, the placenta acts as an endocrine gland, producing four hormones necessary to maintain the pregnancy and support the embryo and fetus: human chorionic gonadotropin (hCG), human placental lactogen (hPL), estrogen, and progesterone. The placenta does not produce nutrients. It functions as a means of metabolic exchange between the maternal and fetal blood supplies. Many bacteria and viruses can cross the placental membrane. One of the early functions of the placenta is as an endocrine gland that produces four hormones (hCG, hPL, estrogen, and progesterone) necessary to maintain the pregnancy and support the embryo and fetus.

The Centers for Disease Control and Prevention (CDC) recommends which therapy for the treatment of the HPV? a.Miconazole ointment b.Topical podofilox 0.5% solution or gel c.Two doses of penicillin administered intramuscularly (IM) d.Metronidazole by mouth

ANS: B Available treatments are imiquimod, podophyllin, and podofilox. Miconazole ointment is used to treat athlete's foot. Penicillin IM is used to treat syphilis. Metronidazole is used to treat bacterial vaginosis.

14. A number of methods can be used for inducing labor. Which cervical ripening method falls under the category of mechanical or physical? a. Prostaglandins are used to soften and thin the cervix. b. Labor can sometimes be induced with balloon catheters or laminaria tents. c. Oxytocin is less expensive and more effective than prostaglandins but creates greater health risks. d. Amniotomy can be used to make the cervix more favorable for labor.

ANS: B Balloon catheters or laminaria tents are mechanical means of ripening the cervix. Ripening the cervix, making it softer and thinner, increases the success rate of induced labor. Prostaglandin E1 is less expensive and more effective than oxytocin but carries a greater risk. Amniotomy is the artificial rupture of membranes, which is used to induce labor only when the cervix is already ripe. DIF: Cognitive Level: Apply REF: p. 781 TOP: Nursing Process: Planning MSC: Client Needs: Health Promotion and Maintenance

10. The nurse who elects to work in the specialty of obstetric care must have the ability to distinguish between preterm birth, preterm labor, and low birth weight. Which statement regarding this terminology is correct? a. Terms preterm birth and low birth weight can be used interchangeably. b. Preterm labor is defined as cervical changes and uterine contractions occurring between 20 and 37 weeks of gestation. c. Low birth weight is a newborn who weighs below 3.7 pounds. d. Preterm birth rate in the United States continues to increase.

ANS: B Before 20 weeks of gestation, the fetus is not viable (miscarriage); after 37 weeks, the fetus can be considered term. Although these terms are used interchangeably, they have different meanings: preterm birth describes the length of gestation (before 37 weeks), regardless of the newborn's weight; low birth weight describes only the infant's weight at the time of birth (2500 g or less), whenever it occurs. Low birth weight is anything below 2500 g or approximately pounds. In 2011, the preterm birth rate in the United States was 11.7 %; it has dropped every year since 2008. DIF: Cognitive Level: Understand REF: p. 759 TOP: Nursing Process: Assessment MSC: Client Needs: Health Promotion and Maintenance

Which statement best exemplifies contemporary maternity nursing? a.Use of midwives for all vaginal deliveries b.Family-centered care c.Free-standing birth clinics d.Physician-driven care

ANS: B Contemporary maternity nursing focuses on the family's needs and desires. Fathers, partners, grandparents, and siblings may be present for the birth and participate in activities such as cutting the baby's umbilical cord. Both midwives and physicians perform vaginal deliveries. Free-standing clinics are an example of alternative birth options. Contemporary maternity nursing is driven by the relationship between nurses and their clients.

In appraising the growth and development potential of a preterm infant, the nurse should be cognizant of the information that is best described in which statement? a.Tell the parents that their child will not catch up until approximately age 10 years (for girls) to age 12 years (for boys). b.Correct for milestones, such as motor competencies and vocalizations, until the child is approximately 2 years of age. c.Know that the greatest catch-up period is between 9 and 15 months postconceptual age. d.Know that the length and breadth of the trunk is the first part of the infant to experience catch-up growth.

ANS: B Corrections are made with a formula that adds gestational age and postnatal age. Whether a girl or boy, the infant experiences catch-up body growth during the first 2 to 3 years of life. Maximum catch-up growth occurs between 36 and 40 weeks of postconceptual age. The head is the first to experience catch-up growth.

12. Maternity nursing care that is based on knowledge gained through research and clinical trials is: a. Derived from the Nursing Intervention Classification b. Known as evidence-based practice c. At odds with the Cochrane School of traditional nursing d. An outgrowth of telemedicine

ANS: B Evidence-based practice is based on knowledge gained from research and clinical trials. The Nursing Intervention Classification is a method of standardizing language and categorizing care. Dr. Cochrane systematically reviewed research trials and is part of the evidence-based practice movement. Telemedicine uses communication technologies to support health care.

Which statement best describes maternity nursing care that is based on knowledge gained through research and clinical trials? a.Maternity nursing care is derived from the Nursing Intervention Classification. b.Maternity nursing care is known as evidence-based practice. c.Maternity nursing care is at odds with the Cochrane School of traditional nursing. d.Maternity nursing care is an outgrowth of telemedicine.

ANS: B Evidence-based practice is based on knowledge gained from research and clinical trials. The Nursing Intervention Classification is a method of standardizing language and categorizing care. Dr. Cochrane systematically reviewed research trials and is part of the evidence-based practice movement. Telemedicine uses communication technologies to support health care.

16. Which statement concerning neurologic and sensory development is accurate? a. Brain waves have been recorded on an electroencephalogram as early as the end of the first trimester (12 weeks). b. Fetuses respond to sound by 24 weeks and can be soothed by the sound of the mother's voice. c. Eyes are first receptive to light at 34 to 36 weeks. d. At term, the fetal brain is at least one third the size of an adult brain.

ANS: B Hearing develops early and is fully developed at birth. Brain waves have been recorded at week 8. Eyes are receptive to light at 28 weeks. The fetal brain is about one fourth the size of an adult brain.

To assist a client in managing the symptoms of PMS, what should the nurse recommend based on current evidence? a. Diet with more body-building and energy foods, such as carbohydrates b. Herbal therapies, yoga, and massage c. Antidepressants for symptom control d. Discouraging the use of diuretics

ANS: B Herbal therapies, yoga, and massage have been reported to have a beneficial effect on the symptoms of PMS. Limiting red meat, simple carbohydrates, caffeinated beverages, and alcohol improves the diet and may mitigate symptoms. Medication is usually begun only if lifestyle changes fail to provide significant relief. Natural diuretics may help reduce fluid retention.

With regard to infants who are SGA (small for gestational age) and intrauterine growth restriction (IUGR), the nurse should be aware of which information? a.In the first trimester, diseases or abnormalities result in asymmetric IUGR. b.Infants with asymmetric IUGR have the potential for normal growth and development. c.In asymmetric IUGR, weight is slightly larger than SGA, whereas length and head circumference are somewhat less than SGA. d.Symmetric IUGR occurs in the later stages of pregnancy.

ANS: B IUGR is either symmetric or asymmetric. The symmetric form occurs in the first trimester; infants who are SGA have reduced brain capacity. The asymmetric form occurs in the later stages of pregnancy. Weight is less than the 10th percentile; head circumference is greater than the 10th percentile. Infants with asymmetric IUGR have the potential for normal growth and development.

28. Which instruction should the nurse provide to reduce the risk of nipple trauma? a. Limit the feeding time to less than 5 minutes. b. Position the infant so the nipple is far back in the mouth. c. Assess the nipples before each feeding. d. Wash the nipples daily with mild soap and water.

ANS: B If the infants mouth does not cover as much of the areola as possible, the pressure during sucking will be applied to the nipple, thus causing trauma to the area. Stimulating the breast for less than 5 minutes will not produce the extra milk the infant may need and will also limit access to the higher-fat hindmilk. Assessing the nipples for trauma is important; however, this action alone will not prevent sore nipples. Soap can be drying to the nipples and should be avoided during breastfeeding.

A 25-year-old single woman comes to the gynecologist's office for a follow-up visit related to her abnormal Papanicolaou (Pap) smear. The test revealed that the client has the HPV. The woman asks, "What is that? Can you get rid of it?" Which is the best response for the nurse to provide? a."It's just a little lump on your cervix. We can just freeze it off." b."HPV stands for 'human papillomavirus.' It is a sexually transmitted infection that may lead to cervical cancer." c."HPV is a type of early human immunodeficiency virus. You will die from this." d."You probably caught this from your current boyfriend. He should get tested for this."

ANS: B Informing the client about STIs and the risks involved with the HPV is important. The health care team has a duty to provide proper information to the client, including information related to STIs and the fact that although the HPV and HIV are both viruses that can be sexually transmitted, they are not the same virus. The onset of the HPV can be insidious. Often STIs go unnoticed. Abnormal bleeding is frequently the initial symptom. The client may have had the HPV before her current boyfriend. The nurse should make no deductions from this limited information.

23. The breastfeeding mother should be taught a safe method to remove the breast from the babys mouth. Which suggestion by the nurse is most appropriate? a. Slowly remove the breast from the babys mouth when the infant has fallen asleep and the jaws are relaxed. b. Break the suction by inserting your finger into the corner of the infants mouth. c. A popping sound occurs when the breast is correctly removed from the infants mouth. d. Elicit the Moro reflex to wake the baby and remove the breast when the baby cries.

ANS: B Inserting a finger into the corner of the babys mouth between the gums to break the suction avoids trauma to the breast. The infant who is sleeping may lose grasp on the nipple and areola, resulting in chewing on the nipple that makes it sore. A popping sound indicates improper removal of the breast from the babys mouth and may cause cracks or fissures in the breast. Most mothers prefer the infant to continue to sleep after the feeding. Gentle wake-up techniques are recommended.

9. An important development that concerns maternity nursing is integrative health care, which: a. Seeks to provide the same health care for all racial and ethnic groups b. Blends complementary and alternative therapies with conventional Western treatment c. Focuses on the disease or condition rather than the client's background d. Has been mandated by Congress

ANS: B Integrative health care tries to mix the old with the new at the discretion of the client and health care providers. Integrative health care is a blending of new and traditional practices. Integrative health care focuses on the whole person, not just the disease or condition. U.S. law supports complementary and alternative therapies but does not mandate them.

Which condition is the least likely cause of amenorrhea in a 17yearold client? a. Anatomic abnormalities b. Type 1 diabetes mellitus c. Obesity d. Pregnancy

ANS: C A moderately obese adolescent (20% to 30% above ideal weight) may have early onset menstruation. Girls who regularly exercise before menarche can have delayed onset of menstruation to age 18 years. Anatomic abnormalities are a possible cause of amenorrhea. Type 1 diabetes mellitus is a possible cause of amenorrhea. Pregnancy is the most common cause of amenorrhea.

While interviewing a 31-year-old woman before her routine gynecologic examination, the nurse collects data about the clients recent menstrual cycles. Which statement by the client should prompt the nurse to collect further information? a. My menstrual flow lasts 5 to 6 days. b. My flow is very heavy. c. I have had a small amount of spotting midway between my periods for the past 2 months. d. The length of my menstrual cycles varies from 26 to 29 days.

ANS: B Menorrhagia is defined as excessive menstrual bleeding, either in duration or in amount. Heavy bleeding can have many causes. The amount of bleeding and its effect on daily activities should be evaluated. A menstrual flow that lasts 5 to 6 days is a normal finding. Mittlestaining, a small amount of bleeding or spotting that occurs at the time of ovulation (14 days before the onset of the next menses), is considered normal. During her reproductive years, a woman may have physiologic variations in her menstrual cycle. Variations in the length of a menstrual cycle are considered normal.

1. A newly married couple plans to use natural family planning. It is important for them to know how long an ovum can live after ovulation. The nurse knows that teaching is effective when the couple responds that an ovum is considered fertile for: a. 6 to 8 hours b. 24 hours c. 2 to 3 days d. 1 week

ANS: B Ova are considered fertile for about 24 hours after ovulation. Ova are considered fertile for much longer than 6 to 8 hours. Most remain fertile for 24 hours. Ova do not remain fertile for 2 to 3 days. If unfertilized by a sperm, the ovum degenerates and is reabsorbed. Ova do not remain viable for 1 week. After 24 hours the ovum degenerates and is reabsorbed.

Necrotizing enterocolitis (NEC) is an inflammatory disease of the gastrointestinal mucosa. The signs of NEC are nonspecific. What are generalized signs and symptoms of this condition? a.Hypertonia, tachycardia, and metabolic alkalosis b.Abdominal distention, temperature instability, and grossly bloody stools c.Hypertension, absence of apnea, and ruddy skin color d.Scaphoid abdomen, no residual with feedings, and increased urinary output

ANS: B Some generalized signs of NEC include decreased activity, hypotonia, pallor, recurrent apnea and bradycardia, decreased oxygen saturation values, respiratory distress, metabolic acidosis, oliguria, hypotension, decreased perfusion, temperature instability, cyanosis, abdominal distention, residual gastric aspirates, vomiting, grossly bloody stools, abdominal tenderness, and erythema of the abdominal wall. The infant may display hypotonia, bradycardia, and metabolic acidosis.

1. A new mother recalls from prenatal class that she should try to feed her newborn daughter when she exhibits feeding readiness cues rather than waiting until the baby is frantically crying. Which feeding cue would indicate that the baby is ready to eat? a. Waves her arms in the air b. Makes sucking motions c. Has the hiccups d. Stretches out her legs straight

ANS: B Sucking motions, rooting, mouthing, and hand-to-mouth motions are examples of feeding readiness cues. Waving her arms in the air, having the hiccups, and stretching out her extremities are not typical feeding readiness cues.

2. Which nursing intervention is paramount when providing care to a client with preterm labor who has received terbutaline? a. Assess deep tendon reflexes (DTRs). b. Assess for dyspnea and crackles. c. Assess for bradycardia. d. Assess for hypoglycemia.

ANS: B Terbutaline is a beta2-adrenergic agonist that affects the mother's cardiopulmonary and metabolic systems. Signs of cardiopulmonary decompensation include adventitious breath sounds and dyspnea. An assessment for dyspnea and crackles is important for the nurse to perform if the woman is taking magnesium sulfate. Assessing DTRs does not address the possible respiratory side effects of using terbutaline. Since terbutaline is a beta2-adrenergic agonist, it can lead to hyperglycemia, not hypoglycemia. Beta2-adrenergic agonist drugs cause tachycardia, not bradycardia. DIF: Cognitive Level: Analyze REF: pp. 767-768 TOP: Nursing Process: Assessment MSC: Client Needs: Physiologic Integrity

17. A new father is ready to take his wife and newborn son home. He proudly tells the nurse who is discharging them that within the next week he plans to start feeding the infant cereal between breastfeeding sessions. Which information should the nurse provide regarding this feeding plan? a. Feeding solid foods before your son is 4 to 6 months old may decrease your sons intake of sufficient calories. b. Feeding solid foods between breastfeeding sessions before your son is 4 to 6 months old will lead to an early cessation of breastfeeding. c. Your feeding plan will help your son sleep through the night. d. Feeding solid foods before your son is 4 to 6 months old will limit his growth.

ANS: B The introduction of solid foods before the infant is 4 to 6 months of age may result in overfeeding and decreased intake of breast milk. The belief that feeding solid foods helps infants sleep through the night is untrue. The proper balance of carbohydrate, protein, and fat for an infant to grow properly is in the breast milk or formula.

A newborn was admitted to the neonatal intensive care unit (NICU) after being delivered at 29 weeks of gestation to a 28-year-old multiparous, married, Caucasian woman whose pregnancy was uncomplicated until the premature rupture of membranes and preterm birth. The newborn's parents arrive for their first visit after the birth. The parents walk toward the bedside but remain approximately 5 feet away from the bed. What is the nurse's most appropriate action? a.Wait quietly at the newborn's bedside until the parents come closer. b.Go to the parents, introduce him or herself, and gently encourage them to meet their infant. Explain the equipment first, and then focus on the newborn. c.Leave the parents at the bedside while they are visiting so that they have some privacy. d.Tell the parents only about the newborn's physical condition and caution them to avoid touching their baby.

ANS: B The nurse is instrumental in the initial interactions with the infant. The nurse can help the parents see the infant rather than focus on the equipment. The importance and purpose of the apparatus that surrounds their infant also should be explained to them. Parents often need encouragement and recognition from the nurse to acknowledge the reality of the infant's condition. Parents need to see and touch their infant as soon as possible to acknowledge the reality of the birth and the infant's appearance and condition. Encouragement from the nurse is instrumental in this process. Telling the parents to avoid touching their baby is inappropriate and unhelpful.

25. How many kilocalories per kilogram (kcal/kg) of body weight does a breastfed term infant require each day? a. 50 to 65 b. 75 to 90 c. 95 to 110 d. 150 to 200

ANS: C For the first 3 months, the infant needs 110 kcal/kg/day. At ages 3 to 6 months, the requirement is 100 kcal/kg/day. This level decreases slightly to 95 kcal/kg/day from 6 to 9 months and increases again to 100 kcal/kg/day until the baby reaches 12 months.

13. With regard to the structure and function of the placenta, the maternity nurse should be aware that: a. As the placenta widens, it gradually thins to allow easier passage of air and nutrients b. As one of its early functions, the placenta acts as an endocrine gland c. The placenta is able to keep out most potentially toxic substances, such as cigarette smoke, to which the mother is exposed d. Optimal blood circulation is achieved through the placenta when the woman is lying on her back or standing

ANS: B The placenta produces four hormones necessary to maintain the pregnancy. The placenta widens until week 20 and continues to grow thicker. Toxic substances such as nicotine and carbon monoxide readily cross the placenta into the fetus. Optimal circulation occurs when the woman is lying on her side.

14. During a prenatal intake interview, the client informs the nurse that she would prefer a midwife to provide both her care during pregnancy and deliver her infant. What information is most appropriate for the nurse to share with this client? a. Midwifery care is only available to clients who are uninsured because their services are less expensive than an obstetrician. Costs are often lower than an obstetric provider. b. The client will receive fewer interventions during the birth process. c. She should be aware that midwives are not certified. d. Her delivery can take place only at home or in a birth center.

ANS: B This client will be able to participate actively in all decisions related to the birth process and is likely to receive fewer interventions during the birth process. Midwifery services are available to all low risk pregnant women, regardless of the type of insurance they have. Midwifery care in all developed countries is strictly regulated by a governing body that ensures that core competencies are met. In the United States, this body is the American College of Nurse-Midwives (ACNM). Midwives can provide care and delivery at home, in freestanding birth centers, and in community and teaching hospitals.

During a prenatal intake interview, the client informs the nurse that she would prefer a midwife to provide both her care during pregnancy and deliver her infant. Which information is most appropriate for the nurse to share with this client? a.Midwifery care is only available to clients who are uninsured because their services are less expensive than an obstetrician. b.She will receive fewer interventions during the birth process. c.She should be aware that midwives are not certified. d.Her delivery can take place only at home or in a birth center.

ANS: B This client will be able to participate actively in all decisions related to the birth process and is likely to receive fewer interventions during the birth process. Midwifery services are available to all low-risk pregnant women, regardless of the type of insurance they have. Midwifery care in all developed countries is strictly regulated by a governing body to ensure that core competencies are met. In the United States, this body is the American College of Nurse-Midwives (ACNM). Midwives can provide care and delivery at home, in freestanding birth centers, and in community and teaching hospitals.

8. A pregnant woman at 29 weeks of gestation has been diagnosed with preterm labor. Her labor is being controlled with tocolytic medications. She asks when she might be able to go home. Which response by the nurse is mostaccurate? a. "After the baby is born." b. "When we can stabilize your preterm labor and arrange home health visits." c. "Whenever your physician says that it is okay." d. "It depends on what kind of insurance coverage you have."

ANS: B This client's preterm labor is being controlled with tocolytics. Once she is stable, home care may be a viable option for this type of client. Care of a client with preterm labor is multidisciplinary and multifactorial; the goal is to prevent delivery. In many cases, this goal may be achieved at home. Managed care may dictate an earlier hospital discharge or a shift from hospital to home care. Insurance coverage may be one factor in client care, but ultimately, client safety remains the most important factor. DIF: Cognitive Level: Apply REF: p. 765 TOP: Nursing Process: Planning MSC: Client Needs: Health Promotion and Maintenance

10. What information should the nurse understand fully regarding rubella and Rh status? a. Breastfeeding mothers cannot be vaccinated with the live attenuated rubella virus. b. Women should be warned that the rubella vaccination is teratogenic and that they must avoid pregnancy for at least 1 month after vaccination. c. Rh immunoglobulin is safely administered intravenously because it cannot harm a nursing infant. d. Rh immunoglobulin boosts the immune system and thereby enhances the effectiveness of vaccinations.

ANS: B Women should understand that they must practice contraception for at least 1 month after being vaccinated. Because the live attenuated rubella virus is not communicable in breast milk, breastfeeding mothers can be vaccinated. Rh immunoglobulin is administered intramuscular (IM); it should never be administered to an infant. Rh immunoglobulin suppresses the immune system and therefore might thwart the rubella vaccination.

23. With regard to the initial physical examination of a woman beginning prenatal care, maternity nurses should be cognizant of: a. Only women who show physical signs or meet the sociologic profile should be assessed for physical abuse. b. The woman should empty her bladder before the pelvic examination is performed. c. The distribution, amount, and quality of body hair are of no particular importance. d. The size of the uterus is discounted in the initial examination.

ANS: B An empty bladder facilitates the examination; this is also an opportunity to get a urine sample easily for a number of tests. All women should be assessed for a history of physical abuse, particularly because the likelihood of abuse increases during pregnancy. Noting body hair is important because body hair reflects nutritional status, endocrine function, and hygiene. Particular attention is paid to the size of the uterus because it is an indication of the duration of gestation.

A woman who has a seizure disorder and takes barbiturates and phenytoin sodium daily asks the nurse about the pill as a contraceptive choice. What is the nurse's best response? a. "Oral contraceptives are a highly effective method, but they have some side effects." b. "Your current medications will reduce the effectiveness of the pill." c. "Oral contraceptives will reduce the effectiveness of your seizure medication." d. "The pill is a good choice for a woman of your age and with your personal history."

ANS: B Because the liver metabolizes oral contraceptives, their effectiveness is reduced when they are simultaneously taken with anticonvulsants. Stating that the pill is an effective birth control method with side effects is a true statement, but this response is not the most appropriate. The anticonvulsant reduces the effectiveness of the pill, not the other way around. Stating that the pill is a good choice for a woman of her age and personal history does not teach the client that the effectiveness of the pill may be reduced because of her anticonvulsant therapy.

14. A 3-year-old girl's mother is 6 months pregnant. What concern is this child likely to verbalize? a. How the baby will "get out" c. Whether her mother will die b. What the baby will eat d. What color eyes the baby has

ANS: B By age 3 or 4, children like to be told the story of their own beginning and accept its comparison with the present pregnancy. They like to listen to the fetal heartbeat and feel the baby move. Sometimes they worry about how the baby is being fed and what it wears. School-age children take a more clinical interest in their mother's pregnancy and may want to know, "How did the baby get in there?" and "How will it get out?" Whether her mother will die does not tend to be the focus of a child's questions about the impending birth of a sibling. The baby's eye color does not tend to be the focus of children's questions about the impending birth of a sibling.

Which action should the nurse first take when meeting with a new client to discuss contraception? a. Obtain data about the frequency of coitus. b. Determine the woman's level of knowledge concerning contraception.. c. Assess the woman's willingness to touch her genitals and cervical mucus. d. Evaluate the woman's contraceptive life plan.

ANS: B Determining the woman's level of knowledge concerning contraception and her commitment to any particular method is the primary step of this nursing assessment and necessary before completing the process and moving on to a nursing diagnosis. Once the client's level of knowledge is determined, the nurse can interact with the woman to compare options, reliability, cost, comfort level, protection from STIs, and her partner's willingness to participate. Although important, obtaining data about the frequency of coitus is not the first action that the nurse should undertake when completing an assessment. Data should include not only the frequency of coitus but also the number of sexual partners, level of contraceptive involvement, and the partner's objections. Assessing the woman's willingness to touch herself is a key factor for the nurse to discuss should the client express an interest in using one of the fertility awareness methods of contraception. The nurse must be aware of the client's plan regarding whether she is attempting to prevent conception, delay conception, or conceive.

12. During the first trimester, a woman can expect which of the following changes in her sexual desire? a. An increase, because of enlarging breasts b. A decrease, because of nausea and fatigue c. No change d. An increase, because of increased levels of female hormones

ANS: B Maternal physiologic changes such as breast enlargement, nausea, fatigue, abdominal changes, perineal enlargement, leukorrhea, pelvic vasocongestion, and orgasmic responses may affect sexuality and sexual expression. Libido may be depressed in the first trimester but often increases during the second and third trimesters. During pregnancy, the breasts may become enlarged and tender; this tends to interfere with coitus, decreasing the desire to engage in sexual activity.

18. In understanding and guiding a woman through her acceptance of pregnancy, a maternity nurse should be aware that: a. Nonacceptance of the pregnancy very often equates to rejection of the child. b. Mood swings most likely are the result of worries about finances and a changed lifestyle as well as profound hormonal changes. c. Ambivalent feelings during pregnancy usually are seen only in emotionally immature or very young mothers. d. Conflicts such as not wanting to be pregnant or childrearing and career-related decisions need not be addressed during pregnancy because they will resolve themselves naturally after birth.

ANS: B Mood swings are natural and are likely to affect every woman to some degree. A woman may dislike being pregnant, refuse to accept it, and still love and accept the child. Ambivalent feelings about pregnancy are normal for mature or immature women, younger or older women. Conflicts such as not wanting to be pregnant or childrearing and career-related decisions need to be resolved. The baby ends the pregnancy but not all the issues.

A male client asks the nurse why it is better to purchase condoms that are not lubricated with nonoxynol-9 (a common spermicide). Which response by the nurse is the most accurate? a. "The lubricant prevents vaginal irritation." b. "Nonoxynol-9 does not provide protection against STIs as originally thought" c. "The additional lubrication improves sex." d. "Nonoxynol-9 improves penile sensitivity."

ANS: B Nonoxynol-9 does not provide protection against STIs as originally thought; it has also been linked to an increase in the transmission of the HIV and can cause genital lesions. Nonoxynol-9 may cause vaginal irritation, has no effect on the quality of sexual activity, and has no effect on penile sensitivity.

8. A pregnant woman at 18 weeks of gestation calls the clinic to report that she has been experiencing occasional backaches of mild-to-moderate intensity. The nurse would recommend that she: a. Do Kegel exercises. c. Use a softer mattress. b. Do pelvic rock exercises. d. Stay in bed for 24 hours.

ANS: B Pelvic rock exercises may help stretch and strengthen the abdominal and lower back muscles and relieve low back pain. Kegel exercises increase the tone of the pelvic area, not the back. A softer mattress may not provide the support needed to maintain proper alignment of the spine and may contribute to back pain. Stretching and other exercises to relieve back pain should be performed several times a day.

1. The nurse caring for a newly pregnant woman would advise her that ideally prenatal care should begin: a. Before the first missed menstrual period. b. After the first missed menstrual period. c. After the second missed menstrual period. d. After the third missed menstrual period.

ANS: B Prenatal care ideally should begin soon after the first missed menstrual period. Regular prenatal visits offer opportunities to ensure the health of the expectant mother and her infant.

Which information is most important to provide to the client interested in using the lactational amenorrhea method for contraception? a. LAM is effective until the infant is 9 months of age. b. This popular method of birth control works best if the mother is exclusively breastfeeding. c. Its typical failure rate is 5%. d. Feeding intervals should be 6 hours during the day.

ANS: B The LAM works best if the mother is exclusively or almost exclusively breastfeeding. Disruption of the breastfeeding pattern increases the risk of pregnancy. After the infant is 6 months of age or menstrual flow has resumed, effectiveness decreases. The typical failure rate is 1% to 2%. Feeding intervals should be no greater than 4 hours during the day and 6 hours at night.

Which condition is the most life-threatening virus to the fetus and neonate? a.Hepatitis A virus (HAV) b.Herpes simplex virus (HSV) c.Hepatitis B virus (HBV) d.Cytomegalovirus (CMV)

ANS: C HBV is the most life-threatening viral condition to the fetus and neonate. HAV is not the most threatening to the fetus nor is HSV the most threatening to the neonate. Although serious, CMV is not the most life-threatening viral condition to the fetus.

8. Significant advances have been made with most reproductive technologies. Which improvement has resulted in increased success related to preimplantation genetic diagnosis? a. Embryos are transferred at the cleavage stage. b. Embryos are transferred at the blastocyst stage. c. More than two embryos can be transferred at a time. d. Two cells are removed from each embryo.

ANS: B Preimplantation genetic diagnosis can be performed on a single cell removed from each embryo after 3 to 4 days. With the availability of extended culture mediums, embryos are transferred at the blastocyst stage (day 5), which increases the chance of a live birth, compared with the older practice of transferring embryos at the cleavage stage (day 3). No more than two embryos should be transferred at a time.

1. Which test is performed around the time of ovulation to diagnose the basis of infertility? a. Hysterosalpingogram b. Vaginal ultrasonography c. Laparoscopy d. Follicle-stimulating hormone (FSH) level

ANS: B Ultrasonography is performed around the time of ovulation to assess pelvic structures for abnormalities, to verify follicular development, and to assess the thickness of the endometrium. A hysterosalpingogram is scheduled 2 to 5 days after menstruation to avoid flushing a potentially fertilized ovum out through a uterine tube into the peritoneal cavity. Laparoscopy is usually scheduled early in the menstrual cycle. Hormone analysis is performed to assess endocrine function of the hypothalamic-pituitary-ovarian axis when menstrual cycles are absent or irregular.

11. Women who have undergone an oophorectomy, have ovarian failure, or a genetic defect may be eligible to receive donor oocytes (eggs). Which statements regarding oocyte donation are accurate? (Select all that apply.) a. Donor is inseminated with semen from the parent. b. Donor eggs can be fertilized with the male partner's sperm. c. Donors are under 35 years of age. d. Recipient undergoes hormonal stimulation. e. Ovum is placed into a surrogate.

ANS: B, C, D Oocyte donation is usually provided by healthy women under the age of 35 years, who are recruited and paid to undergo ovarian stimulation and oocyte retrieval. The donor eggs are fertilized in a laboratory with the male partner's sperm. The woman undergoes hormonal stimulation to allow the development of the uterine lining. Embryos are then transferred. A donor that is inseminated with the male partner's semen or receives the fertilized ovum and then carries it to gestation is known as a surrogate mother.

Which statements regarding the HPV are accurate? (Select all that apply.)HPV infections: a.are thought to be less common in pregnant women than in women who are not pregnant. b.are thought to be more common in pregnant women than in women who are not pregnant. c.were previously called genital warts. d.were previously called herpes. e.may cause cancer.

ANS: B, C, E HPV infections are thought to be more common in pregnant women than in women who are not pregnant, with an increase in incidence from the first trimester to the third trimester. HPV, formerly called venereal or genital warts, is an STI with more than 30 known serotypes, several of which are associated with cervical cancer.

5. Indications for a primary cesarean birth are often nonrecurring. Therefore, a woman who has had a cesarean birth with a low transverse scar may be a candidate for vaginal birth after cesarean (VBAC). Which clients would beless likely to have a successful VBAC? (Select all that apply.) a. Lengthy interpregnancy interval b. African-American race c. Delivery at a rural hospital d. Estimated fetal weight <4000 g e. Maternal obesity (BMI >30)

ANS: B, C, E Indications for a low success rate for a VBAC delivery include a short interpregnancy interval, non-Caucasian race, gestational age longer than 40 weeks, maternal obesity, preeclampsia, fetal weight greater than 4000 g, and delivery at a rural or private hospital. DIF: Cognitive Level: Understand REF: p. 794 TOP: Nursing Process: Assessment MSC: Client Needs: Physiologic Integrity

While educating the client regarding the risks and benefits of a vasectomy, which information should the nurse include? (Select all that apply.) a. Sterilization should be performed under general anesthesia. b. Pain, bleeding, and infection are possible complications. c. For several months, pregnancy may still be possible. d. Vasectomy may affect potency. e. Secondary sex characteristics are unaffected.

ANS: B, C, E Vasectomy is the most commonly used procedure for male sterilization and is performed on an outpatient basis under local anesthesia. Pain, bleeding, swelling, and infection are considered complications. Reversal is generally unsuccessful; however, it may take several weeks to months for all sperm to be cleared from the sperm ducts. Another form of contraception is necessary until the sperm counts are zero. Vasectomy has no effect on potency, and secondary sex characteristics are not affected.

MULTIPLE RESPONSE 1. The nurse recognizes that uterine hyperstimulation with oxytocin requires emergency interventions. What clinical cues alert the nurse that the woman is experiencing uterine hyperstimulation? (Select all that apply.) a. Uterine contractions lasting <90 seconds and occurring >2 minutes in frequency b. Uterine contractions lasting >90 seconds and occurring <2 minutes in frequency c. Uterine tone <20 mm Hg d. Uterine tone >20 mm Hg e. Increased uterine activity accompanied by a nonreassuring FHR and pattern

ANS: B, D, E Uterine contractions that occur less frequently than 2 minutes apart and last longer than 90 seconds, a uterine tone over 20 mm Hg, and a nonreassuring FHR and pattern are indications of uterine hyperstimulation with oxytocin administration. Uterine contractions that occur more frequently than 2 minutes apart and last less than 90 seconds are the expected goal of oxytocin induction. A uterine tone less than 20 mm Hg is normal. DIF: Cognitive Level: Analyze REF: p. 785 TOP: Nursing Process: Implementation MSC: Client Needs: Physiologic Integrity

Greater than one third of women in the United States are now obese (body mass index [BMI] of 30 or greater). Less than one quarter of women in Canada exhibit the same BMI. Obesity in the pregnant woman increases both maternal medical risk factors and negative outcomes for the infant. The nurse is about to perform an assessment on a client who is 28 weeks pregnant and has a BMI of 35. What are the most frequently reported complications for which the nurse must be alert while assessing this client? (Select all that apply.) a.Potential miscarriage b.Diabetes c.Fetal death in utero d.Decreased fertility e.Hypertension

ANS: B, E The two most frequently reported maternal medical risk factors associated with obesity are hypertension associated with pregnancy and diabetes. Decreased fertility, miscarriage, fetal death, and congenital anomalies are also associated with obesity. These clients often experience longer hospital stays and increased use of health services.

14. With regard to the development of the respiratory system, maternity nurses should be aware that: a. The respiratory system does not begin developing until after the embryonic stage b. The infant's lungs are considered mature when the L/S ratio is 1:1, at about 32 weeks c. Maternal hypertension can reduce maternal-placental blood flow, accelerating lung maturity d. Fetal respiratory movements are not visible on ultrasound scans until at least 16 weeks

ANS: C A reduction in placental blood flow stresses the fetus, increases blood levels of corticosteroids, and thus accelerates lung maturity. Development of the respiratory system begins during the embryonic phase and continues into childhood. The infant's lungs are mature when the L/S ratio is 2:1, at about 35 weeks. Lung movements have been seen on ultrasound scans at 11 weeks.

20. While discussing the societal impacts of breastfeeding, the nurse should be cognizant of the benefits and educate the client accordingly. Which statement as part of this discussion would be incorrect? a. Breastfeeding requires fewer supplies and less cumbersome equipment. b. Breastfeeding saves families money. c. Breastfeeding costs employers in terms of time lost from work. d. Breastfeeding benefits the environment.

ANS: C Actually, less time is lost to work by breastfeeding mothers, in part because infants are healthier. Breastfeeding is convenient because it does not require cleaning or transporting bottles and other equipment. It saves families money because the cost of formula far exceeds the cost of extra food for the lactating mother. Breastfeeding uses a renewable resource; it does not need fossil fuels, advertising, shipping, or disposal.

3. A pregnant woman at 25 weeks of gestation tells the nurse that she dropped a pan last week and her baby jumped at the noise. Which response by the nurse is most accurate? a. "That must have been a coincidence; babies can't respond like that." b. "The fetus is demonstrating the aural reflex." c. "Babies respond to sound starting at about 24 weeks of gestation." d. "Let me know if it happens again; we need to report that to your midwife."

ANS: C Babies respond to sound starting at about 24 weeks of gestation. Fetuses can respond to sound by 24 weeks or so. Acoustic stimulations can evoke a fetal heart rate response. There is no such thing as an aural reflex. Stating that a report needs to be made if the fetus moves again at a noise gives the impression that something is wrong.

27. As the nurse assists a new mother with breastfeeding, the client asks, If formula is prepared to meet the nutritional needs of the newborn, what is in breast milk that makes it better? What is the nurses best response? a. More calories b. Essential amino acids c. Important immunoglobulins d. More calcium

ANS: C Breast milk contains immunoglobulins that protect the newborn against infection. The calorie count of formula and breast milk is approximately the same. All the essential amino acids are in both formula and breast milk; however, the concentrations may differ. Calcium levels are higher in formula than in breast milk, which can cause an excessively high renal solute load if the formula is not properly diluted.

21. What is the primary purpose for the use of tocolytic therapy to suppress uterine activity? a. Drugs can be efficaciously administered up to the designated beginning of term at 37 weeks gestation. b. Tocolytic therapy has no important maternal (as opposed to fetal) contraindications. c. The most important function of tocolytic therapy is to provide the opportunity to administer antenatal glucocorticoids. d. If the client develops pulmonary edema while receiving tocolytic therapy, then intravenous (IV) fluids should be given.

ANS: C Buying time for antenatal glucocorticoids to accelerate fetal lung development may be the best reason to use tocolytic therapy. Once the pregnancy has reached 34 weeks, however, the risks of tocolytic therapy outweigh the benefits. Important maternal contraindications to tocolytic therapy exist. Tocolytic-induced edema can be caused by IV fluids. DIF: Cognitive Level: Comprehend REF: p. 766 TOP: Nursing Process: Planning MSC: Client Needs: Physiologic Integrity

What is the drug of choice for the treatment of gonorrhea? a.Penicillin G b.Tetracycline c.Ceftriaxone d.Acyclovir

ANS: C Ceftriaxone is effective for the treatment of all gonococcal infections. Penicillin is used to treat syphilis. Tetracycline is prescribed for chlamydial infections. Acyclovir is used to treat herpes genitalis.

Which sexually transmitted infection (STI) is the most commonly reported in American women? a.Gonorrhea b.Syphilis c.Chlamydia d.Candidiasis

ANS: C Chlamydia is the most common and fastest spreading STI among American women, with an estimated 3 million new cases each year. Infection rates are two and a half times that of men. Gonorrhea is probably the oldest communicable disease in the United States and second to Chlamydia in reported conditions. Syphilis is the earliest described STI. Candidiasis is a relatively common fungal infection.

4. Alternative and complementary therapies: a. Replace conventional Western modalities of treatment b. Are used by only a small number of American adults c. Allow for more client autonomy d. Focus primarily on the disease an individual is experiencing

ANS: C City, county, and state health departments provide annual reports of births and deaths. Maternal and infant death rates are particularly important because they reflect health outcomes that may be preventable. Infant mortality continues to be a concern in all populations. Alternative and complementary therapies are part of an integrative approach to health care. An increasing number of American adults are seeking alternative and complementary health care options. Alternative healing modalities offer a holistic approach to health, focusing on the whole person and not just the disease.

An infant is to receive gastrostomy feedings. Which intervention should the nurse institute to prevent bloating, gastrointestinal reflux into the esophagus, vomiting, and respiratory compromise? a.Rapid bolusing of the entire amount in 15 minutes b.Warm cloths to the abdomen for the first 10 minutes c.Slow, small, warm bolus feedings over 30 minutes d.Cold, medium bolus feedings over 20 minutes

ANS: C Feedings by gravity are slowly accomplished over 20- to 30-minute periods to prevent adverse reactions. Rapid bolusing would most likely lead to the adverse reactions listed. Temperature stability in the newborn is critical. Applying warm cloths to the abdomen would not be appropriate because the environment is not thermoregulated. In addition, abdominal warming is not indicated with feedings of any kind. Small feedings at room temperature are recommended to prevent adverse reactions.

With regard to an eventual discharge of the high-risk newborn or the transfer of the newborn to a different facility, which information is essential to provide to the parents? a.Infants stay in the NICU until they are ready to go home. b.Once discharged to go home, the high-risk infant should be treated like any healthy term newborn. c.Parents of high-risk infants need special support and detailed contact information. d.If a high-risk infant and mother need to be transferred to a specialized regional center, then waiting until after the birth and until the infant is stabilized is best.

ANS: C High-risk infants can cause profound parental stress and emotional turmoil. Parents need support, special teaching, and quick access to various resources available to help them care for their baby. Parents and their high-risk infant should get to spend a night or two in a predischarge room, where care for the infant is provided away from the NICU. Simply because high-risk infants are eventually discharged does not mean they are normal, healthy babies. Follow-up by specialized practitioners is essential. Ideally, the mother and baby are transported with the fetus in utero; this reduces neonatal morbidity and mortality.

16. In order to ensure client safety, the practicing nurse must have knowledge of The Joint Commission's current "Do Not Use" list of abbreviations. Which term is acceptable for use regarding medication administration? a. q.o.d. or Q.O.D b. MSO4 or MgSO4 c. International Unit d. Lack of a leading zero

ANS: C I.U. and i.u. are no longer acceptable because they could be misread as "I.V." or the number 10. Q.O.D. should be written out as "every other day." The period after the "Q" could be mistaken for an "I" and the "o" could also be mistaken for an "i." It is too easy to confuse one medication for another. These medications are used for very different purposes and could put a client at risk for an adverse outcome. They should be written as morphine sulfate andmagnesium sulfate. The decimal point should never be missed before a number, to avoid confusion; i.e., 0.4 rather than .4. A leading zero is the preferred term.

11. A recently delivered mother and her baby are at the clinic for a 6-week postpartum checkup. Which response by the client alerts the nurse that psychosocial outcomes have not been met? a. The woman excessively discusses her labor and birth experience. b. The woman feels that her baby is more attractive and clever than any others. c. The woman has not given the baby a name. d. The woman has a partner or family members who react very positively about the baby.

ANS: C If the mother is having difficulty naming her new infant, it may be a signal that she is not adapting well to parenthood. Other red flags include a refusal to hold or feed the baby, a lack of interaction with the infant, and becoming upset when the baby vomits or needs a diaper change. A new mother who is having difficulty is unwilling to discuss her labor and birth experience. An appropriate nursing diagnosis might be Impaired parenting, related to a long, difficult labor or unmet expectations of birth. A mother who is willing to discuss her birth experience is making a healthy personal adjustment. The mother who is not coping well finds her baby unattractive and messy. She may also be overly disappointed in the babys sex. The client might voice concern that the baby reminds her of a family member whom she does not like. Having a partner and/or other family members react positively is an indication that this new mother has a good support system in place. This support system helps reduce anxiety related to her new role as a mother.

16. Postpartum overdistention of the bladder and urinary retention can lead to which complications? a. Postpartum hemorrhage and eclampsia b. Fever and increased blood pressure c. Postpartum hemorrhage and urinary tract infection d. Urinary tract infection and uterine rupture

ANS: C Incomplete emptying and overdistention of the bladder can lead to urinary tract infection. Overdistention of the bladder displaces the uterus and prevents contraction of the uterine muscle, thus leading to postpartum hemorrhage. No correlation exists between bladder distention and high blood pressure or eclampsia. The risk of uterine rupture decreases after the birth of the infant.

A nurse practicing in the perinatal setting should promote kangaroo care regardless of an infant's gestational age. Which statement regarding this intervention is most appropriate? a.Kangaroo care was adopted from classical British nursing traditions. b.This intervention helps infants with motor and CNS impairments. c.Kangaroo care helps infants interact directly with their parents and enhances their temperature regulation. d.This intervention gets infants ready for breastfeeding.

ANS: C Kangaroo care is skin-to-skin holding in which the infant, dressed only in a diaper, is placed directly on the parent's bare chest and then covered. The procedure helps infants interact with their parents and regulates their temperature, among other developmental benefits. Kangaroo care was established in Bogota, Colombia, assists the infant in maintaining an organized state, and decreases pain perception during heelsticks. Even premature infants who are unable to suckle benefit from kangaroo care. This practice fosters increased vigor and an enhanced breastfeeding experience as the infant matures.

15. Which information should the nurse provide to a breastfeeding mother regarding optimal self-care? a. She will need an extra 1000 calories a day to maintain energy and produce milk. b. She can return to prepregnancy consumption patterns of any drinks as long as she gets enough calcium. c. She should avoid trying to lose large amounts of weight. d. She must avoid exercising because it is too fatiguing.

ANS: C Large weight loss releases fat-stored contaminants into her breast milk, and it also involves eating too little and/or exercising too much. A breastfeeding mother needs to add only 200 to 500 extra calories to her diet to provide the extra nutrients for her infant. However, this is true only if she does not drink alcohol, limits coffee to no more than two cups (including caffeine in chocolate, tea, and some sodas, too), and carefully reads the herbal tea ingredients. Although she needs her rest, moderate exercise is healthy.

8. A new mother wants to be sure that she is meeting her daughters needs while feeding the baby commercially prepared infant formula. The nurse should evaluate the mothers knowledge about appropriate infant feeding techniques. Which statement by the client reassures the nurse that correct learning has taken place? a. Since reaching 2 weeks of age, I add rice cereal to my daughters formula to ensure adequate nutrition. b. I warm the bottle in my microwave oven. c. I burp my daughter during and after the feeding as needed. d. I refrigerate any leftover formula for the next feeding.

ANS: C Most infants swallow air when fed from a bottle and should be given a chance to burp several times during and after the feeding. Solid food should not be introduced to the infant for at least 4 to 6 months after birth. A microwave should never be used to warm any food to be given to an infant. The heat is not distributed evenly, which may pose a risk of burning the infant. Any formula left in the bottle after the feeding should be discarded because the infants saliva has mixed with it.

A 38-year-old Hispanic woman vaginally delivered a 9-pound, 6-ounce baby girl after being in labor for 43 hours. The baby died 3 days later from sepsis. On what grounds could the woman have a legitimate legal case for negligence? a.Inexperienced maternity nurse was assigned to care for the client. b.Client was past her due date by 3 days. c.Standard of care was not met. d.Client refused electronic fetal monitoring.

ANS: C Not meeting the standard of care is a legitimate factor for a case of negligence. An inexperienced maternity nurse would need to display competency before being assigned to care for clients on his or her own. This client may have been past her due date; however, a term pregnancy often goes beyond 40 weeks of gestation. Although fetal monitoring is the standard of care, the client has the right to refuse treatment. This refusal is not a case for negligence, but informed consent should be properly obtained, and the client should have signed an against medical advice form when refusing any treatment that is within the standard of care.

7. A 38-year-old Hispanic woman delivered a 9-lb, 6-oz baby girl vaginally after being in labor for 43 hours. The baby died 3 days later from sepsis. On what grounds could the woman have a legitimate legal case for negligence? a. She is Hispanic. b. She delivered a girl. c. If the standards of care were not met. d. She refused fetal monitoring.

ANS: C Not meeting the standards of care is a legitimate factor for a case of negligence. The client's race is not a factor for a case of negligence. The infant's gender is not a factor for a case of negligence. Although fetal monitoring is the standard of care, the client has the right to refuse treatment. This refusal is not a case for negligence, but informed consent should be properly obtained, and the client should sign an against medical advice form for refusal of any treatment that is within the standard of care

5. The nurses working at a newly established birthing center have begun to compare their performance in providing maternal-newborn care against clinical standards. This comparison process, designed to improve the quality of client care, is called: a.. Best practices network b. Clinical benchmarking c. Outcomes-oriented care d. Evidence-based practice

ANS: C Outcomes-oriented care measures effectiveness of interventions and quality of care against benchmarks or standards. The term best practice refers to a program or service that has been recognized for excellence. Clinical benchmarking is a process used to compare one's own performance against the performance of the best in an area of service. The term evidence-based practice refers to the provision of care based on evidence gained through research and clinical trials.

The nurses working at a newly established birthing center have begun to compare their performance in providing maternal-newborn care against clinical standards. This comparison process is most commonly known as what? a.Best practices network b.Clinical benchmarking c.Outcomes-oriented practice d.Evidence-based practice

ANS: C Outcomes-oriented practice measures the effectiveness of the interventions and quality of care against benchmarks or standards. The term best practice refers to a program or service that has been recognized for its excellence. Clinical benchmarking is a process used to compare one's own performance against the performance of the best in an area of service. The term evidence-based practice refers to the provision of care based on evidence gained through research and clinical trials.

26. Which action by the mother will initiate the milk ejection reflex (MER)? a. Wearing a firm-fitting bra b. Drinking plenty of fluids c. Placing the infant to the breast d. Applying cool packs to her breast

ANS: C Oxytocin, which causes the MER reflex, increases in response to nipple stimulation. A firm bra is important to support the breast; however, it will not initiate the MER reflex. Drinking plenty of fluids is necessary for adequate milk production, but adequate intake of water alone will not initiate the MER reflex. Cool packs to the breast will decrease the MER reflex.

Which statement concerning cyclic perimenstrual pain and discomfort (CPPD) is accurate? a. Premenstrual dysphoric disorder (PMDD) is a milder form of PMS and more common in young women. b. Secondary dysmenorrhea is more intense and more medically significant than primary dysmenorrhea. c. PMS is a complex, poorly understood condition that may include any of a hundred symptoms. d. The causes of PMS have been well established.

ANS: C PMS may manifest itself with one or more of a hundred physical and psychologic symptoms. PDD is a more severe variant of PMS. Secondary dysmenorrhea is characterized by more muted pain than the pain reported in primary dysmenorrhea; however, the medical treatment is close to the same. The cause of PMS is unknown and may be, in fact, a collection of different problems.

7. Prostaglandin gel has been ordered for a pregnant woman at 43 weeks of gestation. What is the primary purpose of prostaglandin administration? a. To enhance uteroplacental perfusion in an aging placenta b. To increase amniotic fluid volume c. To ripen the cervix in preparation for labor induction d. To stimulate the amniotic membranes to rupture

ANS: C Preparations of prostaglandin E1 and E2 are effective when used before labor induction to ripen (i.e., soften and thin) the cervix. Uteroplacental perfusion is not altered by the use of prostaglandins. The insertion of prostaglandin gel has no effect on the level of amniotic fluid. In some cases, women will spontaneously begin laboring after the administration of prostaglandins, thereby eliminating the need for oxytocin. It is not common for a woman's membranes to rupture as a result of prostaglandin use. DIF: Cognitive Level: Apply REF: p. 779 TOP: Nursing Process: Planning MSC: Client Needs: Physiologic Integrity

For clinical purposes, the most accurate definition of preterm and post-term infants is defined as what? a.Preterm: Before 34 weeks of gestation if the infant is appropriate for gestational age (AGA); before 37 weeks if the infant is small for gestational age (SGA) b.Postterm: After 40 weeks of gestation if the infant is large for gestational age (LGA); beyond 42 weeks if the infant is AGA c.Preterm: Before 37 weeks of gestation and postterm beyond 42 weeks of gestation; no matter the size for gestational age at birth d.Preterm: Before 38 to 40 weeks of gestation if the infant is SGA; postterm, beyond 40 to 42 weeks gestation if the infant is LGA

ANS: C Preterm and postterm are strictly measures of time—before 37 weeks and beyond 42 weeks, respectively—regardless of the size for gestational age.

1. In planning for home care of a woman with preterm labor, which concern should the nurse need to address? a. Nursing assessments are different from those performed in the hospital setting. b. Restricted activity and medications are necessary to prevent a recurrence of preterm labor. c. Prolonged bed rest may cause negative physiologic effects. d. Home health care providers are necessary.

ANS: C Prolonged bed rest may cause adverse effects such as weight loss, loss of appetite, muscle wasting, weakness, bone demineralization, decreased cardiac output, risk for thrombophlebitis, alteration in bowel functions, sleep disturbance, and prolonged postpartum recovery. Nursing assessments differ somewhat from those performed in the acute care setting, but this concern does not need to be addressed. Restricted activity and medications may prevent preterm labor but not in all women. In addition, the plan of care is individualized to meet the needs of each client. Many women receive home health nurse visits, but care is individualized for each woman. DIF: Cognitive Level: Analyze REF: p. 777 TOP: Nursing Process: Planning MSC: Client Needs: Health Promotion and Maintenance

24. Which type of formula is not diluted with water, before being administered to an infant? a. Powdered b. Concentrated c. Ready-to-use d. Modified cows milk

ANS: C Ready-to-use formula can be poured directly from the can into the babys bottle and is good (but expensive) when a proper water supply is not available. Formula should be well mixed to dissolve the powder and make it uniform in consistency. Improper dilution of concentrated formula may cause malnutrition or sodium imbalances. Cows milk is more difficult for the infant to digest and is not recommended, even if it is diluted.

The nurse should understand the process by which the HIV infection occurs. Once the virus has entered the body, what is the time frame for seroconversion to HIV positivity? a.6 to 10 days b.2 to 4 weeks c.6 to 12 weeks d.6 months

ANS: C Seroconversion to HIV positivity usually occurs within 6 to 12 weeks after the virus has entered the body. Both 6 to 10 days and 2 to 4 weeks are too short for seroconversion to HIV positivity to occur, and 6 months is too long.

17. The nurse is teaching a client with preterm premature rupture of membranes (PPROM) regarding self-care activities. Which activities should the nurse include in her teaching? a. Report a temperature higher than 40° C. b. Tampons are safe to use to absorb the leaking amniotic fluid. c. Do not engage in sexual activity. d. Taking frequent tub baths is safe.

ANS: C Sexual activity should be avoided because it may induce preterm labor. A temperature higher than 38° C should be reported. To prevent the risk of infection, tub baths should be avoided and nothing should be inserted into the vagina. Further, foul-smelling vaginal fluid, which may be a sign of infection, should be reported. DIF: Cognitive Level: Apply REF: p. 762 TOP: Nursing Process: Implementation MSC: Client Needs: Health Promotion and Maintenance

An infant is being discharged from the NICU after 70 days of hospitalization. The infant was born at 30 weeks of gestation with several conditions associated with prematurity, including RDS, mild bronchopulmonary dysplasia (BPD), and retinopathy of prematurity (ROP), requiring surgical treatment. During discharge teaching, the infant's mother asks the nurse if her baby will meet developmental milestones on time, as did her son who was born at term. What is the nurse's most appropriate response? a."Your baby will develop exactly like your first child." b."Your baby does not appear to have any problems at this time." c."Your baby will need to be corrected for prematurity." d."Your baby will need to be followed very closely."

ANS: C The age of a preterm newborn is corrected by adding the gestational age and the postnatal age. The infant's responses are accordingly evaluated against the norm expected for the corrected age of the infant. The baby is currently 40 weeks of postconceptional age and can be expected to be doing what a 40-week-old infant would be doing. Although predicting with complete accuracy the growth and development potential of each preterm infant is impossible, certain measurable factors predict normal growth and development. The preterm infant experiences catch-up body growth during the first 2 to 3 years of life. Development needs to be evaluated over time. The growth and developmental milestones are corrected for gestational age until the child is approximately years old.

5. A primigravida at 40 weeks of gestation is having uterine contractions every to 2 minutes and states that they are very painful. Her cervix is dilated 2 cm and has not changed in 3 hours. The woman is crying and wants an epidural. What is the likely status of this woman's labor? a. She is exhibiting hypotonic uterine dysfunction. b. She is experiencing a normal latent stage. c. She is exhibiting hypertonic uterine dysfunction. d. She is experiencing precipitous labor.

ANS: C The contraction pattern observed in this woman signifies hypertonic uterine activity. Typically, uterine activity in this phase occurs at 4- to 5-minute intervals lasting 30 to 45 seconds. Women who experience hypertonic uterine dysfunction, or primary dysfunctional labor, are often anxious first-time mothers who are having painful and frequent contractions that are ineffective at causing cervical dilation or effacement to progress. With hypotonic uterine dysfunction, the woman initially makes normal progress into the active stage of labor; then the contractions become weak and inefficient or stop altogether. Precipitous labor is one that lasts less than 3 hours from the onset of contractions until time of birth. DIF: Cognitive Level: Apply REF: p. 773 TOP: Nursing Process: Diagnosis MSC: Client Needs: Health Promotion and Maintenance

By understanding the four mechanisms of heat transfer (convection, conduction, radiation, and evaporation), the nurse can create an environment for the infant that prevents temperature instability. Which significant symptoms will the infant display when experiencing cold stress? a.Decreased respiratory rate b.Bradycardia, followed by an increased heart rate c.Mottled skin with acrocyanosis d.Increased physical activity

ANS: C The infant has minimal-to-no fat stores. During times of cold stress, the skin becomes mottled and acrocyanosis develops, progressing to cyanosis. Even if the infant is being cared for on a radiant warmer or in an isolette, the nurse's role is to observe the infant frequently to prevent heat loss and to respond quickly if signs and symptoms of cold stress occur. The respiratory rate increases, followed by periods of apnea. The infant initially tries to conserve heat and burns more calories, after which the metabolic system goes into overdrive. In the preterm infant who is experiencing heat loss, the heart rate initially increases, followed by periods of bradycardia. In the term infant, increased physical activity is the natural response to heat loss. However, in a term infant who is experiencing respiratory distress or in a preterm infant, physical activity is decreased.

21. In assisting the breastfeeding mother to position the baby, which information regarding positioning is important for the nurse to keep in mind? a. The cradle position is usually preferred by mothers who had a cesarean birth. b. Women with perineal pain and swelling prefer the modified cradle position. c. Whatever the position used, the infant is belly to belly with the mother. d. While supporting the head, the mother should push gently on the occiput.

ANS: C The infant naturally faces the mother, belly to belly. The football position is usually preferred after a cesarean birth. Women with perineal pain and swelling prefer the side-lying position because they can rest while breastfeeding. The mother should never push on the back of the head. It may cause the baby to bite, hyperextend the neck, or develop an aversion to being brought near the breast.

5. A breastfeeding woman develops engorged breasts at 3 days postpartum. What action will help this client achieve her goal of reducing the engorgement? a. Skip feedings to enable her sore breasts to rest. b. Avoid using a breast pump. c. Breastfeed her infant every 2 hours. d. Reduce her fluid intake for 24 hours.

ANS: C The mother should be instructed to attempt feeding her infant every 2 hours while massaging the breasts as the infant is feeding. Skipping feedings may cause further swelling and discomfort. If the infant does not adequately feed and empty the breast, then the mother may pump to extract the milk and relieve some of the discomfort. Dehydration further irritates swollen breast tissue.

11. Sally comes in for her first prenatal examination. This is her first child. She asks you (the nurse), "How does my baby get air inside my uterus?" The correct response is: a. "The baby's lungs work in utero to exchange oxygen and carbon dioxide." b. "The baby absorbs oxygen from your blood system." c. "The placenta provides oxygen to the baby and excretes carbon dioxide into your bloodstream." d. "The placenta delivers oxygen-rich blood through the umbilical artery to the baby's abdomen."

ANS: C The placenta functions by supplying oxygen and excreting carbon dioxide into the maternal bloodstream. The fetal lungs do not function for respiratory gas exchange in utero. The baby does not simply absorb oxygen from a woman's blood system. Blood and gas transport occur through the placenta. The placenta delivers oxygen-rich blood through the umbilical vein, not artery.

7. Under the Newborns and Mothers Health Protection Act, all health plans are required to allow new mothers and newborns to remain in the hospital for a minimum of _____ hours after a normal vaginal birth and for _____ hours after a cesarean birth. What is the correct interpretation of this legislation? a. 24; 72 b. 24; 96 c. 48; 96 d. 48; 120

ANS: C The specified stays are 48 hours (2 days) for a vaginal birth and 96 hours (4 days) for a cesarean birth. The attending provider and the mother together can decide on an earlier discharge. A client may be discharged either 24 hours after a vaginal birth or 72 hours after a cesarean birth if she is stable and her provider is in agreement. A client is unlikely to remain in the hospital for 120 hours after a cesarean birth unless complications have developed.

4. At approximately _____ weeks of gestation, lecithin is forming on the alveolar surfaces, the eyelids open, and the fetus measures approximately 27 cm crown to rump and weighs approximately 1110 g. a. 20 b. 24 c. 28 d. 30

ANS: C These milestones occur at 28 weeks. These milestones are not completed by 20 weeks of gestation. These milestones in human development are not completed at 24 weeks of gestation. These specific milestones are reached as early as 28, not 30, weeks of gestation.

10. The measurement of lecithin in relation to sphingomyelin (L/S ratio) is used to determine fetal lung maturity. Which ratio reflects maturity of the lungs? a. 1.4:1 b. 1.8:1 c. 2:1 d. 1:1

ANS: C This ratio indicates a 2:1 ratio of lecithin to sphingomyelin, an indicator of lung maturity. 1.4:1 ratio indicates immaturity of the fetal lungs. 1.8:1 ratio indicates immaturity of the fetal lungs. 1:1 ratio indicates immaturity of the fetal lungs.

12. Which statement related to cephalopelvic disproportion (CPD) is the least accurate? a. CPD can be related to either fetal size or fetal position. b. The fetus cannot be born vaginally. c. CPD can be accurately predicted. d. Causes of CPD may have maternal or fetal origins.

ANS: C Unfortunately, accurately predicting CPD is not possible. Although CPD is often related to excessive fetal size (macrosomia), malposition of the fetal presenting part is the problem in many cases, not true CPD. When CPD is present, the fetus cannot fit through the maternal pelvis to be born vaginally. CPD may be related to either fetal origins such as macrosomia or malposition or maternal origins such as a too small or malformed pelvis. DIF: Cognitive Level: Understand REF: p. 775 TOP: Nursing Process: Planning MSC: Client Needs: Health Promotion and Maintenance

6. At a 2-month well-baby examination, it was discovered that an exclusively breastfed infant had only gained 10 ounces in the past 4 weeks. The mother and the nurse develop a feeding plan for the infant to increase his weight gain. Which change in dietary management will assist the client in meeting this goal? a. Begin solid foods. b. Have a bottle of formula after every feeding. c. Have one extra breastfeeding session every 24 hours. d. Start iron supplements.

ANS: C Usually the solution to slow weight gain is to improve the feeding technique. Position and the latch-on technique are evaluated, and adjustments are made. Adding a feeding or two within a 24-hour period might help. Solid foods should not be introduced to an infant for at least 4 to 6 months. Bottle feeding may cause nipple confusion and may limit the supply of milk. Iron supplements have no bearing on weight gain.

2. What is the primary role of practicing nurses in the research process? a.Designing research studies b.Collecting data for other researchers c.Identifying researchable problems d.Seeking funding to support research studies

ANS: C When problems are identified, research can be properly conducted. Research of health care issues leads to evidence-based practice guidelines. Designing research studies is only one factor of the research process. Data collection is another factor of research. Financial support is necessary to conduct research, but it is not the primary role of the nurse in the research process.

2. What are the most common causes for subinvolution of the uterus? a. Postpartum hemorrhage and infection b. Multiple gestation and postpartum hemorrhage c. Uterine tetany and overproduction of oxytocin d. Retained placental fragments and infection

D (Subinvolution is the failure of the uterus to return to a nonpregnant state. The most common causes of subinvolution are retained placental fragments and infection. Subinvolution may be caused by an infection and result in hemorrhage. Multiple gestations may cause uterine atony, resulting in postpartum hemorrhaging. Uterine tetany and overproduction of oxytocin do not cause subinvolution.)

6. A woman is having her first child. She has been in labor for 15 hours. A vaginal examination performed 2 hours earlier revealed the cervix to be dilated to 5 cm and 100% effaced, and the presenting part of the fetus was at station 0; however, another vaginal examination performed 5 minutes ago indicated no changes. What abnormal labor pattern is associated with this description? a. Prolonged latent phase b. Protracted active phase c. Secondary arrest d. Protracted descent

ANS: C With a secondary arrest of the active phase, the progress of labor has stopped. This client has not had any anticipated cervical change, indicating an arrest of labor. In the nulliparous woman, a prolonged latent phase typically lasts longer than 20 hours. A protracted active phase, the first or second stage of labor, is prolonged (slow dilation). With a protracted descent, the fetus fails to descend at an anticipated rate during the deceleration phase and second stage of labor. DIF: Cognitive Level: Analyze REF: p. 774 TOP: Nursing Process: Assessment MSC: Client Needs: Health Promotion and Maintenance

18. According to demographic research, which woman is least likely to breastfeed and therefore most likely to need education regarding the benefits and proper techniques of breastfeeding? a. Between 30 and 35 years of age, Caucasian, and employed part time outside the home b. Younger than 25 years of age, Hispanic, and unemployed c. Younger than 25 years of age, African-American, and employed full time outside the home d. 35 years of age or older, Caucasian, and employed full time at home

ANS: C Women least likely to breastfeed are typically younger than 25 years of age, have a lower income, are less educated, are employed full time outside the home, and are African-American.

Which client would be an ideal candidate for injectable progestins such as medroxyprogesterone acetate as a contraceptive choice? a. The ideal candidate wants menstrual regularity and predictability. b. The client has a history of thrombotic problems or breast cancer. c. The ideal candidate has difficulty remembering to take oral contraceptives daily. d. The client is homeless or mobile and rarely receives health care.

ANS: C Advantages of medroxyprogesterone acetate includes its contraceptive effectiveness, compared with the effectiveness of combined oral contraceptives, and the requirement of only four injections a year. The disadvantages of injectable progestins are prolonged amenorrhea and uterine bleeding. The use of injectable progestin carries an increased risk of venous thrombosis and thromboembolism. To be effective, injections must be administered every 11 to 13 weeks. Access to health care is necessary to prevent pregnancy or potential complications.

22. With regard to the initial visit with a client who is beginning prenatal care, nurses should be aware that: a. The first interview is a relaxed, get-acquainted affair in which nurses gather some general impressions. b. If nurses observe handicapping conditions, they should be sensitive and not inquire about them because the client will do that in her own time. c. Nurses should be alert to the appearance of potential parenting problems, such as depression or lack of family support. d. Because of legal complications, nurses should not ask about illegal drug use; that is left to physicians

ANS: C Besides these potential problems, nurses need to be alert to the woman's attitude toward health care. The initial interview needs to be planned, purposeful, and focused on specific content. A lot of ground must be covered. Nurses must be sensitive to special problems, but they do need to inquire because discovering individual needs is important. People with chronic or handicapping conditions forget to mention them because they have adapted to them. Getting information on drug use is important and can be done confidentially. Actual testing for drug use requires the client's consent.

32. In response to requests by the U.S. Public Health Service for new models of prenatal care, an innovative new approach to prenatal care known as centering pregnancy was developed. Which statement would accurately apply to the centering model of care? a. Group sessions begin with the first prenatal visit. b. At each visit, blood pressure, weight, and urine dipsticks are obtained by the nurse. c. Eight to 12 women are placed in gestational-age cohort groups. d. Outcomes are similar to those of traditional prenatal care.

ANS: C Gestational age cohorts comprise the groups, with approximately 8 to 12 women in each group. This group remains intact throughout the pregnancy. Individual follow-up visits are scheduled as needed. Group sessions begin at 12 to 16 weeks of gestation and end with an early postpartum visit. Before group sessions the client has an individual assessment, physical examination, and history. At the beginning of each group meeting, clients measure their own blood pressure, weight, and urine dips and enter these in their record. Fetal heart rate assessment and fundal height are obtained by the nurse. Results evaluating this approach have been very promising. In a study of adolescent clients, there was a decrease in low-birth-weight infants and an increase in breastfeeding rates.

In the acronym BRAIDED, which letter is used to identify the key components of informed consent that the nurse must document? a. B stands for birth control. b. R stands for reproduction. c. A stands for alternatives. d. I stands for ineffective.

ANS: C In the acronym BRAIDED, A stands for alternatives and information about other viable methods. B stands for benefits and information about the advantages of a particular birth control method and its success rates. R stands for risks and information about the disadvantages of a particular method and its failure rates. I stands for inquiries and the opportunity to ask questions.

19. With regard to a woman's reordering of personal relationships during pregnancy, the maternity nurse should understand that: a. Because of the special motherhood bond, a woman's relationship with her mother is even more important than with the father of the child. b. Nurses need not get involved in any sexual issues the couple has during pregnancy, particularly if they have trouble communicating them to each other. c. Women usually express two major relationship needs during pregnancy: feeling loved and valued and having the child accepted by the father. d. The woman's sexual desire is likely to be highest in the first trimester because of the excitement and because intercourse is physically easier.

ANS: C Love and support help a woman feel better about her pregnancy. The most important person to the pregnant woman is usually the father. Nurses can facilitate communication between partners about sexual matters if, as is common, they are nervous about expressing their worries and feelings. The second trimester is the time when a woman's sense of well-being, along with certain physical changes, increases her desire for sex. Desire is decreased in the first and third trimesters.

The nurse is providing contraceptive instruction to a young couple who are eager to learn. The nurse should be cognizant of which information regarding the natural family planning method? a. The natural family planning method is the same as coitus interruptus or "pulling out." b. This contraception method uses the calendar method to align the woman's cycle with the natural phases of the moon. c. This practice is the only contraceptive method acceptable to the Roman Catholic Church. d. The natural family planning method relies on barrier methods during the fertility phases.

ANS: C Natural family planning is the only contraceptive practice acceptable to the Roman Catholic Church. "Pulling out" is not the same as periodic abstinence, another name for natural family planning. The phases of the moon are not part of the calendar method or any method. Natural family planning is another name for periodic abstinence, which is the accepted way to pass safely through the fertility phases without relying on chemical or physical barriers.

11. The nurse should be aware that the partner's main role in pregnancy is to: a. Provide financial support. b. Protect the pregnant woman from "old wives' tales." c. Support and nurture the pregnant woman. d. Make sure the pregnant woman keeps prenatal appointments.

ANS: C The partner's main role in pregnancy is to nurture the pregnant woman and respond to her feelings of vulnerability. In older societies, the man enacted the ritual couvade. Changing cultural and professional attitudes have encouraged fathers' participation in the birth experience over the past 30 years.

27. To provide the patient with accurate information about dental care during pregnancy, maternity nurses should be aware that: a. Dental care can be dropped from the priority list because the woman has enough to worry about and is getting a lot of calcium anyway. b. Dental surgery, in particular, is contraindicated because of the psychologic stress it engenders. c. If dental treatment is necessary, the woman will be most comfortable with it in the second trimester. d. Dental care interferes with the expectant mother's need to practice conscious relaxation.

ANS: C The second trimester is best for dental treatment because that is when the woman will be able to sit most comfortably in the dental chair. Dental care such as brushing with fluoride toothpaste is especially important during pregnancy because nausea during pregnancy may lead to poor oral hygiene. Emergency dental surgery is permissible, but the mother must clearly understand the risks and benefits. Conscious relaxation is useful, and it may even help the woman get through any dental appointments; it is not a reason to avoid them

24. With regard to follow-up visits for women receiving prenatal care, nurses should be aware that: a. The interview portions become more intensive as the visits become more frequent over the course of the pregnancy. b. Monthly visits are scheduled for the first trimester, every 2 weeks for the second trimester, and weekly for the third trimester. c. During the abdominal examination, the nurse should be alert for supine hypotension. d. For pregnant women, a systolic blood pressure (BP) of 130 and a diastolic BP of 80 is sufficient to be considered hypertensive.

ANS: C The woman lies on her back during the abdominal examination, possibly compressing the vena cava and aorta, which can cause a decrease in blood pressure and a feeling of faintness. The interview portion of follow-up examinations is less extensive than in the initial prenatal visits, during which so much new information must be gathered. Monthly visits are routinely scheduled for the first and second trimesters; visits increase to every 2 weeks at week 28 and to once a week at week 36. For pregnant women hypertension is defined as a systolic BP of 140 or greater and a diastolic BP of 90 or greater.

A woman will be taking oral contraceptives using a 28-day pack. What advice should the nurse provide to protect this client from an unintended pregnancy? a. Limit sexual contact for one cycle after starting the pill. b. Use condoms and foam instead of the pill for as long as the client takes an antibiotic. c. Take one pill at the same time every day. d. Throw away the pack and use a backup method if two pills are missed during week 1 of her cycle.

ANS: C To maintain adequate hormone levels for contraception and to enhance compliance, clients should take oral contraceptives at the same time each day. If contraceptives are to be started at any time other than during normal menses or within 3 weeks after birth or an abortion, then another method of contraception should be used through the first week to prevent the risk of pregnancy. Taken exactly as directed, oral contraceptives prevent ovulation, and pregnancy cannot occur. No strong pharmacokinetic evidence indicates a link between the use of broad-spectrum antibiotics and altered hormonal levels in oral contraceptive users. If the client misses two pills during week 1, then she should take two pills a day for 2 days and finish the package and use a backup contraceptive method for the next 7 consecutive days.

4. A pregnant woman at 10 weeks of gestation jogs three or four times per week. She is concerned about the effect of exercise on the fetus. The nurse should inform her: a. "You don't need to modify your exercising any time during your pregnancy." b. "Stop exercising because it will harm the fetus." c. "You may find that you need to modify your exercise to walking later in your pregnancy, around the seventh month." d. "Jogging is too hard on your joints; switch to walking now."

ANS: C Typically running should be replaced with walking around the seventh month of pregnancy. The nurse should inform the woman that she may need to reduce her exercise level as the pregnancy progresses. Physical activity promotes a feeling of well-being in pregnant women. It improves circulation, promotes relaxation and rest, and counteracts boredom. Simple measures should be initiated to prevent injuries, such as warm-up and stretching exercises to prepare the joints for more strenuous exercise.

9. A woman who is 8 months pregnant asks the nurse, "Does my baby have any antibodies to fight infection?" The most appropriate response by the nurse is: a. "Your baby has all the immunoglobulins necessary: IgG, IgM, and IgA." b. "Your baby won't receive any antibodies until he is born and you breastfeed him." c. "Your baby does not have any antibodies to fight infection." d. "Your baby has IgG and IgM immunoglobulins."

ANS: D During the third trimester the only immunoglobulin that crosses the placenta, IgG, provides passive acquired immunity to specific bacterial toxins. The fetus produces IgM by the end of the first trimester. During the third trimester the only immunoglobulin that crosses the placenta, IgG, provides passive acquired immunity to specific bacterial toxins. The fetus produces IgM by the end of the first trimester. IgA immunoglobulins are not produced by the baby. By the third trimester the fetus has IgG and IgM. Breastfeeding supplies the baby with IgA immunoglobulins. By the third trimester, the fetus has IgG and IgM immunoglobulins to fight infection.

3. A couple comes in for an infertility workup, having attempted to achieve pregnancy for 2 years. The woman, 37 years of age, has always had irregular menstrual cycles but is otherwise healthy. The man has fathered two children from a previous marriage and had a vasectomy reversal 2 years ago. The man has had two normal semen analyses, but the sperm seem to be clumped together. What additional testing is needed? a. Testicular biopsy b. Antisperm antibodies c. Serum prolactin level d. Examination for testicular infection

ANS: C This scenario does not indicate that the woman has had any testing related to her irregular menstrual cycles. Hormone analysis is performed to assess endocrine function of the hypothalamic-pituitary-ovarian axis when menstrual cycles are absent or irregular. Determining the blood levels of prolactin, FSH, luteinizing hormone (LH), estradiol, progesterone, and thyroid hormones may be necessary to diagnose the cause of the woman's irregular menstrual cycles. A testicular biopsy is indicated only in cases of azoospermia (no sperm cells) or severe oligospermia (low number of sperm cells). Although unlikely to be the case because the husband has already produced children, antisperm antibodies may be produced by the man against his own sperm. Examination for testicular infection would be performed before semen analysis. Furthermore, infection would affect spermatogenesis.

4. A couple is attempting to cope with an infertility problem. They want to know what they can do to preserve their emotional equilibrium. What is the nurse's most appropriate response? a. "Tell your friends and family so that they can help you." b. "Talk only to other friends who are infertile, because only they can help." c. "Get involved with a support group. I'll give you some names." d. "Start adoption proceedings immediately, because adopting an infant can be very difficult."

ANS: C Venting negative feelings may unburden the couple. A support group may provide a safe haven for the couple to share their experiences and gain insight from others' experiences. Although talking about their feelings may unburden them of negative feelings, infertility can be a major stressor that affects the couple's relationships with family and friends. Limiting their interactions to other infertile couples may be a beginning point for addressing psychosocial needs. However, depending on where the other couple is in their own recovery process, limiting their interactions may not be of assistance to them. Telling the couple to start adoption proceedings immediately is not supportive of the psychosocial needs of this couple and may be detrimental to their well-being

Which statement(s) might the nurse appropriately include when teaching a client about calcium intake for osteoporosis? (Select all that apply.) a. You should try to increase your protein intake when you are taking calcium. b. It is best to take calcium in one large dose. c. Tums are the most soluble form of calcium. d. You should take calcium with vitamin D because the vitamin D helps your body better absorb calcium. e. Its okay to take calcium if you have had a history of kidney stones.

ANS: C, D Teaching the client to take calcium with vitamin D is accurate. Excessive protein should be avoided. Calcium is best taken in divided doses to increase absorption. Calcium should be taken with vitamin D to increase absorption. Calcium is contraindicated in women with a history of kidney stones.

The National Quality Forum has issued a list of "never events" specifically pertaining to maternal and child health. These include all of the following except: a.infant discharged to the wrong person. b.kernicterus associated with the failure to identify and treat hyperbilirubinemia. c.artificial insemination with the wrong donor sperm or egg. d.foreign object retained after surgery.

ANS: D Although a foreign object retained after surgery is a never event, it does not specifically pertain to obstetric clients. A client undergoing any type of surgery may be at risk for this event. An infant discharged to the wrong person specifically pertains to postpartum care. Death or serious disability as a result of kernicterus pertains to newborn assessment and care. Artificial insemination affects families seeking care for infertility.

A 21-year-old client exhibits a greenish, copious, and malodorous discharge with vulvar irritation. A speculum examination and wet smear are performed. Which condition is this client most likely experiencing? a.Bacterial vaginosis b.Candidiasis c.Yeast infection d.Trichomoniasis

ANS: D Although uncomfortable, a speculum examination is always performed and a wet smear obtained if the client exhibits symptoms of trichomoniasis. The presence of many white blood cell protozoa is a positive finding for trichomoniasis. A normal saline test is used to test for bacterial vaginosis. A potassium hydroxide preparation is used to test for candidiasis. Yeast infection is the common name for candidiasis, for which the test is a potassium hydroxide preparation.

The _____ is/are responsible for oxygen and carbon dioxide transport to and from the maternal bloodstream. a. Decidua basalis b. Blastocyst c. Germ layer d. Chorionic villi

ANS: D Chorionic villi are finger-like projections that develop out of the trophoblast and extend into the blood-filled spaces of the endometrium. The villi obtain oxygen and nutrients from the maternal bloodstream and dispose of carbon dioxide and waste products into the maternal blood. The decidua basalis is the portion of the decidua (endometrium) under the blastocyst where the villi attach. The blastocyst is the embryonic development stage after the morula. Implantation occurs at this stage. The germ layer is a layer of the blastocyst.

A 23-year-old African-American woman is pregnant with her first child. Based on the statistics for infant mortality, which plan is most important for the nurse to implement? a.Perform a nutrition assessment. b.Refer the woman to a social worker. c.Advise the woman to see an obstetrician, not a midwife. d.Explain to the woman the importance of keeping her prenatal care appointments.

ANS: D Consistent prenatal care is the best method of preventing or controlling risk factors associated with infant mortality. Nutritional status is an important modifiable risk factor, but it is not the most important action a nurse should take in this situation. The client may need assistance from a social worker at some time during her pregnancy, but a referral to a social worker is not the most important aspect the nurse should address at this time. If the woman has identifiable high-risk problems, then her health care may need to be provided by a physician. However, it cannot be assumed that all African-American women have high-risk issues. In addition, advising the woman to see an obstetrician is not the most important aspect on which the nurse should focus at this time, and it is not appropriate for a nurse to advise or manage the type of care a client is to receive.

When the nurse is unsure how to perform a client care procedure that is high risk and low volume, his or her best action in this situation would be what? a.Ask another nurse. b.Discuss the procedure with the client's physician. c.Look up the procedure in a nursing textbook. d.Consult the agency procedure manual, and follow the guidelines for the procedure.

ANS: D Following the agency's policies and procedures manual is always best when seeking information on correct client procedures. These policies should reflect the current standards of care and the individual state's guidelines. Each nurse is responsible for his or her own practice. Relying on another nurse may not always be a safe practice. Each nurse is obligated to follow the standards of care for safe client care delivery. Physicians are responsible for their own client care activity. Nurses may follow safe orders from physicians, but they are also responsible for the activities that they, as nurses, are to carry out. Information provided in a nursing textbook is basic information for general knowledge. Furthermore, the information in a textbook may not reflect the current standard of care or the individual state or hospital policies.

11. The nurse is performing an assessment on a client who thinks she may be experiencing preterm labor. Which information is the most important for the nurse to understand and share with the client? a. Because all women must be considered at risk for preterm labor and prediction is so variable, teaching pregnant women the symptoms of preterm labor probably causes more harm through false alarms. b. Braxton Hicks contractions often signal the onset of preterm labor. c. Because preterm labor is likely to be the start of an extended labor, a woman with symptoms can wait several hours before contacting the primary caregiver. d. Diagnosis of preterm labor is based on gestational age, uterine activity, and progressive cervical change.

ANS: D Gestational age of 20 to 37 weeks, uterine contractions, and a cervix that is 80% effaced or dilated 2 cm indicates preterm labor. It is essential that nurses teach women how to detect the early symptoms of preterm labor. Braxton Hicks contractions resemble preterm labor contractions, but they are not true labor. Waiting too long to see a health care provider could result in essential medications failing to be administered. Preterm labor is not necessarily long-term labor. DIF: Cognitive Level: Understand REF: p. 759 TOP: Nursing Process: Planning MSC: Client Needs: Safe and Effective Care Environment

When a nurse is counseling a woman for primary dysmenorrhea, which nonpharmacologic intervention might be recommended? a. Increasing the intake of red meat to replace blood loss b. Reducing the intake of diuretic foods, such as peaches and asparagus c. Temporarily substituting physical activity for a sedentary lifestyle d. Using a heating pad on the abdomen to relieve cramping

ANS: D Heat minimizes cramping by increasing vasodilation and muscle relaxation and minimizing uterine ischemia. Dietary changes such as a lowfat vegetarian diet may be recommended for women experiencing dysmenorrhea. Increasing the intake of diuretics, including natural diuretics such as asparagus, cranberry juice, peaches, parsley, and watermelon, may help ease the symptoms associated with dysmenorrhea. Exercise has been found to help relieve menstrual discomfort through increased vasodilation and subsequent decreased ischemia.

13. Which statement related to the induction of labor is most accurate? a. Can be achieved by external and internal version techniques b. Is also known as a trial of labor (TOL) c. Is almost always performed for medical reasons d. Is rated for viability by a Bishop score

ANS: D Induction of labor is likely to be more successful with a Bishop score of 9 or higher for first-time mothers or 5 or higher for veterans. Version is the turning of the fetus to a better position by a physician for an easier or safer birth. A TOL is the observance of a woman and her fetus for several hours of active labor to assess the safety of vaginal birth. Two thirds of cases of induced labor are elective and not done for medical reasons. DIF: Cognitive Level: Understand REF: p. 780 TOP: Nursing Process: Diagnosis MSC: Client Needs: Safe and Effective Care Environment

22. When would an internal version be indicated to manipulate the fetus into a vertex position? a. Fetus from a breech to a cephalic presentation before labor begins b. Fetus from a transverse lie to a longitudinal lie before a cesarean birth c. Second twin from an oblique lie to a transverse lie before labor begins d. Second twin from a transverse lie to a breech presentation during a vaginal birth

ANS: D Internal version is used only during a vaginal birth to manipulate the second twin into a presentation that allows it to be vaginally born. For internal version to occur, the cervix needs to be completely dilated. DIF: Cognitive Level: Remember REF: p. 779 TOP: Nursing Process: Assessment MSC: Client Needs: Physiologic Integrity

9. Because a full bladder prevents the uterus from contracting normally, nurses intervene to help the woman spontaneously empty her bladder as soon as possible. If all else fails, what tactic might the nurse use? a. Pouring water from a squeeze bottle over the womans perineum b. Placing oil of peppermint in a bedpan under the woman c. Asking the physician to prescribe analgesic agents d. Inserting a sterile catheter

ANS: D Invasive procedures are usually the last to be tried, especially with so many other simple and easy methods available (e.g., water, peppermint vapors, pain pills). Pouring water over the perineum may stimulate voiding. It is easy, noninvasive, and should be tried first. The oil of peppermint releases vapors that may relax the necessary muscles. It, too, is easy, noninvasive, and should be tried early on. If the woman is anticipating pain from voiding, then pain medications may be helpful. Other nonmedical means should be tried first, but medications still come before the insertion of a catheter.

Which alteration in cyclic bleeding best describes bleeding that occurs at any time other than menses? a. Oligomenorrhea b. Menorrhagia c. Leiomyoma d. Metrorrhagia

ANS: D Metrorrhagia (intermenstrual bleeding) refers to any episode or degree of bleeding that occurs between periods. It may be caused by contraceptives that contain progesterone or by intrauterine devices (IUDs).Oligomenorrhea is infrequent or scanty menstruation. Menorrhagia is excessive menstruation. Leiomyoma is a common cause of excessive bleeding.

Nafarelin (Synarel) is used to treat mildtosevere endometriosis. What instruction or information should the nurse provide to a client regarding nafarelin administration? a. Nafarelin stimulates the secretion of gonadotropin-releasing hormone (GnRH), thereby stimulating ovarian activity. b. It should be administered by intramuscular (IM) injection. c. Nafarelin should be administered by a subcutaneous implant. d. It can cause the client to experience some hot flashes and vaginal dryness.

ANS: D Nafarelin is a GnRH agonist, and its side effects are similar to those of menopause. The hypoestrogenism effect results in hot flashes and vaginal dryness. Nafarelin is a GnRH agonist that suppresses the secretion of GnRH. Nafarelin is administered twice daily by nasal spray and can be intranasally administered. Leuprolide is given once per month by IM injection. Goserelin is administered by subcutaneous implant.

During a prenatal intake interview, the nurse is in the process of obtaining an initial assessment of a 21-year-old Hispanic client with limited English proficiency. Which action is the most important for the nurse to perform? a.Use maternity jargon to enable the client to become familiar with these terms. b.Speak quickly and efficiently to expedite the visit. c.Provide the client with handouts. d.Assess whether the client understands the discussion.

ANS: D Nurses contribute to health literacy by using simple, common words, avoiding jargon, and evaluating whether the client understands the discussion. Speaking slowly and clearly and focusing on what is important will increase understanding. Most client education materials are written at a level too high for the average adult and may not be useful for a client with limited English proficiency.

4. A 25-year-old multiparous woman gave birth to an infant boy 1 day ago. Today her husband brings a large container of brown seaweed soup to the hospital. When the nurse enters the room, the husband asks for help with warming the soup so that his wife can eat it. What is the nurses most appropriate response? a. Didnt you like your lunch? b. Does your physician know that you are planning to eat that? c. What is that anyway? d. I'll warm the soup in the microwave for you.

ANS: D Offering to warm the food shows cultural sensitivity to the dietary preferences of the woman and is the most appropriate response. Cultural dietary preferences must be respected. Women may request that family members bring favorite or culturally appropriate foods to the hospital. Asking the woman to identify her food does not show cultural sensitivity.

Group B Streptococcus (GBS) is part of the normal vaginal flora in 20% to 30% of healthy pregnant women. GBS has been associated with poor pregnancy outcomes and is an important factor in neonatal morbidity and mortality. Which finding is not a risk factor for neonatal GBS infection? a.Positive prenatal culture b.Preterm birth at 37 weeks or less of gestation c.Maternal temperature of 38° C or higher d.Premature rupture of membranes (PROM) 24 hours or longer before the birth

ANS: D PROM 18 hours or longer before the birth increases the risk for neonatal GBS infection. Positive prenatal culture is a risk factor for neonatal GBS infection. Preterm birth at 37 weeks or less of gestation remains a risk factor for neonatal GBS infection. Maternal temperature of 38° C or higher is also a risk factor for neonatal GBS infection.

Syphilis is a complex disease that can lead to serious systemic illness and even death if left untreated. Which manifestation differentiates primary syphilis from secondary syphilis? a.Fever, headache, and malaise b.Widespread rash c.Identified by serologic testing d.Appearance of a chancre 2 months after infection

ANS: D Primary syphilis is characterized by a primary lesion (the chancre), which appears 5 to 90 days after infection. The chancre begins as a painless papule at the site of inoculation and erodes to form a nontender, shallow, and clean ulcer several millimeters to centimeters in size. Secondary syphilis occurs 6 weeks to 6 months after the appearance of the chancre and is characterized by a widespread maculopapular rash. The individual may also experience fever, headache, and malaise. Latent syphilis are those infections that lack clinical manifestations; however, they are detected by serologic testing.

Which risk factor would the nurse recognize as being frequently associated with osteoporosis? a. African-American race b. Lowprotein intake c. Obesity d. Cigarette smoking

ANS: D Smoking is associated with earlier and greater bone loss and decreased estrogen production. Women at risk for osteoporosis are likely to be Caucasian or Asian. Inadequate calcium intake is a risk factor for osteoporosis. Women at risk for osteoporosis are likely to be small boned and thin. Obese women have higher estrogen levels as a result of the conversion of androgens in the adipose tissue. Mechanical stress from extra weight also helps preserve bone mass.

Which system responses would the nurse recognize as being unrelated to prostaglandin (PGF2) release? a. Systemic responses b. Gastrointestinal system c. Central nervous system d. Genitourinary system

ANS: D Systemic responses to PGF2 include backache, weakness, and sweating. Gastrointestinal system changes include nausea, vomiting, anorexia, and diarrhea. Central nervous system changes manifest themselves as dizziness, syncope, headache, and poor concentration; they usually begin at the onset of menstruation and last 8 to 48 hours.

4. A primiparous woman is delighted with her newborn son and wants to begin breastfeeding as soon as possible. How should the client be instructed to position the infant to facilitate correct latch-on? a. The infant should be positioned with his or her arms folded together over the chest. b. The infant should be curled up in a fetal position. c. The woman should cup the infants head in her hand. d. The infants head and body should be in alignment with the mother.

ANS: D The infants head and body should be in correct alignment with the mother and the breast during latch-on and feeding. The infant should be facing the mother with his arms hugging the breast. The babys body should be held in correct alignment (i.e., ears, shoulder, and hips in a straight line) during feedings. The mother should support the babys neck and shoulders with her hand and not push on the occiput.

A client currently uses a diaphragm and spermicide for contraception. She asks the nurse to explain the major differences between the cervical cap and the diaphragm. What is the most appropriate response by the nurse? a. "No spermicide is used with the cervical cap, so it's less messy." b. "The diaphragm can be left in place longer after intercourse." c. "Repeated intercourse with the diaphragm is more convenient." d. "The cervical cap can be safely used for repeated acts of intercourse without adding more spermicide later."

ANS: D The cervical cap can be inserted hours before sexual intercourse without the need for additional spermicide later. Spermicide should be used inside the cap as an additional chemical barrier. The cervical cap should remain in place for 6 hours after the last act of intercourse. Repeated intercourse with the cervical cap is more convenient because no additional spermicide is needed.

Nurses should be cognizant of what information regarding the non-contraceptive medical effects of combination oral contraceptives (COCs)? a. COCs can cause TSS if the prescription is wrong. b. Hormonal withdrawal bleeding is usually a little more profuse than in normal menstruation and lasts a week for those who use COCs. c. COCs increase the risk of endometrial and ovarian cancers. d. Effectiveness of COCs can be altered by some over-the-counter medications and herbal supplements.

ANS: D The effectiveness of COCs can be altered by some over-the-counter medications and herbal supplements. TSS can occur in some who use the diaphragm, but it is not a consequence of taking oral contraceptive pills. Hormonal withdrawal bleeding usually is lighter than in normal menstruation and lasts a couple of days. Oral contraceptive pills offer protection against the risk of endometrial and ovarian cancers.

2. A man smokes two packs of cigarettes a day. He wants to know if smoking is contributing to the difficulty he and his wife are having getting pregnant. Which guidance should the nurse provide? a. "Your sperm count seems to be okay in the first semen analysis." b. "Only marijuana cigarettes affect sperm count." c. "Although smoking has no effect on sperm count, it can give you lung cancer." d. "Smoking can reduce the quality of your sperm."

ANS: D Cigarette smoking has detrimental effects on sperm and has been associated with abnormal sperm, a decreased number of sperm, and chromosomal damage. The nurse may suggest a smoking cessation program to increase the fertility of the male partner. Sperm counts vary from day to day and are dependent on emotional and physical status and sexual activity. Therefore, a single analysis may be inconclusive. A minimum of two analyses must be performed several weeks apart to assess male fertility. Marijuana use may depress the number and motility of sperm. Smoking is indeed a causative agent for lung cancer.

6. Male fertility declines slowly after age 40 years; however, no cessation of sperm production analogous to menopause in women occurs in men. What condition is not associated with advanced paternal age? a. Autosomal dominant disorder b. Schizophrenia c. Autism spectrum disorder d. Down syndrome

ANS: D Paternal age older than 40 years is associated with an increased risk for autosomal dominant disorder, schizophrenia, and autism spectrum disorder in their offspring. Although Down syndrome can occur in any pregnancy, it is often associated with advanced maternal age.

TRUE/FALSE 1. Researchers have found that most client education materials used are written at too high a reading level for the average adult. Is this true or false?

ANS: T As a result of the increasing multicultural U.S. population, there is an urgent need to address health literacy as a component of culturally and linguistically competent care. Health care providers contribute to health literacy by using simple common words, avoiding jargon, and developing appropriate written materials.

3. Which client is most likely to experience strong and uncomfortable afterpains? a. A woman who experienced oligohydramnios b. A woman who is a gravida 4, para 4-0-0-4 c. A woman who is bottle-feeding her infant d. A woman whose infant weighed 5 pounds, 3 ounces

B (Afterpains are more common in multiparous women. In a woman who experienced polyhydramnios, afterpains are more noticeable because the uterus was greatly distended. Breastfeeding may cause the afterpains to intensify. In a woman who delivered a large infant, afterpains are more noticeable because the uterus was greatly distended.)

Many first-time parents do not plan on their parents' help immediately after the newborn arrives. What statement by the nurse is the most appropriate when counseling new parents about the involvement of grandparents? A. "You should tell your parents to leave you alone." B. "Grandparents can help you with parenting skills and also help preserve family traditions." C. "Grandparent involvement can be very disruptive to the family." D. "They are getting old. You should let them be involved while they can."

B A. Incorrect: Intergenerational help may be perceived as interference, but a statement of this sort is not therapeutic to the adaptation of the family. B. Correct: This is the most appropriate response. C. Incorrect: Not only is this statement invalid, it also is not an appropriate nursing response. D. Incorrect: Regardless of age, grandparents can help with parenting skills and preserve family traditions. This is not the most appropriate statement, and it does not demonstrate sensitivity on the part of the nurse. p. 631

With regard to the adaptation of other family members, mainly siblings and grandparents, to the newborn, nurses should be aware that: A. Sibling rivalry cannot be dismissed as overblown psychobabble; negative feelings and behaviors can take a long time to blow over. B. Participation in preparation classes helps both siblings and grandparents. C. In the United States, paternal and maternal grandparents consider themselves of equal importance and status. D. Since 1990, the number of grandparents providing permanent care to their grandchildren has been declining.

B A. Incorrect: Sibling rivalry should be expected initially, but the negative behaviors associated with it have been overemphasized and stop in a comparatively short time. B. Correct: Preparing older siblings, as well as grandparents, helps with everyone's adaptation. C. Incorrect: In the United States, in contrast to other cultures, paternal grandparents frequently consider themselves secondary to maternal grandparents. D. Incorrect: The number of grandparents providing permanent child care has been rising. pp. 629-630

When dealing with parents who have some form of sensory impairment, nurses should realize that all of these statements are true except: A. One of the major difficulties visually impaired parents experience is the skepticism of health care professionals. B. Visually impaired mothers cannot overcome the infant's need for eye-to-eye contact. C. The best approach for the nurse is to assess the parents' capabilities rather than focusing on their disabilities. D. Technologic advances, including the Internet, can provide deaf parents with a full range of parenting activities and information.

B A. Incorrect: The skepticism, open or hidden, of health care professionals throws up an additional and unneeded hurdle for the parents. B. Correct: Other sensory output can be provided by the parent, other people can participate, and other coping devices can be used. C. Incorrect: After the parents' capabilities have been assessed (including some the nurse may not have expected), the nurse can help find ways to assist the parents that play to their strengths. D. Incorrect: The Internet affords an extra teaching tool for the deaf, as do videos with subtitles or nurses signing. A number of electronic devices can turn sound into light flashes to help pick up a child's cry. Sign language is acquired readily by young children. p. 628

In follow-up appointments or visits with parents and their new baby, it may be useful if the nurse can identify parental behaviors that can either facilitate or inhibit attachment. What is a facilitating behavior? A. The parents have difficulty naming the infant. B. The parents hover around the infant, directing attention to and pointing at the infant. C. The parents make no effort to interpret the actions or needs of the infant. D. The parents do not move from fingertip touch to palmar contact and holding.

B A. Incorrect: This would be an inhibiting behavior. B. Correct: Hovering over the infant, as well as obviously paying attention to the baby, is a facilitating behavior. C. Incorrect: This would be an inhibiting behavior. D. Incorrect: This would be an inhibiting behavior. p. 614

27. What is the correct terminology for the nerve block that provides anesthesia to the lower vagina and perineum? a. Epidural b. Pudendal c. Local d. Spinal block

B (A pudendal block anesthetizes the lower vagina and perineum to provide anesthesia for an episiotomy and the use of low forceps, if needed. An epidural provides anesthesia for the uterus, perineum, and legs. A local provides anesthesia for the perineum at the site of the episiotomy. A spinal block provides anesthesia for the uterus, perineum, and down the legs.)

13. Anxiety is commonly associated with pain during labor. Which statement regarding anxiety is correct? a. Even mild anxiety must be treated. b. Severe anxiety increases tension, increases pain, and then, in turn, increases fear and anxiety, and so on. c. Anxiety may increase the perception of pain, but it does not affect the mechanism of labor. d. Women who have had a painful labor will have learned from the experience and have less anxiety the second time because of increased familiarity.

B (Anxiety and pain reinforce each other in a negative cycle that will slow the progress of labor. Mild anxiety is normal for a woman in labor and likely needs no special treatment other than the standard reassurances. Anxiety increases muscle tension and ultimately can sufficiently build to slow the progress of labor. Unfortunately, an anxious, painful first labor is likely to carry over, through expectations and memories, into an anxious and painful experience in the second pregnancy.)

8. What three measures should the nurse implement to provide intrauterine resuscitation? a. Call the provider, reposition the mother, and perform a vaginal examination. b. Turn the client onto her side, provide oxygen (O2) via face mask, and increase intravenous (IV) fluids. c. Administer O2 to the mother, increase IV fluids, and notify the health care provider. d. Perform a vaginal examination, reposition the mother, and provide O2 via face mask.

B (Basic interventions for the management of any abnormal FHR pattern include administering O2 via a nonrebreather face mask at a rate of 8 to 10 L/min, assisting the woman onto a side-lying (lateral) position, and increasing blood volume by increasing the rate of the primary IV infusion. The purpose of these interventions is to improve uterine blood flow and intervillous space blood flow and to increase maternal oxygenation and cardiac output. The term intrauterine resuscitation is sometimes used to refer to these interventions. If these interventions do not quickly resolve the abnormal FHR issue, then the primary provider should be immediately notified.)

26. Developing a realistic birth plan with the pregnant woman regarding her care is important for the nurse. How would the nurse explain the major advantage of nonpharmacologic pain management? a. Greater and more complete pain relief is possible. b. No side effects or risks to the fetus are involved. c. The woman will remain fully alert at all times. d. Labor will likely be more rapid.

B (Because nonpharmacologic pain management does not include analgesics, adjunct drugs, or anesthesia, it is harmless to the mother and the fetus. However, pain relief is lessened with nonpharmacologic pain management during childbirth. Although the womans alertness is not altered by medication, the increase in pain may decrease alertness. Pain management may or may not alter the length of labor. At times when pain is decreased, the mother relaxes and labor progresses at a quicker pace.)

12. A new client and her partner arrive on the labor, delivery, recovery, and postpartum (LDRP) unit for the birth of their first child. The nurse applies the electronic fetal monitor (EFM) to the woman. Her partner asks you to explain what is printing on the graph, referring to the EFM strip. He wants to know what the babys heart rate should be. What is the nurses best response? a. Dont worry about that machine; thats my job. b. The babys heart rate will fluctuate in response to what is happening during labor. c. The top line graphs the babys heart rate, and the bottom line lets me know how strong the contractions are. d. Your physician will explain all of that later.

B (Explaining what indicates a normal FHR teaches the partner about fetal monitoring and provides support and information to alleviate his fears. Telling the partner not to worry discredits his feelings and does not provide the teaching he is requesting. Telling the partner that the graph indicates how strong the contractions are provides inaccurate information and does not address the partners concerns about the FHR. The EFM graphs the frequency and duration of the contractions, not their intensity. Nurses should take every opportunity to provide teaching to the client and her family, especially when information is requested.)

20. Which characteristic correctly matches the type of deceleration with its likely cause? a. Early decelerationumbilical cord compression b. Late decelerationuteroplacental insufficiency c. Variable decelerationhead compression d. Prolonged decelerationunknown cause

B (Late deceleration is caused by uteroplacental insufficiency. Early deceleration is caused by head compression. Variable deceleration is caused by umbilical cord compression. Prolonged deceleration has a variety of either benign or critical causes.)

8. What is the role of the nurse as it applies to informed consent? a. Inform the client about the procedure, and ask her to sign the consent form. b. Act as a client advocate, and help clarify the procedure and the options. c. Call the physician to see the client. d. Witness the signing of the consent form.

B (Nurses play a part in the informed consent process by clarifying and describing procedures or by acting as the womans advocate and asking the primary health care provider for further explanations. The physician is responsible for informing the woman of her options, explaining the procedure, and advising the client about potential risk factors. The physician must be present to explain the procedure to the client. However, the nurses responsibilities go further than simply asking the physician to see the client. The nurse may witness the signing of the consent form. However, depending on the states guidelines, the womans husband or another hospital health care employee may sign as a witness.)

18. According to professional standards (the Association of Womens Health, Obstetric and Neonatal Nurses [AWHONN], 2007), which action cannot be performed by the nonanesthetist registered nurse who is caring for a woman with epidural anesthesia? a. Monitoring the status of the woman and fetus b. Initiating epidural anesthesia c. Replacing empty infusion bags with the same medication and concentrate d. Stopping the infusion, and initiating emergency measures

B (Only qualified, licensed anesthesia care providers are permitted to insert a catheter, initiate epidural anesthesia, verify catheter placement, inject medication through the catheter, or alter the medication or medications including type, amount, or rate of infusion. The nonanesthetist nurse is permitted to monitor the status of the woman, the fetus, and the progress of labor. Replacement of the empty infusion bags or syringes with the same medication and concentration is permitted. If the need arises, the nurse may stop the infusion, initiate emergency measures, and remove the catheter if properly educated to do so. Complications can require immediate interventions. Nurses must be prepared to provide safe and effective care during an emergency situation.)

6. What should the laboring client who receives an opioid antagonist be told to expect? a. Her pain will decrease. b. Her pain will return. c. She will feel less anxious. d. She will no longer feel the urge to push.

B (Opioid antagonists such as naloxone (Narcan) promptly reverse the CNS-depressant effects of opioids. In addition, the antagonist counters the effect of the stress-induced levels of endorphins. An opioid antagonist is especially valuable if the labor is more rapid than expected and the birth is anticipated when the opioid is at its peak effect. The woman should be told that the pain that was relieved by the opioid analgesic will return with the administration of the opioid antagonist. Her pain level will increase rather than decrease. Opioid antagonists have no effect on anxiety levels. They are primarily administered to reverse the excessive CNS depression in the mother, newborn, or both. An opioid antagonist (e.g., naloxone) has no effect on the mothers urge or ability to push. The practice of giving lower doses of IV opioids has reduced the incidence and severity of opioid-induced CNS depression; therefore, opioid antagonists are used less frequently.)

9. A first-time mother is concerned about the type of medications she will receive during labor. The client is in a fair amount of pain and is nauseated. In addition, she appears to be very anxious. The nurse explains that opioid analgesics are often used along with sedatives. How should the nurse phrase the rationale for this medication combination? a. The two medications, together, reduce complications. b. Sedatives enhance the effect of the pain medication. c. The two medications work better together, enabling you to sleep until you have the baby. d. This is what your physician has ordered for you.

B (Sedatives may be used to reduce the nausea and vomiting that often accompany opioid use. In addition, some ataractic drugs reduce anxiety and apprehension and potentiate the opioid analgesic affects. A potentiator may cause two drugs to work together more effectively, but it does not ensure zero maternal or fetal complications. Sedation may be a related effect of some ataractic drugs; however, sedation is not the goal. Furthermore, a woman is unlikely to be able to sleep through transitional labor and birth. Although the physician may have ordered the medication, This is what your physician has ordered for you is not an acceptable comment for the nurse to make.)

17. During labor a fetus displays an average FHR of 135 beats per minute over a 10-minute period. Which statement best describes the status of this fetus? a. Bradycardia b. Normal baseline heart rate c. Tachycardia d. Hypoxia

B (The baseline FHR is measured over 10 minutes; a normal range is 110 to 160 beats per minute. Bradycardia is a FHR less than 110 beats per minute for 10 minutes or longer. Tachycardia is a FHR higher than 160 beats per minutes for 10 minutes or longer. Hypoxia is an inadequate supply of oxygen; no indication of hypoxia exists with a baseline FHR in the normal range.)

25. The nurse should be aware of what important information regarding systemic analgesics administered during labor? a. Systemic analgesics cross the maternal blood-brain barrier as easily as they do the fetal blood-brain barrier. b. Effects on the fetus and newborn can include decreased alertness and delayed sucking. c. Intramuscular (IM) administration is preferred over IV administration. d. IV patient-controlled analgesia (PCA) results in increased use of an analgesic.

B (The effects of analgesics depend on the specific drug administered, the dosage, and the timing. Systemic analgesics cross the fetal blood-brain barrier more readily than the maternal blood-brain barrier. IV administration is preferred over IM administration because the drug acts faster and more predictably. PCA results in a decrease in the use of an analgesic.)

29. What is the rationale for the use of a blood patch after spinal anesthesia? a. Hypotension b. Headache c. Neonatal respiratory depression d. Loss of movement

B (The subarachnoid block may cause a postspinal headache resulting from the loss of cerebrospinal fluid from the puncture in the dura. When blood is injected into the epidural space in the area of the dural puncture, it forms a seal over the hole to stop the leaking of cerebrospinal fluid. Hypotension is prevented by increasing fluid volume before the procedure. Neonatal respiratory depression is not an expected outcome with spinal anesthesia. Loss of movement is an expected outcome of spinal anesthesia.)

4. What is the most likely cause for variable FHR decelerations? a. Altered fetal cerebral blood flow b. Umbilical cord compression c. Uteroplacental insufficiency d. Fetal hypoxemia

B (Variable FHR decelerations can occur at any time during the uterine contracting phase and are caused by compression of the umbilical cord. Altered fetal cerebral blood flow results in early decelerations in the FHR. Uteroplacental insufficiency results in late decelerations in the FHR. Fetal hypoxemia initially results in tachycardia and then bradycardia if hypoxia continues.)

Which statement by the student nurse about pelvic inflammatory disease (PID) indicates effective learning? a. PID causes miscarriage. b. The menstrual period facilitates the development of PID. c. N. Gonorrhoeae is the only organism that causes PID. d. PID occurs as organisms spread from the upper genital tract to the vagina.

B During the menstrual period, the cervical os is slightly open and the cervical mucus barrier is absent. In addition, menstrual blood is an excellent medium for the growth of organisms that cause PID. PID may also develop after a miscarriage or an induced abortion. A wide variety of anaerobic and aerobic bacteria, including N. Gonorrhoeae, may cause PID. C. trachomatis is now estimated to cause half of all cases of PID. PID results from the spread of microorganisms from the vagina and endocervix to the upper genital tract. P. 153

The nurse suspects that a client with frequent complaints of diarrhea, weight loss, and rash has human immunodeficiency virus (HIV) infection. The client is afraid of having blood drawn for screening purposes. What does the nurse tell the client? a. "You can take some time to prepare for this test." b. "You can provide a saliva sample for testing." c. "A blood test is the only means of detecting the virus." d. "If you don't get this test done, the virus may prove fatal."

B If a client is unwilling to take a blood test, an oral fluid sample can also be obtained for testing for the presence of HIV infection. It has a sensitivity and specificity rate of more than 99%. The nurse should not advise the client to take time to prepare for the test, because the client may be infected, and prompt treatment may be required. The blood test is not the sole means of detecting the virus. The client can provide a blood sample obtained by fingerstick or venipuncture, an oral fluid sample, or a urine sample for testing. Telling the client that the virus may prove fatal will increase the client's anxiety. Instead the nurse should inform the client about alternative methods of testing. p. 160

17. Which statement by a newly delivered woman indicates that she knows what to expect regarding her menstrual activity after childbirth? a. My first menstrual cycle will be lighter than normal and then will get heavier every month thereafter. b. My first menstrual cycle will be heavier than normal and will return to my prepregnant volume within three or four cycles. c. I will not have a menstrual cycle for 6 months after childbirth. d. My first menstrual cycle will be heavier than normal and then will be light for several months after.

B (My first menstrual cycle will be heavier than normal and will return to my prepregnant volume within three or four cycles is an accurate statement and indicates her understanding of her expected menstrual activity. She can expect her first menstrual cycle to be heavier than normal, which occurs by 3 months after childbirth, and the volume of her subsequent cycles will return to prepregnant levels within three to four cycles.)

13. The nurse should be cognizant of which postpartum physiologic alteration? a. Cardiac output, pulse rate, and stroke volume all return to prepregnancy normal values within a few hours of childbirth. b. Respiratory function returns to nonpregnant levels by 6 to 8 weeks after childbirth. c. Lowered white blood cell count after pregnancy can lead to false-positive results on tests for infections. d. Hypercoagulable state protects the new mother from thromboembolism, especially after a cesarean birth.

B (Respirations should decrease to within the womans normal prepregnancy range by 6 to 8 weeks after childbirth. Stroke volume increases and cardiac output remains high for a couple of days. However, the heart rate and blood pressure quickly return to normal. Leukocytosis increases 10 to 12 days after childbirth, which can obscure the diagnosis of acute infections, producing false-negative test results. The hypercoagulable state increases the risk of thromboembolism, especially after a cesarean birth.)

11. Which statement regarding the postpartum uterus is correct? a. At the end of the third stage of labor, the postpartum uterus weighs approximately 500 g. b. After 2 weeks postpartum, it should be abdominally nonpalpable. c. After 2 weeks postpartum, it weighs 100 g. d. Postpartum uterus returns to its original (prepregnancy) size by 6 weeks postpartum.

B (The uterus does not return to its original size. At the end of the third stage of labor, the uterus weighs approximately 1000 g. After 2 weeks postpartum, the uterus weighs approximately 350 g. The normal self-destruction of excess hypertrophied tissue accounts for the slight increase in uterine size after each pregnancy.)

19. Which description of postpartum restoration or healing times is accurate? a. The cervix shortens, becomes firm, and returns to form within a month postpartum. b. Vaginal rugae reappear by 3 weeks postpartum. c. Most episiotomies heal within a week. d. Hemorrhoids usually decrease in size within 2 weeks of childbirth.

B (Vaginal rugae reappear by 3 weeks postpartum; however, they are never as prominent as in nulliparous women. The cervix regains its form within days; the cervical os may take longer. Most episiotomies take 2 to 3 weeks to heal. Hemorrhoids can take 6 weeks to decrease in size.)

Herbal preparations have long been used for the management of menstrual problems, including dysmenorrhea, cramping and discomfort, and breast pain. For the nurse to counsel adequately the client who elects to use this alternative modality, understanding the action of these herbal preparations is important. Match the herbal medicine with the appropriate action. a. Uterine antispasmodic b. Uterotonic c. Antiinflammatory d. Estrogen-like luteinizing hormone suppressant e. Decreases prolactin levels

B 1. Fennel, dong quai E 2. Chaste tree fruit D 3. Black cohosh root A 4. Valerian, wild yam C 5. Ginger

1. Maternal hypotension is a potential side effect of regional anesthesia and analgesia. What nursing interventions could the nurse use to increase the clients blood pressure? (Select all that apply.) a. Place the woman in a supine position. b. Place the woman in a lateral position. c. Increase IV fluids. d. Administer oxygen. e. Perform a vaginal examination.

B, C, D (Nursing interventions for maternal hypotension arising from analgesia or anesthesia include turning the woman to a lateral position, increasing IV fluids, administering oxygen via face mask, elevating the womans legs, notifying the physician, administering an IV vasopressor, and monitoring the maternal and fetal status at least every 5 minutes until the woman is stable. Placing the client in a supine position causes venous compression, thereby limiting blood flow to and oxygenation of the placenta and fetus. A sterile vaginal examination has no bearing on maternal blood pressure.)

Which statements about genital herpes are accurate? Select all that apply. a. Genital herpes is also known as genital warts. b. Stress, menstruation, trauma, and illnesses have been known to trigger recurrences. c. Genital herpes is chronic and recurring, and has no known cure. d. Plain soap and water are all that are needed to clean hands that have come into contact with herpetic lesions. e. Contact isolation is needed for clients with genital herpes.

B, C, D Recurrence of genital herpes can be triggered by stress and illness. The disease is chronic and has no known cure. Good handwashing can prevent the spread of infection from contact with the lesions. Genital warts are one of the most common sexually transmitted infections (STIs); however, they are also known as human papillomavirus (HPV), not genital herpes. Isolation is not appropriate for patients with genital herpes. p. 156-157

Which instructions are given to a client with genital herpes simplex virus (HSV) infection? Select all that apply. a. "Avoid the use of aspirin for pain relief." b. "Take warm sitz baths with baking soda." c. "Take 400 mg of acyclovir orally three times daily for a week." d. "Dry the lesions with cool air from a hair dryer." e. "Clean the lesions once a day with warm water."

B, C, D Warm sitz baths with baking soda increase the client's comfort when the lesions are active. The nurse would also provide the client teaching about the administration of acyclovir, famciclovir, or valacyclovir, which can help decrease the number of HSV infections. The client should dry the lesions by directing cool air from a hair dryer onto them or by patting them dry with a soft towel. Oral analgesics such as aspirin or ibuprofen can be used to relieve pain, because there are no side effects with these medications. The lesions need to be cleaned twice a day with saline to prevent secondary infection. P. 157

1. The breast-feeding mother should be taught to expect which changes to the condition of the breasts? (Select all that apply.) a. Breast tenderness is likely to persist for approximately 1 week after the start of lactation. b. As lactation is established, a mass may form that can be distinguished from cancer by its positional shift from day to day. c. In nonlactating mothers, colostrum is present for the first few days after childbirth. d. If suckling is never begun or is discontinued, then lactation ceases within a few days to a week. e. Little change occurs to the breasts in the first 48 hours.

B, C, D (Breasts become fuller and heavier as colostrum transitions to milk; this fullness should last 72 to 96 hours. The movable, noncancerous mass is a filled milk sac. Colostrum is present for a few days whether or not the mother breastfeeds. A mother who does not want to breastfeed should also avoid stimulating her nipples. Little change to the breasts occurs in the first 24 hours of childbirth.)

3. Which FHR decelerations would require the nurse to change the maternal position? (Select all that apply.) a. Early decelerations b. Late decelerations c. Variable decelerations d. Moderate decelerations e. Prolonged decelerations

B, C, E (Early decelerations (and accelerations) do not generally need any nursing intervention. Late decelerations suggest that the nurse should change the maternal position (lateral). Variable decelerations also require a maternal position change (side to side). Moderate decelerations are not an accepted category. Prolonged decelerations are late or variable decelerations that last for a prolonged period (longer than 2 minutes) and require intervention.)

Which findings would lead to increased bilirubin levels in the newborn? A. Cord clamped immediately following delivery of newborn B. Meconium passed after 24 hours C. Initiation of newborn feedings were delayed following birth D. Hyperglycemia E. Twin to twin transfusion syndrome

B, C, E Delay in passage of meconium and delay in newborn feedings could lead to increased bilirubin levels due to increased enterohepatic circulation. Twin to twin transfusion syndrome could lead to increased bilirubin levels due to increased amount of hemoglobin. An increase in bilirubin levels would be seen if cord clamping was delayed following birth. Hypoglycemia could lead to increased bilirubin levels due to alterations in hepatic function and perfusion

2. According to the National Institute of Child Health and Human Development (NICHD) Three-Tier System of FHR Classification, category III tracings include all FHR tracings not categorized as category I or II. Which characteristics of the FHR belong in category III? (Select all that apply.) a. Baseline rate of 110 to 160 beats per minute b. Tachycardia c. Absent baseline variability not accompanied by recurrent decelerations d. Variable decelerations with other characteristics such as shoulders or overshoots e. Absent baseline variability with recurrent variable decelerations f. Bradycardia

B, D, E, F (Tachycardia, variable decelerations with other characteristics, absent baseline variability with recurrent variable decelerations, and bradycardia are characteristics that are considered nonreassuring or abnormal and belong in category III. A FHR of 110 to 160 beats per minute is considered normal and belongs in category I. Absent baseline variability not accompanied by recurrent decelerations is a category II characteristic.)

What are the maternal effects of Chlamydia? Select all that apply. a. Meningitis b. Preterm labor c. Chorioamnionitis d. Postpartum sepsis e. Postpartum endometritis

B, E Chlamydia is a bacterial infection caused by Chlamydia trachomatis. The patient with Chlamydia has a risk of pelvic inflammatory disease, due to which the client may have preterm labor and postpartum endometritis. Meningitis, chorioamnionitis, and postpartum sepsis are most commonly seen in clients with gonorrhea. Chlamydia trachomatis does not affect the brain, fetal membranes, chorion, and amnion. p. 165

The nurse is caring for an infant with breathing difficulty. Upon auscultating, the infant the nurse finds that the infant has a murmur. What suggestion does the nurse give to the parents about infant care? A. "Use formula milk." B. "Additional cardiac testing is necessary." C. "The infant should be wrapped in a thick blanket." D. "Maintain skin-to-skin contact with the mother."

B. "Additional cardiac testing is necessary." Typically, the presence of cardiac murmurs in infants has no pathologic significance. However, when murmurs are associated with other conditions, such as breathing difficulty, which may cause apnea and cyanosis, they are considered abnormal. In this case the primary health care provider will send the child for cardiac testing to diagnose any more serious condition. While skin-to-skin contact is useful in enhancing thermoregulation in infants, it will not have any effect on heart murmurs. Wrapping the infant in a thick blanket prevents cold distress in the infant, but does not affect the cardiac murmur. Feeding the infant with formula milk is unrelated to cardiac murmurs.

Which statement made by a lactating woman leads the nurse to believe that the woman might have lactose intolerance? a."I always have heartburn after I drink milk." b."If I drink more than a cup of milk, I usually have abdominal cramps and bloating." c."Drinking milk usually makes me break out in hives." d."Sometimes I notice that I have bad breath after I drink a cup of milk."

B. "If I drink more than a cup of milk, I usually have abdominal cramps and bloating."

A client tells the nurse, "While crying, my baby often moves its hand towards its mouth and also gets startled by the sound of the rattle." What statement given by the nurse best explains this behavior? "The baby: A. "Is hungry." B. "Is consoling itself." C. "Wants to interact with you." D. "Is frightened by some noise."

B. "Is consoling itself." Newborns adopt one of several ways to console themselves to diminish their anguish. Making hand-to-mouth movements and becoming alert to voices, noise, and visual stimuli are common observations and indications of consoling. Hunger, grabbing attention, and phonophobia are the reasons for why the infant cries.

To prevent gastrointestinal (GI) upset, clients should be instructed to take iron supplements: a.On a full stomach b.At bedtime c.After eating a meal d.With milk

B. At bedtime

Fertilization takes place in the _____________. A. Ovaries B. Fallopian tube C. Uterus

B. Fallopian tube

True or False: Genetics is the study of genes and their function and related technology. A. True B. False

B. False

The nurse is caring for a neonate immediately after delivery. What does the nurse expect to find while assessing the neonate during the first 30 minutes after birth? A. Heart rate increases from 100 to 120 beats/minute. B. Fine crackles may be present on auscultation. C. Peristaltic waves may benoted over the abdomen. D. Respirations are shallow and may reach up to 60 breaths/minute.

B. Fine crackles may be present on auscultation. The first stage of the transition period lasts for up to 30 minutes after birth. During this period, fine crackles may be noted on auscultation. The newborn's heart rate increases rapidly from 160 to 180 beats/minute. After the first stage of the transition period, the neonate may maintain a baseline rate of 100 to 120 beats/minute. Though bowel sounds are audible in the first 30 minutes, peristaltic waves may be noted over the abdomen only after the first 30 minutes. An irregular respiratory rate between 60 to 80 breaths/minute may be noted during the first 30 minutes. After 30 minutes, respirations usually become rapid and shallow and reach up to 60 breaths/minute.

The nurse is caring for an infant experiencing cold stress. Which complication does the nurse suspect in the infant? A. Hyperglycemia B. Hyperbilirubinemia C. Respiratory alkalosis D. Decreased metabolic rate

B. Hyperbilirubinemia As a result, excessive fatty acids may be produced displacing the bilirubin from the albumin binding sites, leading to hyperbilirubinemia. In addition, cold stress may also result in excessive glycolysis. This in turn reduces the blood glucose levels and causes hypoglycemia instead of hyperglycemia. Due to the increased production of acids during cold stress, infants have respiratory acidosis rather than respiratory alkalosis. During cold stress, the metabolic rate usually increases rather than decreasing, to cause thermogenesis.

The developing human is most vulnerable to teratogens during A. The first 4 weeks of gestation B. The first 8 weeks of gestation C. The first 12 weeks of gestation D. The first 16 weeks of gestation

B. The first 8 weeks of gestation

What is the basic mechanism for conserving internal heat within infants? A. Shivering B. Vasoconstriction C. Metabolism of brown fat D. Decrease in muscle activity

B. Vasoconstriction The posterior hypothalamus stimulates the sympathetic nervous system and initiates powerful vasoconstriction throughout the body. This results in decreased conduction of heat from the internal core to the skin. Production of heat through shivering mechanism is rarely operable in infants unless there is prolonged cold exposure. Newborns generate heat by metabolism of brown fat, which is a unique feature and is not possible in infants. Brown fat has a richer vascular and nerve supply than ordinary fat. Heat produced by intense lipid metabolic activity in brown fat can warm the newborn by increasing heat production as much as 100%. Reserves of brown fat, usually present for several weeks after birth, are rapidly depleted with cold stress. In response to cold the neonate attempts to generate heat (thermogenesis) by increasing muscle activity, but not by decreasing muscle activity.

An examiner who discovers unequal movement or uneven gluteal skinfolds during the Ortolani maneuver: A. tells the parents that one leg may be longer than the other, but they will equal out by the time the infant is walking. B. alerts the physician that the infant may have a dislocated hip. C. informs the parents and physician that molding has not taken place. D. suggests that if the condition does not change, surgery to correct vision problems might be needed.

B. alerts the physician that the infant may have a dislocated hip. The Ortolani maneuver is a technique for checking hip integrity. Unequal movement suggests that the hip is dislocated. The physician should be notified. Telling the parents that one of the infant's legs might be longer than the other is an inappropriate statement that may result in unnecessary anxiety for the new parents. Molding refers to movement of the cranial bones and has nothing to do with the infant's hips. The Ortolani maneuver is not a technique used to evaluate visual acuity in the newborn. This maneuver checks hip integrity.

The nurse helps a breastfeeding mother change the diaper of her 16-hour-old newborn after the first bowel movement. The mother expresses concern because the large amount of thick, sticky stool is very dark green, almost black in color. She asks the nurse if something is wrong. The nurse should respond to this mother's concern by: A. telling the mother not to worry because breastfed babies have this type of stool. B. explaining to the mother that the stool is called meconium and is expected of all newborns for the first few bowel movements. C. asking the mother what she ate at her last meal. D. suggesting that the mother ask her pediatrician to explain newborn stool patterns to her.

B. explaining to the mother that the stool is called meconium and is expected of all newborns for the first few bowel movements. At this early age this type of stool (meconium) is typical of both bottle-fed and breastfed newborns. This type of stool is the first stool that all newborns have, not just breastfed babies. The mother's nutritional intake is not responsible for the appearance of meconium stool. The nurse is fully capable of and responsible for teaching a new mother about the characteristics of her newborn, including expected stool patterns.

Obstetricians today are seeing more morbidly obese pregnant women (those that weigh 400 lb or greater). A new medical subspecialty referred to as ____________ obstetrics has subsequently arisen.

Bariatric

Which infections are collectively known as TORCH infections? Select all that apply. a. Chlamydia b. Gonorrhea c. Toxoplasmosis d. German measles e. Cytomegalovirus f. Herpes genitalis

C, D, E, F Toxoplasmosis, German measles, cytomegalovirus, and herpes genitalis are collectively known as TORCH infections. The causative agents of these infections cross the placenta and cause influenza-like symptoms in the mother and significant birth defects in the newborn. Chlamydia and gonorrhea do not cause significant birth defects and fetal death. Therefore, Chlamydia and gonorrhea are not TORCH infections. p. 165

The nurse is developing a plan of care for a Hispanic client who just delivered a newborn. Which cultural variation is most important to include in the care plan? a. Breastfeeding is encouraged immediately after birth. b. Male infants are typically circumcised. c. Maternal grandmother participates in the care of the mother and her infant. d. Bathing is encouraged immediately after delivery.

C

Which traditional family structure is decreasing in numbers and attributable to societal changes? a. Extended family b. Binuclear family c. Nuclear family d. Blended family

C

Of the many factors that influence parental responses, nurses should be aware that all of these statements regarding age are true except: A. An adolescent mother's egocentricity and unmet developmental needs interfere with her ability to parent effectively. B. An adolescent mother is likely to use less verbal instruction, be less responsive, and interact less positively than other mothers. C. Adolescent mothers have a higher documented incidence of child abuse. D. Mothers older than 35 often deal with more stress related to work and career issues, as well as decreasing libido.

C A. Incorrect: Adolescent mothers are more inclined to have a number of parenting difficulties that benefit from counseling, but a higher incidence of child abuse is not one of them. B. Incorrect: Adolescent mothers are more inclined to have a number of parenting difficulties that benefit from counseling, but a higher incidence of child abuse is not one of them. C. Correct: Adolescent mothers are more inclined to have a number of parenting difficulties that benefit from counseling, but a higher incidence of child abuse is not one of them. D. Incorrect: Midlife mothers have many competencies but are more likely to have to deal with career and sexual issues than are younger mothers. p. 625

With regard to parents' early and extended contact with their infant and the relationships built, nurses should be aware that: A. Immediate contact is essential for the parent-child relationship. B. Skin-to-skin contact is preferable to contact with the body totally wrapped in a blanket. C. Extended contact is especially important for adolescents and low-income women because they are at risk for parenting inadequacies. D. Mothers need to take precedence over their partners and other family matters.

C A. Incorrect: Immediate contact facilitates the attachment process but is not essential; otherwise, adopted infants would not establish the affectionate ties they do. B. Incorrect: The mode of infant-mother contact does not appear to have any important effect. C. Correct: Nurses should encourage any activity that optimizes family extended contact. D. Incorrect: Mothers and their partners are considered equally important. p. 616

The nurse hears a primiparous woman talking to her son and telling him that his chin is just like his dad's chin. This woman's statement reflects: A. Mutuality B. Synchrony C. Claiming D. Reciprocity

C A. Incorrect: Mutuality occurs when the infant's behaviors and characteristics call forth a corresponding set of maternal behaviors and characteristics. B. Incorrect: Synchrony refers to the "fit" between the infant's cues and the parent's responses. C. Correct: Claiming refers to the process by which the child is identified in terms of likeness to other family members. D. Incorrect: Reciprocity is a type of body movement or behavior that provides the observer with cues. p. 613

The early postpartum period is a time of emotional and physical vulnerability. Many mothers can easily become psychologically overwhelmed by the reality of their new parental responsibilities. Fatigue compounds these issues. Although the baby blues are a common occurrence in the postpartum period, about one-half million women in America experience a more severe syndrome known as postpartum depression. Which statement regarding postpartum depression (PPD) is essential for the nurse to be aware of when attempting to formulate a nursing diagnosis? A. PPD symptoms are consistently severe B. This syndrome affects only new mothers C. PPD can easily go undetected D. Only mental health professionals should teach new parents about this condition

C A. Incorrect: PPD symptoms range from mild to severe, with women having both good day and bad days. B. Incorrect: Screening should be done for both mothers and fathers. PPD in new fathers ranges from 1% to 26%. C. Correct: PPD can go undetected because parents do not voluntarily admit to this type of emotional distress out of embarrassment, fear, or guilt. D. Incorrect: The nurse should include information on PPD and how to differentiate this from the baby blues for all clients on discharge. Nurses also can urge new parents to report symptoms and seek follow-up care promptly if they occur. pp. 621, 622

17. A woman in labor is breathing into a mouthpiece just before the start of her regular contractions. As she inhales, a valve opens and gas is released. She continues to inhale the gas slowly and deeply until the contraction starts to subside. When the inhalation stops, the valve closes. Which statement regarding this procedure is correct? a. The application of nitrous oxide gas is not often used anymore. b. An inhalation of gas is likely to be used in the second stage of labor, not during the first stage. c. An application of nitrous oxide gas is administered for pain relief. d. The application of gas is a prelude to a cesarean birth.

C (A mixture of nitrous oxide with oxygen in a low concentration can be used in combination with other nonpharmacologic and pharmacologic measures for pain relief. This procedure is still commonly used in Canada and in the United Kingdom. Nitrous oxide inhaled in a low concentration will reduce but not eliminate pain during the first and second stages of labor. Nitrous oxide inhalation is not generally used before a caesarean birth. Nitrous oxide does not appear to depress uterine contractions or cause adverse reactions in the newborn.)

14. Which statement is not an expected outcome for the client who attends a reputable childbirth preparation program? a. Childbirth preparation programs increase the womans sense of control. b. Childbirth preparation programs prepare a support person to help during labor. c. Childbirth preparation programs guarantee a pain-free childbirth. d. Childbirth preparation programs teach distraction techniques.

C (All methods try to increase a womans sense of control, prepare a support person, and train the woman in physical conditioning, which includes breathing techniques. These programs cannot, and reputable ones do not, promise a pain-free childbirth. Increasing a womans sense of control is the goal of all childbirth preparation methods. Preparing a support person to help in labor is a vitally important component of any childbirth education program. The coach may learn how to touch a womans body to detect tense and contracted muscles. The woman then learns how to relax in response to the gentle stroking by the coach. Distraction techniques are a form of care that are effective to some degree in relieving labor pain and are taught in many childbirth programs. These distractions include imagery, feedback relaxation, and attention-focusing behaviors.)

5. A laboring woman has received meperidine (Demerol) intravenously (IV), 90 minutes before giving birth. Which medication should be available to reduce the postnatal effects of meperidine on the neonate? a. Fentanyl (Sublimaze) b. Promethazine (Phenergan) c. Naloxone (Narcan) d. Nalbuphine (Nubain)

C (An opioid antagonist can be given to the newborn as one part of the treatment for neonatal narcosis, which is a state of central nervous system (CNS) depression in the newborn produced by an opioid. Opioid antagonists, such as naloxone (Narcan), can promptly reverse the CNS depressant effects, especially respiratory depression. Fentanyl (Sublimaze), promethazine (Phenergan), and nalbuphine (Nubain) do not act as opioid antagonists to reduce the postnatal effects of meperidine on the neonate.)

24. In which clinical situation would the nurse most likely anticipate a fetal bradycardia? a. Intraamniotic infection b. Fetal anemia c. Prolonged umbilical cord compression d. Tocolytic treatment using terbutaline

C (Fetal bradycardia can be considered a later sign of fetal hypoxia and is known to occur before fetal death. Bradycardia can result from placental transfer of drugs, prolonged compression of the umbilical cord, maternal hypothermia, and maternal hypotension. Intraamniotic infection, fetal anemia, and tocolytic treatment using terbutaline would most likely result in fetal tachycardia.)

12. Nurses with an understanding of cultural differences regarding likely reactions to pain may be better able to help their clients. Which clients may initially appear very stoic but then become quite vocal as labor progresses until late in labor, when they become more vocal and request pain relief? a. Chinese b. Arab or Middle Eastern c. Hispanic d. African-American

C (Hispanic women may be stoic early in labor but more vocal and ready for medications later. Chinese women may not show reactions to pain. Medical interventions must be offered more than once. Arab or Middle Eastern women may be vocal in response to labor pain from the start; they may prefer pain medications. African-American women may openly express pain; the use of medications for pain is more likely to vary with the individual.)

9. The nurse who provides care to clients in labor must have a thorough understanding of the physiologic processes of maternal hypotension. Which outcome might occur if the interventions for maternal hypotension are inadequate? a. Early FHR decelerations b. Fetal arrhythmias c. Uteroplacental insufficiency d. Spontaneous rupture of membranes

C (Low maternal blood pressure reduces placental blood flow during uterine contractions, resulting in fetal hypoxemia. Maternal hypotension does not result in early FHR decelerations nor is it associated with fetal arrhythmias. Spontaneous rupture of membranes is not a result of maternal hypotension.)

21. Which information related to a prolonged deceleration is important for the labor nurse to understand? a. Prolonged decelerations present a continuing pattern of benign decelerations that do not require intervention. b. Prolonged decelerations constitute a baseline change when they last longer than 5 minutes. c. A disruption to the fetal oxygen supply causes prolonged decelerations. d. Prolonged decelerations require the customary fetal monitoring by the nurse.

C (Prolonged decelerations are caused by a disruption in the fetal oxygen supply. They usually begin as a reflex response to hypoxia. If the disruption continues, then the fetal cardiac tissue, itself, will become hypoxic, resulting in direct myocardial depression of the FHR. Prolonged decelerations can be caused by prolonged cord compression, uteroplacental insufficiency, or perhaps sustained head compression. Prolonged decelerations lasting longer than 10 minutes are considered a baseline change that may require intervention. A prolonged deceleration is a visually apparent decrease (may be either gradual or abrupt) in the FHR of at least 15 beats per minute below the baseline and lasting longer than 2 minutes but shorter than 10 minutes. Nurses should immediately notify the physician or nurse-midwife and initiate appropriate treatment of abnormal patterns when they see prolonged decelerations.)

1. Which statement by the client will assist the nurse in determining whether she is in true labor as opposed to false labor? a. I passed some thick, pink mucus when I urinated this morning. b. My bag of waters just broke. c. The contractions in my uterus are getting stronger and closer together. d. My baby dropped, and I have to urinate more frequently now.

C (Regular, strong contractions with the presence of cervical change indicate that the woman is experiencing true labor. The loss of the mucous plug (operculum) often occurs during the first stage of labor or before the onset of labor, but it is not the indicator of true labor. Spontaneous rupture of membranes (ROM) often occurs during the first stage of labor, but it is not the indicator of true labor. The presenting part of the fetus typically becomes engaged in the pelvis at the onset of labor, but this is not the indicator of true labor.)

20. A woman in labor has just received an epidural block. What is the most important nursing intervention at this time? a. Limit parenteral fluids. b. Monitor the fetus for possible tachycardia. c. Monitor the maternal blood pressure for possible hypotension. d. Monitor the maternal pulse for possible bradycardia.

C (The most important nursing intervention for a woman who has received an epidural block is for the nurse to monitor the maternal blood pressure frequently for signs of hypotension. IV fluids are increased for a woman receiving an epidural to prevent hypotension. The nurse also observes for signs of fetal bradycardia and monitors for signs of maternal tachycardia, secondary to hypotension.)

4. The uterine contractions of a woman early in the active phase of labor are assessed by an internal uterine pressure catheter (IUPC). The uterine contractions occur every 3 to 4 minutes and last an average of 55 to 60 seconds. They are becoming more regular and are moderate to strong. Based on this information, what would a prudent nurse do next? a. Immediately notify the womans primary health care provider. b. Prepare to administer an oxytocic to stimulate uterine activity. c. Document the findings because they reflect the expected contraction pattern for the active phase of labor. d. Prepare the woman for the onset of the second stage of labor.

C (The nurse is responsible for monitoring the uterine contractions to ascertain whether they are powerful and frequent enough to accomplish the work of expelling the fetus and the placenta. In addition, the nurse documents these findings in the clients medical record. This labor pattern indicates that the client is in the active phase of the first stage of labor. Nothing indicates a need to notify the primary health care provider at this time. Oxytocin augmentation is not needed for this labor pattern; this contraction pattern indicates that the woman is in active labor. Her contractions will eventually become stronger, last longer, and come closer together during the transition phase of the first stage of labor. The transition phase precedes the second stage of labor, or delivery of the fetus.)

5. Which action is correct when palpation is used to assess the characteristics and pattern of uterine contractions? a. Placing the hand on the abdomen below the umbilicus and palpating uterine tone with the fingertips b. Determining the frequency by timing from the end of one contraction to the end of the next contraction c. Evaluating the intensity by pressing the fingertips into the uterine fundus d. Assessing uterine contractions every 30 minutes throughout the first stage of labor

C (The nurse or primary health care provider may assess uterine activity by palpating the fundal section of the uterus using the fingertips. Many women may experience labor pain in the lower segment of the uterus, which may be unrelated to the firmness of the contraction detectable in the uterine fundus. The frequency of uterine contractions is determined by palpating from the beginning of one contraction to the beginning of the next contraction. Assessment of uterine activity is performed in intervals based on the stage of labor. As labor progresses, this assessment is performed more frequently.)

23. A woman has requested an epidural for her pain. She is 5 cm dilated and 100% effaced. The baby is in a vertex position and is engaged. The nurse increases the womans IV fluid for a preprocedural bolus. The nurse reviews her laboratory values and notes that the womans hemoglobin is 12 g/dl, hematocrit is 38%, platelets are 67,000, and white blood cells (WBCs) are 12,000/mm3. Which factor would contraindicate an epidural for this woman? a. She is too far dilated. b. She is anemic. c. She has thrombocytopenia. d. She is septic.

C (The platelet count indicates a coagulopathy, specifically, thrombocytopenia (low platelets), which is a contraindication to epidural analgesia and anesthesia. Typically, epidural analgesia and anesthesia are used in the laboring woman when a regular labor pattern has been achieved, as evidenced by progressive cervical change. The laboratory values show that the womans hemoglobin and hematocrit levels are in the normal range and show a slight increase in the WBC count that is not uncommon in laboring women.)

The nurse is caring for an infant after a forceps-assisted birth. Which feature does the nurse attribute to a forceps-assisted birth? A. Erythematous skin B. Blotchy or mottled skin C. Edema and ecchymosis D. Cyanotic discoloration

C. Edema and ecchymosis An infant who had a forceps-assisted birth is likely to have edema of the face and ecchymosis, or bruising. It is normal for the term infant to have erythematous, or red skin, for a few hours after birth. The skin gradually fades to its normal color. The skin often appears blotchy or mottled, especially over the extremities in aterm infant. It is normal for the infant to have acrocyanosis, or cyanotic discoloration of the hands and feet.The discoloration is caused by vasomotor instability and capillary stasis and may appear intermittently over the first 7 to 10 days, especially with exposure to cold.

15. What is a distinct advantage of external EFM? a. The ultrasound transducer can accurately measure short-term variability and beat-to-beat changes in the FHR. b. The tocotransducer can measure and record the frequency, regularity, intensity, and approximate duration of uterine contractions. c. The tocotransducer is especially valuable for measuring uterine activity during the first stage of labor. d. Once correctly applied by the nurse, the transducer need not be repositioned even when the woman changes positions.

C (The tocotransducer is valuable for measuring uterine activity during the first stage of labor and is especially true when the membranes are intact. Short-term variability and beat-to-beat changes cannot be measured with this technology. The tocotransducer cannot measure and record the intensity of uterine contractions. The transducer must be repositioned when the woman or the fetus changes position.)

5. The nurse providing care for a high-risk laboring woman is alert for late FHR decelerations. Which clinical finding might be the cause for these late decelerations? a. Altered cerebral blood flow b. Umbilical cord compression c. Uteroplacental insufficiency d. Meconium fluid

C (Uteroplacental insufficiency results in late FHR decelerations. Altered fetal cerebral blood flow results in early FHR decelerations. Umbilical cord compression results in variable FHR decelerations. Meconium-stained fluid may or may not produce changes in the FHR, depending on the gestational age of the fetus and whether other causative factors associated with fetal distress are present.)

A client who is breastfeeding has been diagnosed with gonorrhea. Which treatment plan should be instituted? a. Amoxicillin 500 mg three times a day for 1 week. b. Benzathine Penicillin G 2.4 million units one injection c. Amoxicillin 500 mg three times a day for 7 days and ceftriaxone 250 mg IM injection d. Ceftriaxone 250 mg IM injectio

C Dual therapy of amoxicillin and ceftriaxone can be used for treatment of gonorrhea and empirical treatment of chlamydia. Amoxicillin and ceftriaxone can both be part of the treatment plan for gonorrhea but the client should be treated empirically for chlamydia as well. Benzathine penicillin is indicated for treatment of syphilis in the lactating client. p. 148

What is the most common causative agent of ophthalmia neonatorum? a. Neisseria gonorrhea b. Human papillomavirus c. Chlamydia trachomatis d. Gardnerella and Mobiluncus

C Ophthalmia neonatorum is a conjunctivitis that occurs in newborns. It is most commonly caused by Chlamydia trachomatis. Therefore, it is most commonly seen in the neonates born to clients with Chlamydial infection. Neisseria gonorrhea causes gonococcal infections. Human papillomavirus causes condylomata acuminate or genital warts, but not ophthalmia neonatorum. Gardnerella and Mobiluncus are the anaerobic bacteria that cause bacterial vaginosis, but not ophthalmia neonatorum. p. 148

21. What should the nurses next action be if the clients white blood cell (WBC) count is 25,000/mm3 on her second postpartum day? a. Immediately inform the physician. b. Have the laboratory draw blood for reanalysis. c. Recognize that this count is an acceptable range at this point postpartum. d. Immediately begin antibiotic therapy.

C (During the first 10 to 12 days after childbirth, WBC values between 20,000 and 25,000/mm3 are common. Because a WBC count of 25,000/mm3 on her second postpartum day is normal, alerting the physician is not warranted nor is reassessment or antibiotics needed; the WBC count is not elevated.)

20. Which statement, related to the reconditioning of the urinary system after childbirth, should the nurse understand? a. Kidney function returns to normal a few days after birth. b. Diastasis recti abdominis is a common condition that alters the voiding reflex. c. Fluid loss through perspiration and increased urinary output accounts for a weight loss of more than 2 kg during the puerperium. d. With adequate emptying of the bladder, bladder tone is usually restored 2 to 3 weeks after childbirth.

C (Excess fluid loss through other means besides perspiration and increased urinary output occurs as well. Kidney function usually returns to normal in approximately 1 month. Diastasis recti abdominis is the separation of muscles in the abdominal wall and has no effect on the voiding reflex. Bladder tone is usually restored 5 to 7 days after childbirth.)

5. Which hormone remains elevated in the immediate postpartum period of the breastfeeding woman? a. Estrogen b. Progesterone c. Prolactin d. Human placental lactogen

C (Prolactin levels in the blood progressively increase throughout pregnancy. In women who breastfeed, prolactin levels remain elevated into the sixth week after birth. Estrogen levels decrease significantly after expulsion of the placenta, reaching their lowest levels 1 week into the postpartum period. Progesterone levels decrease significantly after expulsion of the placenta, reaching their lowest levels 1 week into the postpartum period. Human placental lactogen levels dramatically decrease after expulsion of the placenta.)

6. Two days ago a woman gave birth to a full-term infant. Last night she awakened several times to urinate and noted that her gown and bedding were wet from profuse diaphoresis. Which physiologic alteration is the cause for the diaphoresis and diuresis that this client is experiencing? a. Elevated temperature caused by postpartum infection b. Increased basal metabolic rate after giving birth c. Loss of increased blood volume associated with pregnancy d. Increased venous pressure in the lower extremities

C (Within 12 hours of birth, women begin to lose the excess tissue fluid that has accumulated during pregnancy. One mechanism for reducing these retained fluids is the profuse diaphoresis that often occurs, especially at night, for the first 2 or 3 days after childbirth. Postpartal diuresis is another mechanism by which the body rids itself of excess fluid. An elevated temperature causes chills and possibly dehydration, not diaphoresis and diuresis. Diaphoresis and diuresis are sometimes referred to as reversal of the water metabolism of pregnancy, not as the basal metabolic rate. Postpartal diuresis may be caused by the removal of increased venous pressure in the lower extremities.)

2. Changes in blood volume after childbirth depend on several factors such as blood loss during childbirth and the amount of extravascular water (physiologic edema) mobilized and excreted. What amount of blood loss does the postpartum nurse anticipate? (Select all that apply.) a. 100 ml b. 250 ml or less c. 300 to 500 ml d. 500 to 1000 ml e. 1500 ml or greater

C, D (The average blood loss for a vaginal birth of a single fetus ranges from 300 to 500 ml (10% of blood volume). The typical blood loss for women who gave birth by cesarean is 500 to 1000 ml (15% to 30% of blood volume). During the first few days after childbirth, the plasma volume further decreases as a result of diuresis. Pregnancy-induced hypervolemia (i.e., an increase in blood volume of at least 35%) allows most women to tolerate considerable blood loss during childbirth.)

5. The baseline FHR is the average rate during a 10-minute segment. Changes in FHR are categorized as periodic or episodic. These patterns include both accelerations and decelerations. The labor nurse is evaluating the clients most recent 10-minute segment on the monitor strip and notes a late deceleration. Which is likely to have caused this change? (Select all that apply.) a. Spontaneous fetal movement b. Compression of the fetal head c. Placental abruption d. Cord around the babys neck e. Maternal supine hypotension

C, E (Late decelerations are almost always caused by uteroplacental insufficiency. Insufficiency is caused by uterine tachysystole, maternal hypotension, epidural or spinal anesthesia, IUGR, intraamniotic infection, or placental abruption. Spontaneous fetal movement, vaginal examination, fetal scalp stimulation, fetal reaction to external sounds, uterine contractions, fundal pressure, and abdominal palpation are all likely to cause accelerations of the FHR. Early decelerations are most often the result of fetal head compression and may be caused by uterine contractions, fundal pressure, vaginal examination, and the placement of an internal electrode. A variable deceleration is likely caused by umbilical cord compression, which may happen when the umbilical cord is around the babys neck, arm, leg, or other body part or when a short cord, a knot in the cord, or a prolapsed cord is present.)

A 27-year-old pregnant woman had a preconceptual body mass index (BMI) of 19. The nurse knows that this woman's total recommended weight gain during pregnancy should be at least: a.20 kg (44 lb) b.16 kg (35 lb) c.12.5 kg (27.5 lb) d.10 kg (22 lb)

C. 12.5 kg (27.5 lb)

In most healthy newborns, blood glucose levels stabilize at _________ mg/dl during the first hours after birth. A. 30 to 40 B. 40 to 50 C. 50 to 60 D. 60 to 70

C. 50 to 60 In most healthy term newborns, blood glucose levels stabilize at 50 to 60 mg/dl during the first several hours after birth. A blood sugar level less than 40 mg/dl in the newborn is considered abnormal and warrants intervention. This infant can display classic symptoms of jitteriness, lethargy, apnea, feeding problems, or seizures. By the third day of life, the blood glucose levels should be approximately 60 to 70 mg/dl.

Upon assessment, the nurse finds that the infant has a sunken abdomen, bowel sounds heard in the chest, nasal flaring, and grunting. What clinical condition does the nurse suspect the infant has based on these findings? A. Epispadias. B. A ruptured viscus. C. A diaphragmatic hernia. D. Hirschsprung's disease.

C. A diaphragmatic hernia. The infant has a sunken abdomen (scaphoid) with bowel sounds heard in the chest. Nasal flaring and grunting indicate respiratory distress. All these symptoms indicate a diaphragmatic hernia. Epispadias, ruptured viscus, and Hirschsprung's disease are not associated with these symptoms. Epispadias is the condition where the urethral opening is located in an abnormal position. Ruptured viscus is due to abdominal distentionat birth, caused by abdominal wall defects. Hirschsprung's disease is a congenital disorder that involves an imperforate anus.

Which meal provides the most absorbable iron? a.Toasted cheese sandwich, celery sticks, tomato slices, and a grape drink b.Oatmeal, whole wheat toast, jelly, and low-fat milk c.Black bean soup, wheat crackers, ambrosia (orange sections, coconut, and pecans), and prunes d.Red beans and rice, cornbread, mixed greens, and decaffeinated tea

C. Black bean soup, wheat crackers, ambrosia (orange sections, coconut, and pecans), and prunes

While assessing a 1-week-old infant, the nurse observes that the newborn has apnea, lethargy, jitteriness, and feeding problems. What could be the possible reason for the infant's symptoms? A. Heart rate of 120 beats/min. B. Body temperature of 99.5° F. C. Blood glucose level of 38 mg/dl. D. Blood pressure (BP) of 80/40 mm Hg

C. Blood glucose level of 38 mg/dl. Apnea, lethargy, jitteriness, and feeding problems are the symptoms of hypoglycemia (less than 40 mg/dl of blood glucose levels). Therefore the infant with a blood glucose level of 38 mg/dl (hypoglycemia) would have these symptoms. A body temperature of 99.5° F, heart rate of 120beats/min and BP of 80/40 mm Hg are normal values for a newborn, and are not associated with the infant's manifestations.

The nurse observes that the lips, feet, and palms of a newborn are pale blue even 48 hours after birth. What can the nurse suspect from this observation about the newborn's clinical condition? A. Acrocyanosis. B. Polycythemia. C. Central cyanosis. D. Transient tachypnea.

C. Central cyanosis. When pale blue discoloration of the lips, feet, and palms of the newborn persists for more than 24 hours after birth, it is referred to as central cyanosis. Central cyanosis can be the result of an inadequate supply of oxygen to the alveoli, poor perfusion of the lungs that inhibits gas exchange, or cardiac dysfunction. Because central cyanosis is a late sign of distress, newborns usually have significant hypoxemia when cyanosis appears. Transient tachypnea is a condition in which the newborn has difficulty breathing due to the obstruction of the nasal passage. If the newborn has polycythemia, the newborn's face would have a dark red complexion, but the newborn would not have pale blue lips, feet, and palms. Acrocyanosis is a condition in which the infant shows bluish discoloration of the hands and feet for about 24 hours after birth. Since the newborn in this scenario shows bluish discoloration 48 hours after birth, it indicates that the infant has central cyanosis and not acrocyanosis.

The nurse is caring for a neonate in the nursery. What behavior in the neonate does the nurse recognize as thermogenesis? A. Starts shivering incessantly B. Assumes position of extension C. Cries and appears restless D. Develops pallor and seizures

C. Cries and appears restless Thermogenesis is the process by which the neonate tries to generate heat in response to cold. The neonate increases muscle activity by crying and being restless in a quest to stay warm.The shivering mechanism is used to produce heat in adults; however, this mechanism is rarely operable in the newborn unless there is prolonged exposure to cold. The neonate assumes a position of flexion, not extension, to conserve heat. This position reduces the amount of body surface exposed to the environment. The neonate with hyperthermia may develop pallor and seizures due to neurologic injury.

Pregnant adolescents are at high risk for _______________ due to lower body mass indexes (BMIs) and "fad" dieting. a.Obesity b.Gestational diabetes c.Low-birth-weight babies d.High-birth-weight babies

C. Low-birth-weight babies

The nurse is caring for a baby who is 4 weeks old. The nurse finds that the newborn is breathing through the mouth. What does the nurse expect to be the most likely clinical condition for this observation? A. Hypoxemia. B. Cardiac disorder. C. Nasal obstruction. D. Laryngeal obstruction.

C. Nasal obstruction. Newborns are generally nose breathers. After 3 weeks of age, newborns develop a reflex response that allows them to use their mouths for breathing at times of nasal obstruction. If the newborn has hypoxemia, the infant would breathe deeply through nose and not through the mouth. Mouth breathing in infants is a normal finding and does not indicate a cardiac problem. If the infant has laryngeal obstruction, the infant would be unable to breathe. This is a life-threatening condition.

The nurse clamps the umbilical cord of a preterm infant after 3 minutes of birth. What would be the possible effect in the newborn associated with this action? A. Epispadias B. Polydactyly C. Polycythemia D. Hyperbilirubinemia

C. Polycythemia Clamping the umbilical cord after 2 minutes of birth refers to delayed clamping. Delayed clamping of the cord results in polycythemia (greater plasma volume) and improves hematocrit and iron status. Polycythemia is more commonly observed in preterm infants than in term infants. Epispadias is an abnormal position of the urethral opening and is a congenital abnormality that is not associated with the umbilical cord. Polydactyly is the presence of extra digits on the extremities and is a congenital abnormality. Hyperbilirubinemia (increased bilirubin) is not related to delayed clamping of the umbilical cord, though it may lead to jaundice in the infant.

You are a nurse working in a prenatal clinic. Yourpatient, an 18-year-old woman, is in her 10th gesta-tional week. She wants to know when her baby's heartwill start to beat. You inform her that it usually begins to beat around: A. The 2nd week of gestation B. The 3rd week of gestation C. The 4th week of gestation D. The 5th week of gestation

C. The 4th week of gestation

The nurse is caring for a healthy caucasian neonate who was born at 37 weeks of gestation. What does the nurse find while performing the skin assessment of the newborn immediately after the birth? A. Bluish-black areas on the body B. Desquamation of the epidermis C. Vernix caseosa covering the body D. Dark red-colored swellings on the body

C. Vernix caseosa covering the body After 35 weeks of gestation the newborn's body gets covered with vernix caseosa, which resembles a cheesy white substance and is fused with the epidermis of the skin. It is formed to protect the fetus' skin from the contents of the uterus. Postdate fetuses lose the vernix caseosa and the epidermis may become desquamated. Desquamation (peeling) of the skin occurs a few days after birth. Mongolian spots are characterized by bluish-black pigmentation of the skin, and are generally observed in Mediterranean, Latin American, Asian, or African newborns. They are not usually observed in European newborns. A nevus vascularis is a common type of capillary hemangioma, in which the infant develops dark red-colored swellings. Because the child is healthy, the nurse will not find dark red-colored lesions on the body.

With regard to weight gain during pregnancy, maternity nurses should know that: a.In this case, the woman's height is not a factor in determining her target weight b.Obese women may have their health concerns, but their risk of giving birth to a child with major congenital defects is the same as with normal-weight women c.Women with inadequate weight gain have an increased risk of delivering an infant with intrauterine growth restriction (IUGR) d.Greater than expected weight gain during pregnancy is almost always due to old-fashioned overeating

C. Women with inadequate weight gain have an increased risk of delivering an infant with intrauterine growth restriction (IUGR) .

12. A client asks the nurse when her ovaries will begin working again. Which explanation by the nurse is most accurate? a. Almost 75% of women who do not breastfeed resume menstruating within 1 month after birth. b. Ovulation occurs slightly earlier for breastfeeding women. c. Because of menstruation and ovulation schedules, contraception considerations can be postponed until after the puerperium. d. The first menstrual flow after childbirth usually is heavier than normal.

D (The first flow is heavier, but within three or four cycles, the flow is back to normal. Ovulation can occur within the first month, but for 70% of nonlactating women, it returns in approximately 3 months. Women who are breastfeeding take longer to resume ovulation. Because many women ovulate before their first postpartum menstrual period, contraceptive options need to be discussed early in the puerperium.)

After giving birth to a healthy infant boy, a primiparous woman, 16, is admitted to the postpartum unit. An appropriate nursing diagnosis for her at this time is "risk for impaired parenting related to deficient knowledge of newborn care." In planning for the woman's discharge, what should the nurse be certain to include in the plan of care? A. Tell the woman how to feed and bathe her infant B. Give the woman written information on bathing her infant C. Advise the woman that all mothers instinctively know how to care for their infants D. Provide time for the woman to bathe her infant after she views an infant bath demonstration

D A. Incorrect: Although verbalizing how to care for the infant is a form of client education, it is not the most developmentally appropriate teaching for a teenage mother. B. Incorrect: Although providing written information is useful, it is not the most developmentally appropriate teaching for a teenage mother. C. Incorrect: This statement is inappropriate; it is belittling and false. D. Correct: Having the mother demonstrate infant care is a valuable method of assessing the client's understanding of her newly acquired knowledge, especially in this age group, because she may inadvertently neglect her child. p. 625

After birth, a crying infant may be soothed by being held in a position in which the newborn can hear the mother's heartbeat. This phenomenon is known as: A. Entrainment B. Reciprocity C. Synchrony D. Biorhythmicity

D A. Incorrect: Entrainment is the movement of newborns in time to the structure of adult speech. B. Incorrect: Reciprocity is body movement or behavior that gives cues to the person's desires. These take several weeks to develop with a new baby. C. Incorrect: Synchrony is the fit between the infant's behavioral cues and the parent's responses. D. Correct: The newborn is in rhythm with the mother. The infant develops a personal biorhythm with the parents' help over time. p. 617

7. Which term best describes the interval between the birth of the newborn and the return of the reproductive organs to their normal nonpregnant state? a. Involutionary period because of what happens to the uterus b. Lochia period because of the nature of the vaginal discharge c. Mini-tri period because it lasts only 3 to 6 weeks d. Puerperium, or fourth trimester of pregnancy

D (The puerperium, also called the fourth trimester or the postpartum period of pregnancy, is the final period of pregnancy and lasts approximately 3 to 6 weeks. Involution marks the end of the puerperium. Lochia refers to the various vaginal discharges during the puerperium.)

2. A woman who is pregnant for the first time is dilated 3 cm and having contractions every 5 minutes. She is groaning and perspiring excessively; she states that she did not attend childbirth classes. What is the optimal intervention for the nurse to provide at this time? a. Notify the womans health care provider. b. Administer the prescribed narcotic analgesic. c. Assure her that her labor will be over soon. d. Assist her with simple breathing and relaxation instructions.

D (By reducing tension and stress, both focusing and relaxation techniques will allow the woman in labor to rest and conserve energy for the task of giving birth. For those who have had no preparation, instruction in simple breathing and relaxation can be given in early labor and is often successful. The nurse can independently perform many functions in labor and birth, such as teaching and support. Pain medication may be an option for this client. However, the initial response of the nurse should include teaching the client about her options. The length of labor varies among individuals, but the first stage of labor is the longest. At 3 cm of dilation with contractions every 5 minutes, this woman has a significant amount of labor yet to experience.)

1. An 18-year-old pregnant woman, gravida 1, para 0, is admitted to the labor and birth unit with moderate contractions every 5 minutes that last 40 seconds. The client states, My contractions are so strong, I dont know what to do. Before making a plan of care, what should the nurses first action be? a. Assess for fetal well-being. b. Encourage the woman to lie on her side. c. Disturb the woman as little as possible. d. Recognize that pain is personalized for each individual.

D (Each womans pain during childbirth is unique and is influenced by a variety of physiologic, psychosocial, and environmental factors. A critical issue for the nurse is how support can make a difference in the pain of the woman during labor and birth. This scenario includes no information that would indicate fetal distress or a logical reason to be overly concerned about the well-being of the fetus. The left lateral position is used to alleviate fetal distress, not maternal stress. The nurse has an obligation to provide physical, emotional, and psychosocial care and support to the laboring woman. This client clearly needs support.)

29. What physiologic change occurs as the result of increasing the infusion rate of nonadditive IV fluids? a. Maintaining normal maternal temperature b. Preventing normal maternal hypoglycemia c. Increasing the oxygen-carrying capacity of the maternal blood d. Expanding maternal blood volume

D (Filling the mothers vascular system increases the amount of blood available to perfuse the placenta and may correct hypotension. Increasing fluid volume may alter the maternal temperature only if she is dehydrated. Most IV fluids for laboring women are isotonic and do not provide extra glucose. Oxygen-carrying capacity is increased by adding more red blood cells.)

11. Which statement correctly describes the effects of various pain factors? a. Higher prostaglandin levels arising from dysmenorrhea can blunt the pain of childbirth. b. Upright positions in labor increase the pain factor because they cause greater fatigue. c. Women who move around trying different positions experience more pain. d. Levels of pain-mitigating beta-endorphins are higher during a spontaneous, natural childbirth.

D (Higher endorphin levels help women tolerate pain and reduce anxiety and irritability. Higher prostaglandin levels correspond to more severe labor pains. Upright positions in labor usually result in improved comfort and less pain. Moving freely to find more comfortable positions is important for reducing pain and muscle tension.)

7. Which FHR finding is the most concerning to the nurse who is providing care to a laboring client? a. Accelerations with fetal movement b. Early decelerations c. Average FHR of 126 beats per minute d. Late decelerations

D (Late decelerations are caused by uteroplacental insufficiency and are associated with fetal hypoxemia. Late FHR decelerations are considered ominous if they are persistent and left uncorrected. Accelerations with fetal movement are an indication of fetal well-being. Early decelerations in the FHR are associated with head compression as the fetus descends into the maternal pelvic outlet; they are not generally a concern during normal labor. An FHR finding of 126 beats per minute is normal and not a concern.)

14. The nurse is using intermittent auscultation (IA) to locate the fetal heartbeat. Which statement regarding this method of surveillance is accurate? a. The nurse can be expected to cover only two or three clients when IA is the primary method of fetal assessment. b. The best course is to use the descriptive terms associated with EFM when documenting results. c. If the heartbeat cannot be immediately found, then a shift must be made to EFM. d. Ultrasound can be used to find the FHR and to reassure the mother if the initial difficulty is a factor.

D (Locating fetal heartbeats often takes time. Mothers can be verbally reassured and reassured by viewing the ultrasound pictures if that device is used to help locate the heartbeat. When used as the primary method of fetal assessment, IA requires a nurse-to-client ratio of one to one. Documentation should use only terms that can be numerically defined; the usual visual descriptions of EFM are inappropriate.)

When attempting to communicate with a client who speaks a different language, which action is the most appropriate? a. Promptly and positively respond to project authority. b. Never use a family member as an interpreter. c. Talk to the interpreter to avoid confusing the client. d. Provide as much privacy as possible.

D

Which resource best describes a health care service representing the tertiary level of prevention? a. Stress management seminars b. Childbirth education classes for single parents c. BSE pamphlet and teaching d. Premenstrual syndrome (PMS) support group

D

While working in the prenatal clinic, nurses care for a very diverse group of clients. Which cultural factor related to health is most likely to drive acceptance of planned interventions? a. Educational achievement b. Income level c. Subcultural group d. Individual beliefs

D

11. The perinatal nurse realizes that an FHR that is tachycardic, bradycardic, has late decelerations, or loss of variability is nonreassuring and is associated with which condition? a. Hypotension b. Cord compression c. Maternal drug use d. Hypoxemia

D (Nonreassuring FHR patterns are associated with fetal hypoxemia. Fetal bradycardia may be associated with maternal hypotension. Variable FHR decelerations are associated with cord compression. Maternal drug use is associated with fetal tachycardia.)

21. A woman in the active phase of the first stage of labor is using a shallow pattern of breathing, which is approximately twice the normal adult breathing rate. She starts to complain about feeling lightheaded and dizzy and states that her fingers are tingling. Which intervention should the nurse immediately initiate? a. Contact the womans physician. b. Tell the woman to slow her pace of her breathing. c. Administer oxygen via a mask or nasal cannula. d. Help her breathe into a paper bag.

D (This woman is experiencing the side effects of hyperventilation, which include the symptoms of lightheadedness, dizziness, tingling of the fingers, or circumoral numbness. Having the woman breathe into a paper bag held tightly around her mouth and nose may eliminate respiratory alkalosis and enable her to rebreathe carbon dioxide and replace the bicarbonate ion.)

During the childbearing experience, which behavior might the nurse expect from an African-American client? a. Seeking prenatal care early in her pregnancy b. Avoiding self-treatment of pregnancy-related discomfort c. Requesting liver in the postpartum period to prevent anemia d. Arriving at the hospital in advanced labor

D

What information should the nurse be aware of regarding telephonic nursing care such as warm lines? a. Were developed as a reaction to impersonal telephonic nursing care b. Were set up to take complaints concerning health maintenance organizations (HMOs) c. Are the second option when 9-1-1 hotlines are busy d. Refer to community service telephone lines designed to provide new parents with encouragement and basic information

D

What is a limitation of a home postpartum visit? a. Distractions limit the nurses ability to teach. b. Identified problems cannot be resolved in the home setting. c. Necessary items for infant care are not available. d. Home visits to different families may require the nurse to travel a great distance.

D

Nursing activities that promote parent-infant attachment are many and varied. One activity that should not be overlooked is the management of the environment. While providing routine mother-baby care, the nurse should ensure that: A. The baby is able to return to the nursery at night so that the new mother can sleep B. Routine times for care are established to reassure the parents C. The father should be encouraged to go home at night to prepare for mother-baby discharge D. An environment that fosters as much privacy as possible should be created

D A. Incorrect: Once the baby has demonstrated adjustment to extrauterine life (either in the mother's room or the transitional nursery), all care should be provided in one location. This important principle of family-centered maternity care fosters attachment by giving parents the opportunity to learn about their infant 24 hours a day. One nurse should provide care to both mother and baby in this couplet care or rooming-in model. It is not necessary for the baby to return to the nursery at night. In fact, the mother will sleep better with the infant close by. B. Incorrect: Care should be individualized to meet the parents' needs, not the routines of the staff. Teaching goals should be developed in collaboration with the parents. C. Incorrect: The father or other significant other should be permitted to sleep in the room with the mother. The maternity unit should develop policies that allow for the presence of significant others as much as the new mother desires. D. Correct: Care providers need to knock before gaining entry. Nursing care activities should be grouped. pp. 614-616

In follow-up appointments or visits with parents and their new baby, it is useful if the nurse can identify infant behaviors that can either facilitate or inhibit attachment. What is an inhibiting behavior? A. The infant cries only when hungry or wet. B. The infant's activity is somewhat predictable. C. The infant clings to the parents. D. The infant seeks attention from any adult in the room.

D A. Incorrect: These are facilitating behaviors. Facilitating and inhibiting behaviors are behaviors that build or discourage bonding (attitudes); they do not reflect any value judgments on what might be healthy or unhealthy. B. Incorrect: These are facilitating behaviors. Facilitating and inhibiting behaviors are behaviors that build or discourage bonding (attitudes); they do not reflect any value judgments on what might be healthy or unhealthy. C. Incorrect: These are facilitating behaviors. Facilitating and inhibiting behaviors are behaviors that build or discourage bonding (attitudes); they do not reflect any value judgments on what might be healthy or unhealthy. D. Correct: Parents want to be the focus of the infant's existence, just as the infant is the focus of their existence. Facilitating and inhibiting behaviors are behaviors that build or discourage bonding (attitudes); they do not reflect any value judgments on what might be healthy or unhealthy. p. 613

A 30-year-old multiparous woman has a boy who is 2 1/2 years old and now an infant girl. She tells the nurse, "I don't know how I'll ever manage both children when I get home." Which suggestion would best help this woman alleviate sibling rivalry? A. Tell the older child that he is a big boy now and should love his new sister B. Let the older child stay with his grandparents for the first 6 weeks to allow him to adjust to the newborn C. Ask friends and relatives not to bring gifts to the older sibling because you do not want to spoil him D. Realize that the regression in habits and behaviors in the older child is a typical reaction and that he needs extra love and attention at this time

D A. Incorrect: This strategy is a negative approach to facilitating sibling acceptance of the new infant. B. Incorrect: Reactions of siblings may result from temporary separation from the mother. Removing the older child from the home when the new infant arrives may enhance negative behaviors from the older child caused by separation from the mother. C. Incorrect: Providing small gifts from the infant to the older child is a strategy for facilitating sibling acceptance of the new infant. D. Correct: The older child may regress in habits or behaviors (e.g., toileting and sleep habits) as a method of seeking attention. Parents need to distribute their attention in an equitable manner. p. 629

19. Which definition of an acceleration in the fetal heart rate (FHR) is accurate? a. FHR accelerations are indications of fetal well-being when they are periodic. b. FHR accelerations are greater and longer in preterm gestations. c. FHR accelerations are usually observed with breech presentations when they are episodic. d. An acceleration in the FHR presents a visually apparent and abrupt peak.

D (Acceleration of the FHR is defined as a visually apparent abrupt (only to peak 30 seconds) increase in the FHR above the baseline rate. Periodic accelerations occur with uterine contractions and are usually observed with breech presentations. Episodic accelerations occur during fetal movement and are indications of fetal well-being. Preterm accelerations peak at 10 beats per minute above the baseline and last for at least 10 seconds.)

28. The obstetric nurse is preparing the client for an emergency cesarean birth, with no time to administer spinal anesthesia. The nurse is aware of and prepared for the greatest risk of administering general anesthesia to the client. What is this risk? a. Respiratory depression b. Uterine relaxation c. Inadequate muscle relaxation d. Aspiration of stomach contents

D (Aspiration of acidic gastric contents with possible airway obstruction is a potentially fatal complication of general anesthesia. Respirations can be altered during general anesthesia, and the anesthesiologist will take precautions to maintain proper oxygenation. Uterine relaxation can occur with some anesthesia but can be monitored and prevented. Inadequate muscle relaxation can be improved with medication.)

2. When a nulliparous woman telephones the hospital to report that she is in labor, what guidance should the nurse provide or information should the nurse obtain? a. Tell the woman to stay home until her membranes rupture. b. Emphasize that food and fluid intake should stop. c. Arrange for the woman to come to the hospital for labor evaluation. d. Ask the woman to describe why she believes she is in labor.

D (Assessment begins at the first contact with the woman, whether by telephone or in person. By asking the woman to describe her signs and symptoms, the nurse can begin her assessment and gather data. The initial nursing activity should be to gather data about the womans status. The amniotic membranes may or may not spontaneously rupture during labor. The client may be instructed to stay home until the uterine contractions become strong and regular. Before instructing the woman to come to the hospital, the nurse should initiate her assessment during the telephone interview. After this assessment has been made, the nurse may want to discuss the appropriate oral intake for early labor, such as light foods or clear liquids, depending on the preference of the client or her primary health care provider.)

7. A client is in early labor, and her nurse is discussing the pain relief options she is considering. The client states that she wants an epidural no matter what! What is the nurses best response? a. Ill make sure you get your epidural. b. You may only have an epidural if your physician allows it. c. You may only have an epidural if you are going to deliver vaginally. d. The type of analgesia or anesthesia used is determined, in part, by the stage of your labor and the method of birth.

D (To avoid suppressing the progress of labor, pharmacologic measures for pain relief are generally not implemented until labor has advanced to the active phase of the first stage and the cervix is dilated approximately 4 to 5 cm. A plan of care is developed for each woman that addresses her particular clinical and nursing problems. The nurse collaborates with the primary health care provider and the laboring woman in selecting features of care relevant to the woman and her family. The decision whether to use an epidural to relieve labor pain is multifactorial. The nurse should not make a blanket statement guaranteeing the client one pharmacologic option over another until a complete history and physical examination has been obtained. A physicians order is required for pharmacologic options for pain management. However, expressing this requirement is not the nurses best response. An epidural is an effective pharmacologic pain management option for many laboring women. It can also be used for anesthesia control if the woman undergoes an operative delivery.)

26. The nurse is evaluating the EFM tracing of the client who is in active labor. Suddenly, the FHR drops from its baseline of 125 down to 80 beats per minute. The mother is repositioned, and the nurse provides oxygen, increased IV fluids, and performs a vaginal examination. The cervix has not changed. Five minutes have passed, and the FHR remains in the 80s. What additional nursing measures should the nurse take next? a. Call for help. b. Insert a Foley catheter. c. Start administering Pitocin. d. Immediately notify the care provider.

D (To relieve an FHR deceleration, the nurse can reposition the mother, increase IV fluids, and provide oxygen. If oxytocin is infusing, then it should be discontinued. If the FHR does not resolve, then the primary care provider should be immediately notified. Inserting a Foley catheter is an inappropriate nursing action. If the FHR were to continue in a nonreassuring pattern, then a cesarean section could be warranted, which would require a Foley catheter. However, the physician must make that determination. The administration of Pitocin may place additional stress on the fetus.)

15. Maternity nurses often have to answer questions about the many, sometimes unusual, ways people have tried to make the birthing experience more comfortable. Which information regarding nonpharmacologic pain relief is accurate? a. Music supplied by the support person has to be discouraged because it could disturb others or upset the hospital routine. b. Women in labor can benefit from sitting in a bathtub, but they must limit immersion to no longer than 15 minutes at a time. c. Effleurage is permissible, but counterpressure is almost always counterproductive. d. Electrodes attached to either side of the spine to provide high-intensity electrical impulses facilitate the release of endorphins.

D (Transcutaneous electrical nerve stimulation (TENS) may help and is most useful for lower back pain that occurs during the first stage of labor. Music may be very helpful for reducing tension and certainly can be accommodated by the hospital. Women can stay in a bath as long as they want, although repeated baths with breaks might be more effective than one long bath. Counterpressure can help the woman cope with lower back pain.)

Which infection is prevented with the Cervarix and Gardasil vaccines? a. Bacterial vaginosis (BV) b. Vulvovaginal candidiasis (VVC) c. Pelvic inflammatory disease (PID) d. Human papillomavirus (HPV)

D The Cervarix and Gardasil vaccines are recommended for children aged 9 to 26. The vaccines are effective in protecting against human papillomavirus (HPV) infections, which can lead to genital warts and cancers. Bacterial vaginosis (BV) is treated with oral metronidazole (Flagyl); there are not any preventive vaccines for this. Exogenous lactobacillus (found in dairy products or powder, tablet, capsule, or suppository supplements) and garlic have been suggested for prevention and treatment of vulvovaginal candidiasis. Vaccination is not effective in preventing pelvic inflammatory disease (PID). It can be prevented only by practicing risk reduction measures and using barrier methods. p. 156

18. The nurse is providing instruction to the newly delivered client regarding postbirth uterine and vaginal discharge, called lochia. Which statement is the most appropriate? a. Lochia is similar to a light menstrual period for the first 6 to 12 hours. b. It is usually greater after cesarean births. c. Lochia will usually decrease with ambulation and breastfeeding. d. It should smell like normal menstrual flow unless an infection is present.

D (An offensive odor usually indicates an infection. Lochia flow should approximate a heavy menstrual period for the first 2 hours and then steadily decrease. Less lochia is usually seen after cesarean births and usually increases with ambulation and breastfeeding.)

9. A client is concerned that her breasts are engorged and uncomfortable. What is the nurses explanation for this physiologic change? a. Overproduction of colostrum b. Accumulation of milk in the lactiferous ducts and glands c. Hyperplasia of mammary tissue d. Congestion of veins and lymphatic vessels

D (Breast engorgement is caused by the temporary congestion of veins and lymphatic vessels. An overproduction of colostrum, an accumulation of milk in the lactiferous ducts and glands, and hyperplasia of mammary tissue do not cause breast engorgement.)

14. Which condition, not uncommon in pregnancy, is likely to require careful medical assessment during the puerperium? a. Varicosities of the legs b. Carpal tunnel syndrome c. Periodic numbness and tingling of the fingers d. Headaches

D (Headaches in the postpartum period can have a number of causes, some of which deserve medical attention. Total or nearly total regression of varicosities is expected after childbirth. Carpal tunnel syndrome is relieved in childbirth when the compression on the median nerve is lessened. Periodic numbness of the fingers usually disappears after childbirth unless carrying the baby aggravates the condition.)

4. A woman gave birth to a healthy infant boy 5 days ago. What type of lochia does the nurse expect to find when evaluating this client? a. Lochia rubra b. Lochia sangra c. Lochia alba d. Lochia serosa

D (Lochia serosa, which consists of blood, serum, leukocytes, and tissue debris, generally occurs around day 3 or 4 after childbirth. Lochia rubra consists of blood and decidual and trophoblastic debris. The flow generally lasts 3 to 4 days and pales, becoming pink or brown. Lochia sangra is not a real term. Lochia alba occurs in most women after day 10 and can continue up to 6 weeks after childbirth.)

16. Pelvic floor exercises, also known as Kegel exercises, will help to strengthen the perineal muscles and encourage healing after childbirth. The nurse requests the client to repeat back instructions for this exercise. Which response by the client indicates successful learning? a. I contract my thighs, buttocks, and abdomen. b. I perform 10 of these exercises every day. c. I stand while practicing this new exercise routine. d. I pretend that I am trying to stop the flow of urine in midstream.

D (The woman can pretend that she is attempting to stop the passing of gas or the flow of urine midstream, which will replicate the sensation of the muscles drawing upward and inward. Each contraction should be as intense as possible without contracting the abdomen, buttocks, or thighs. Guidelines suggest that these exercises should be performed 24 to 100 times per day. Positive results are shown with a minimum of 24 to 45 repetitions per day. The best position to learn Kegel exercises is to lie supine with the knees bent. A secondary position is on the hands and knees.)

While caring for an infant, which method should the nurse adapt to prevent heat loss due to evaporation? A. Wrap the infant in a cloth. B. Place the infant in a warm crib. C. Place the crib away from the windows. D. Dry the infant immediately after the bath.

D. Dry the infant immediately after the bath. The infant loses heat due to the evaporation of moisture from the body. To prevent heat loss in the infant, the nurse should immediately dry the infant after the bath. Vasoconstriction of the skin may lead to acrocyanosis. Wrapping the infant in a cloth protects the infant from exposure to cold and prevents pneumonia. The newborn is placed in the warm crib to minimize heat loss caused by conduction. Placing the crib away from the windows helps prevent heat loss due to radiation.

The nurse is examining the external genitalia of a female infant. What finding must the nurse report? A. Slight bloody spotting B. Presence of hymenal tag C. Mucoid vaginal discharge D. Fecal discharge from vagina

D. Fecal discharge from vagina Fecal discharge from the vagina indicates a rectovaginal fistula. This finding should be reported to the neonatal nurse practitioner for further evaluation. Slight bloody spotting, or pseudomenstruation, is normal and need not be reported. Nearly all female infants are born with hymenal tags. The nurse must report the absence of such tags, which can indicate vaginal agenesis. The presence of mucoid vaginal discharge is a normal finding. The discharge occurs due to an increase in estrogen during pregnancy followed by a drop after birth.

A mother reports that her baby's skin always appears flushed. What does the nurse suspect to be the reason for this condition in the infant? A. Loss of water and fluids B. Increased acid production C. Increased heat production D. Loss of heat from the body

D. Loss of heat from the body Loss of heat from the infant's body dilates the skin vessels, therefore causing the skin to appear flushed. Loss of water and fluids from the body occurs to prevent overheating complications, such as cerebral damage from dehydration or even heat stroke and death. Increased production of acids result in increased bilirubin levels which leads to jaundice. If the infant has increased heat production in the body because of sepsis, vessels in the skin are constricted and the skin appears pale.

The nurse is caring for a patient who is breastfeeding a term newborn. What does the nurse teach the patient about how normal stool should appear on the fourth day after birth? A. Greenish-black stool B. Greenish-brown stool C. Pale yellow to brown stool D. Pasty yellow to golden stool

D. Pasty yellow to golden stool The breastfed newborn passes pale yellow to golden stool on the fourth day. The stool is pasty in consistency with an odor similar to sour milk. The newborn's first stool is meconium, which is viscous in its consistency and greenish-black in color. It contains amniotic fluid, and its constituents include intestinal secretions, shed mucosal cells, and blood. The newborn passes transitional stools by the third day after initiation of feeding. Transitional stools appear greenish-brown to yellowish-brown in color. They are thinner and less viscous than meconium and may contain milk curds. By the fourth day, the newborn fed on formula milk passes pale yellow to light brown stool with a foul odor.

A first-time mother at 18 weeks of gestation is in for her regularly scheduled prenatal visit. The client tells the nurse that she is afraid that she is going into premature labor because she is beginning to have regular contractions. The nurse explains that this is the Braxton Hicks sign and teaches the client that this type of contraction: a.Is painless b.Increases with walking c.Causes cervical dilation d.Impedes oxygen flow to the fetus

a.Is painless

In the past, factors to determine whether a woman was likely to have a high risk pregnancy were evaluated primarily from a medical point of view. A broader, more comprehensive approach to high risk pregnancy has been adopted. There are now four categories based on threats to the health of the woman and the outcome of pregnancy. Which of the options listed here is not included as a category? Biophysical Psychosocial Geographic Environmental

Geographic Rationale: The fourth category is correctly referred to as the sociodemographic risk category. The factors stem from the mother and her family. Ethnicity may be one of the risks to pregnancy; however, it is not the only factor in this category. Low income, lack of prenatal care, age, parity, and marital status are included. Biophysical is one of the broad categories used for determining risk. It includes genetic considerations, nutritional status, and medical and obstetric disorders. Psychosocial risks include smoking, caffeine, drugs, alcohol, and psychologic status. All of these adverse lifestyles can have a negative effect on the health of the mother or fetus. Environmental risks are those that can affect fertility and fetal development. They include infections, chemicals, radiation, pesticides, illicit drugs, and industrial pollutants.

A nurse providing care for the antepartum woman should understand that the contraction stress test (CST): Sometimes uses vibroacoustic stimulation. Is an invasive test; however, contractions are stimulated. Is considered to have a negative result if no late decelerations are observed with the contractions. Is more effective than nonstress test (NST) if the membranes have already been ruptured.

Is considered to have a negative result if no late decelerations are observed with the contractions Rationale: No late decelerations indicate a positive CST result. Vibroacoustic stimulation is sometimes used with NST. CST is invasive if stimulation is performed by IV oxytocin but not if by nipple stimulation. CST is contraindicated if the membranes have ruptured..

What is an appropriate indicator for performing a contraction stress test? Increased fetal movement and small for gestational age Maternal diabetes mellitus and postmaturity Adolescent pregnancy and poor prenatal care History of preterm labor and intrauterine growth restriction

Maternal diabetes mellitus and postmaturity Rationale: Decreased fetal movement is an indicator for performing a contraction stress test; the size (small for gestational age) is not an indicator. Although adolescent pregnancy and poor prenatal care are risk factors for poor fetal outcomes, they are not indicators for performing a contraction stress test. Intrauterine growth restriction is an indicator; history of a previous stillbirth, not preterm labor, is another indicator.

d

Nurses need to understand the basic definitions and incidence data regarding PPH. Which statement regarding this condition is most accurate? a. PPH is easy to recognize early; after all, the woman is bleeding. b. Traditionally, it takes more than 1000 ml of blood after vaginal birth and 2500 ml after cesarean birth to define the condition as PPH. c. If anything, nurses and physicians tend to overestimate the amount of blood loss. d. Traditionally, PPH has been classified as early PPH or late PPH with respect to birth.

A patient has undergone an amniocentesis for evaluation of fetal well-being. Which intervention would be included in the nurse's plan of care after the procedure? (Select all that apply.) Perform ultrasound to determine fetal positioning. Observe the patient for possible uterine contractions. Administer RhoGAM to the patient if she is Rh negative. Perform a minicatheterization to obtain a urine specimen to assess for bleeding.

Observe the patient for possible uterine contractions. Administer RhoGAM to the patient if she is Rh negative. Rationale: Ultrasound is used prior to the procedure as a visualization aid to assist with insertion of transabdominal needle. There is no need to assess the urine for bleeding as this is not considered to be a typical presentation or complication.

The mucous plug that forms in the endocervical canal is called the a.Operculum b.Leukorrhea c.Funic souffle d.Ballottement

a.Operculum

Most women with uncomplicated pregnancies can use the nurse as their primary source for nutritional information. There are times when the nurse or midwife should refer a client to a registered dietitian for in-depth nutritional counseling. These pregnant women include those with: a.Preexisting or gestational illness such as diabetes b.Ethnic or cultural food patterns c.Obesity d.Vegetarian diets

Preexisting or gestational illness such as diabetes Ethnic or cultural food patterns Obesity Vegetarian diets

26. As the United States and Canada continue to become more culturally diverse, recognizing a wide range of varying cultural beliefs and practices is increasingly important for the nursing staff. A client is from which country if she requests to have the babys father in attendance? a. Mexico b. China c. Iran d. India

a (Hispanic women routinely have fathers and female relatives in attendance during the second stage of labor. The father of the baby is expected to provide encouragement, support, and reassurance that all will be well. In China, fathers are usually not present. The side-lying position is preferred for labor and birth because it is believed that this will reduce trauma to the infant. In China, the client has a stoic response to pain. In Iran, the father will not be present. Female support persons and female health care providers are preferred. For many, a male caregiver is unacceptable. In India, the father is usually not present, but female relatives are usually in attendance. Natural childbirth methods are preferred.)

24. In recovery, if a woman is asked to either raise her legs (knees extended) off the bed or flex her knees, and then place her feet flat on the bed and raise her buttocks well off the bed, the purpose of this exercise is to assess what? a. Recovery from epidural or spinal anesthesia b. Hidden bleeding underneath her c. Flexibility d. Whether the woman is a candidate to go home after 6 hours

a (If the numb or prickly sensations are gone from her legs after these movements, then she has likely recovered from the epidural or spinal anesthesia. Assessing the client for bleeding beneath her buttocks before discharge from the recovery is always important; however, she should be rolled to her side for this assessment. The nurse is not required to assess the woman for flexibility. This assessment is performed to evaluate whether the client has recovered from spinal anesthesia, not to determine if she is a candidate for early discharge.)

17. Which component of the physical examination are Leopolds maneuvers unable to determine? a. Gender of the fetus b. Number of fetuses c. Fetal lie and attitude d. Degree of the presenting parts descent into the pelvis

a (Leopolds maneuvers help identify the number of fetuses, the fetal lie and attitude, and the degree of descent of the presenting part into the pelvis. The gender of the fetus cannot be determined by performing Leopolds maneuvers.)

30. When assessing a multiparous woman who has just given birth to an 8-pound boy, the nurse notes that the womans fundus is firm and has become globular in shape. A gush of dark red blood comes from her vagina. What is the nurses assessment of the situation? a. The placenta has separated. b. A cervical tear occurred during the birth. c. The woman is beginning to hemorrhage. d. Clots have formed in the upper uterine segment.

a (Placental separation is indicated by a firmly contracting uterus, a change in the uterus from a discoid to a globular ovoid shape, a sudden gush of dark red blood from the introitus, an apparent lengthening of the umbilical cord, and a finding of vaginal fullness. Cervical tears that do not extend to the vagina result in minimal blood loss. Signs of hemorrhage are a boggy uterus, bright red vaginal bleeding, alterations in vital signs, pallor, lightheadedness, restlessness, decreased urinary output, and alteration in the level of consciousness. If clots have formed in the upper uterine segment, then the nurse would expect to find the uterus boggy and displaced to the side.)

20. Which technique is an adequate means of controlling the birth of the fetal head during delivery in a vertex presentation? a. Ritgen maneuver b. Fundal pressure c. Lithotomy position d. De Lee apparatus

a (The Ritgen maneuver extends the head during the actual birth and protects the perineum. Gentle, steady pressure against the fundus of the uterus facilitates vaginal birth. The lithotomy position has been commonly used in Western cultures, partly because it is convenient for the health care provider. The De Lee apparatus is used to suction fluid from the infants mouth.)

13. What is the most critical nursing action in caring for the newborn immediately after the birth? a. Keeping the airway clear b. Fostering parent-newborn attachment c. Drying the newborn and wrapping the infant in a blanket d. Administering eye drops and vitamin K

a (The care given immediately after the birth focuses on assessing and stabilizing the newborn. Although fostering parent-newborn attachment is an important task for the nurse, it is not the most critical nursing action in caring for the newborn immediately after birth. The care given immediately after birth focuses on assessing and stabilizing the newborn. The nursing activities are (in order of importance) to maintain a patent airway, to support respiratory effort, and to prevent cold stress by drying the newborn and covering him or her with a warmed blanket or placing the newborn under a radiant warmer. After the newborn has been stabilized, the nurse assesses the newborns physical condition, weighs and measures the newborn, administers prophylactic eye ointment and a vitamin K injection, affixes an identification bracelet, wraps the newborn in warm blankets, and then gives the newborn to the partner or to the mother of the infant.)

9. Under which circumstance should the nurse assist the laboring woman into a hands-and-knees position? a. Occiput of the fetus is in a posterior position. b. Fetus is at or above the ischial spines. c. Fetus is in a vertex presentation. d. Membranes have ruptured.

a (The hands-and-knees position is effective in helping to rotate the fetus from a posterior to an anterior position. Many women experience the irresistible urge to push when the fetus is at the level of the ischial spines. In some cases, this urge is felt before the woman is fully dilated. The woman should be instructed not to push until complete cervical dilation has occurred. No one position is correct for childbirth. The two most common positions assumed by women are the sitting and side-lying positions. The woman may be encouraged into a hands-and-knees position if the umbilical cord prolapsed when the membranes ruptured.)

38. A laboring woman is reclining in the supine position. What is the most appropriate nursing action at this time? a. Ask her to turn to one side. b. Elevate her feet and legs. c. Take her blood pressure. d. Determine whether fetal tachycardia is present.

a (The womans supine position may cause the heavy uterus to compress her inferior vena cava, thus reducing blood return to her heart and reducing placental blood flow. Elevating her legs will not relieve the pressure from the inferior vena cava. If the woman is allowed to stay in the supine position and blood flow to the placental is reduced significantly, then fetal tachycardia may occur. The most appropriate nursing action is to prevent this from occurring by turning the woman to her side. Blood pressure readings may be obtained when the client is in the appropriate and safest position.)

19. The nurse should be aware of which information related to a womans intake and output during labor? a. Traditionally, restricting the laboring woman to clear liquids and ice chips is being challenged because regional anesthesia is used more often than general anesthesia. b. Intravenous (IV) fluids are usually necessary to ensure that the laboring woman stays hydrated. c. Routine use of an enema empties the rectum and is very helpful for producing a clean, clear delivery. d. When a nulliparous woman experiences the urge to defecate, it often means birth will quickly follow.

a (Women are awake with regional anesthesia and are able to protect their own airway, which reduces the worry over aspiration. Routine IV fluids during labor are unlikely to be beneficial and may be harmful. The routine use of an enema is, at best, ineffective and may be harmful. Having the urge to defecate followed by the birth of her fetus is true for a multiparous woman but not for a nulliparous woman.)

Which nutritional recommendation about fluids is accurate? a.A woman's daily intake should be six to eight glasses of water, milk, and/or juice. b.Coffee should be limited to no more than 2 cups, but tea and cocoa can be consumed without worry. c.Of the artificial sweeteners, only aspartame has not been associated with any maternity health concerns. d.Water with fluoride is especially encouraged because it reduces the child's risk of tooth decay.

a. A woman's daily intake should be six to eight glasses of water, milk, and/or juice.

A woman at 10 weeks of gestation who is seen in the prenatal clinic with presumptive signs and symptoms of pregnancy likely has: a.Amenorrhea b.Positive pregnancy test c.Chadwick sign d.Hegar sign

a. Amenorrhea

The diagnosis of pregnancy is based on which positive signs of pregnancy? Choose all that apply. a.Identification of fetal heartbeat b.Palpation of fetal outline c.Visualization of the fetus d.Verification of fetal movement e.Positive human chorionic gonadotropin (hCG) test

a. Identification of fetal heartbeat c.Visualization of the fetus d. Verification of fetal movement

A woman is in her seventh month of pregnancy. She has been complaining of nasal congestion and occasional epistaxis. The nurse suspects that: a.This is a normal respiratory change in pregnancy caused by elevated levels of estrogen b.This is an abnormal cardiovascular change and the nosebleeds are an ominous sign c.The woman is a victim of domestic violence and is being hit in the face by her partner d.The woman has been using cocaine intranasally

a. This is a normal respiratory change in pregnancy caused by elevated levels of estrogen

To help a woman reduce the severity of nausea caused by morning sickness, the nurse might suggest that she: a.Try a tart food or drink, such as lemonade, or salty foods, such as potato chips b.Drink plenty of fluids early in the day c.Brush her teeth immediately after eating d.Never snack before bedtime

a. Try a tart food or drink, such as lemonade, or salty foods, such as potato chips

A pregnant woman tells her nurse that she is worried about the blotchy, brownish coloring over her cheeks, nose, and forehead. The nurse can reassure her that this is a normal condition related to hormonal change, commonly called the mask of pregnancy or, scientifically: a.Chloasma b.Linea nigra c.Striae gravidarum d.Palmar erythema

a.Chloasma

Appendicitis may be difficult to diagnose in pregnancy because the appendix is: a.Displaced upward and laterally, high and to the right b.Displaced upward and laterally, high and to the left c.Deep at McBurney's point d.Displaced downward and laterally, low and to the right

a.Displaced upward and laterally, high and to the right

40. What is the primary rationale for the thorough drying of the infant immediately after birth? a. Stimulates crying and lung expansion b. Removes maternal blood from the skin surface c. Reduces heat loss from evaporation d. Increases blood supply to the hands and feet

c (Infants are wet with amniotic fluid and blood at birth, and this accelerates evaporative heat loss. The primary purpose of drying the infant is to prevent heat loss. Although rubbing the infant stimulates crying, it is not the main reason for drying the infant. This process does not remove all the maternal blood.)

In order to reassure and educate pregnant clients about the functioning of their kidneys in eliminating waste products, maternity nurses should be aware that: a.Increased urinary output makes pregnant women less susceptible to urinary infection b.Increased bladder sensitivity and then compression of the bladder by the enlarging uterus result in the urge to urinate even if the bladder is almost empty c.Renal (kidney) function is more efficient when the woman assumes a supine position d.Using diuretics during pregnancy can help keep kidney function regular

b. Increased bladder sensitivity and then compression of the bladder by the enlarging uterus result in the urge to urinate even if the bladder is almost empty

22. Which statement concerning the third stage of labor is correct? a. The placenta eventually detaches itself from a flaccid uterus. b. An expectant or active approach to managing this stage of labor reduces the risk of complications. c. It is important that the dark, roughened maternal surface of the placenta appears before the shiny fetal surface. d. The major risk for women during the third stage is a rapid heart rate.

b (Active management facilitates placental separation and expulsion, reducing the risk of complications. The placenta cannot detach itself from a flaccid (relaxed) uterus. Which surface of the placenta comes out first is not clinically important. The major risk for women during the third stage of labor is postpartum hemorrhaging.)

36. A woman who is gravida 3 para 2 arrives on the intrapartum unit. What is the most important nursing assessment at this time? a. Contraction pattern, amount of discomfort, and pregnancy history b. FHR, maternal vital signs, and the womans nearness to birth c. Identification of ruptured membranes, womans gravida and para, and her support person d. Last food intake, when labor began, and cultural practices the couple desires

b (All options describe relevant intrapartum nursing assessments; however, this focused assessment has a priority. If the maternal and fetal conditions are normal and birth is not imminent, then other assessments can be performed in an unhurried manner; these include: gravida, para, support person, pregnancy history, pain assessment, last food intake, and cultural practices.)

34. Under which circumstance would it be unnecessary for the nurse to perform a vaginal examination? a. Admission to the hospital at the start of labor b. When accelerations of the FHR are noted c. On maternal perception of perineal pressure or the urge to bear down d. When membranes rupture

b (An accelerated FHR is a positive sign; therefore, a vaginal examination would not be necessary. A vaginal examination should be performed when the woman is admitted to the hospital, when she perceives perineal pressure or the urge to bear down, when her membranes rupture, when a significant change in her uterine activity has occurred, or when variable decelerations of the FHR are noted.)

32. A woman who is 39 weeks pregnant expresses fear about her impending labor and how she will manage. What is the nurses ideal response? a. Dont worry about it. Youll do fine. b. Its normal to be anxious about labor. Lets discuss what makes you afraid. c. Labor is scary to think about, but the actual experience isnt. d. You can have an epidural. You wont feel anything.

b (Its normal to be anxious about labor. Lets discuss what makes you afraid is a statement that allows the woman to share her concerns with the nurse and is a therapeutic communication tool. Dont worry about it. Youll do fine negates the womans fears and is not therapeutic. Labor is scary to think about, but the actual experience isnt negates the womans fears and offers a false sense of security. To suggest that every woman can have an epidural is untrue. A number of criteria must be met before an epidural is considered. Furthermore, many women still experience the feeling of pressure with an epidural.)

14. What is the rationale for the administration of an oxytocic (e.g., Pitocin, Methergine) after expulsion of the placenta? a. To relieve pain b. To stimulate uterine contraction c. To prevent infection d. To facilitate rest and relaxation

b (Oxytocics stimulate uterine contractions, which reduce blood loss after the third stage of labor. Oxytocics are not used to treat pain, do not prevent infection, and do not facilitate rest and relaxation.)

31. After an emergency birth, the nurse encourages the woman to breastfeed her newborn. What is the primary purpose of this activity? a. To facilitate maternal-newborn interaction b. To stimulate the uterus to contract c. To prevent neonatal hypoglycemia d. To initiate the lactation cycle

b (Stimulation of the nipples through breastfeeding or manual stimulation causes the release of oxytocin and prevents maternal hemorrhage. Breastfeeding facilitates maternal-newborn interaction, but it is not the primary reason a woman is encouraged to breastfeed after an emergency birth. The primary intervention for preventing neonatal hypoglycemia is thermoregulation. Cold stress can result in hypoglycemia. The woman is encouraged to breastfeed after an emergency birth to stimulate the release of oxytocin, which prevents hemorrhaging. Breastfeeding is encouraged to initiate the lactation cycle, but it is not the primary reason for this activity after an emergency birth.)

15. Which description of the phases of the first stage of labor is most accurate? a. Latent: mild, regular contractions; no dilation; bloody show b. Active: moderate, regular contractions; 4 to 7 cm dilation c. Lull: no contractions; dilation stable d. Transition: very strong but irregular contractions; 8 to 10 cm dilation

b (The active phase is characterized by moderate and regular contractions, 4 to 7 cm dilation, and duration of 3 to 6 hours. The latent phase is characterized by mild-to-moderate and irregular contractions, dilation up to 3 cm, brownish-to-pale pink mucus, and duration of 6 to 8 hours. No official lull phase exists in the first stage. The transition phase is characterized by strong to very strong and regular contractions, 8 to 10 cm dilation, and duration of 20 to 40 minutes.)

25. A woman who has a history of sexual abuse may have a number of traumatic memories triggered during labor. She may fight the labor process and react with pain or anger. The nurse can implement a number of care measures to help her client view the childbirth experience in a positive manner. Which intervention is key for the nurse to use while providing care? a. Tell the client to relax and that it wont hurt much. b. Limit the number of procedures that invade her body. c. Reassure the client that, as the nurse, you know what is best. d. Allow unlimited care providers to be with the client.

b (The number of invasive procedures such as vaginal examinations, internal monitoring, and IV therapy should be limited as much as possible. The nurse should always avoid words and phrases that may result in the clients recalling the phrases of her abuser (i.e., Relax, this wont hurt or Just open your legs). The womans sense of control should be maintained at all times. The nurse should explain procedures at the clients pace and wait for permission to proceed. Protecting the clients environment by providing privacy and limiting the number of staff who observe the client will help to make her feel safe.)

Maternal nutritional status is an especially significant factor of the many that influence the outcome of pregnancy because: a.It is very difficult to adjust because of people's ingrained eating habits b.It is an important preventive measure for a variety of problems c.Women love obsessing about their weight and diets d.A woman's preconception weight becomes irrelevant

b. It is an important preventive measure for a variety of problems

12. Through a vaginal examination, the nurse determines that a woman is 4 cm dilated. The external fetal monitor shows uterine contractions every to 4 minutes. The nurse reports this as what stage of labor? a. First stage, latent phase b. First stage, active phase c. First stage, transition phase d. Second stage, latent phase

b (This maternal progress indicates that the woman is in the active phase of the first stage of labor. During the latent phase of the first stage of labor, the expected maternal progress is 0 to 3 cm dilation with contractions every 5 to 30 minutes. During the transition phase of the first stage of labor, the expected maternal progress is 8 to 10 cm dilation with contractions every 2 to 3 minutes. During the latent phase of the second stage of labor, the woman is completely dilated and experiences a restful period of laboring down.)

A pregnant woman is infected with human immunodeficiency virus (HIV), with a viral load of more than 400 copies/ml. It is therefore recommended that she have a caesarean birth at 38 weeks of gestation. On assessment at 36 weeks, the nurse finds that the patient has a viral load of 800 copies/ml and ruptured membranes, and the labor is progressing rapidly. What is the primary health care provider likely to order? a. Scalp pH sampling b. Immediate vaginal birth c. Immediate caesarean birth d. Use of fetal scalp electrode

b. Ruptured membranes and a rapidly progressing labor indicate that the client is ready for vaginal birth. If the viral load in the client is less than 1000 copies/ml at 36 weeks, the patient can be prepared for a vaginal birth. Scalp pH sampling is avoided, because it may result in inoculation of the virus into the fetus. If the viral load is more than 1000 copies/ml, it is recommended that the client have a caesarean birth. The use of a fetal scalp electrode is avoided, because it increases the risk of transmission of infection to the fetus. p. 161

A woman's obstetric history indicates that she is pregnant for the fourth time, and all her children from previous pregnancies are living. One was born at 39 weeks of gestation, twins were born at 34 weeks of gestation, and another child was born at 35 weeks of gestation. What is her gravidity and parity using the GTPAL system? a.3-1-1-1-3 b.4-1-2-0-4 c.3-0-3-0-3 d.4-2-1-0-3

b. 4-1-2-0-4

Three servings of milk, yogurt, or cheese plus two servings of meat, poultry, or fish will adequately supply the recommended amount of protein for the pregnant woman. Many clients are concerned about the increased levels of mercury in fish and may be afraid to include this source of nutrients in their diet. Sound advice by the nurse to assist the client in determining which fish is safe to consume includes: a.Canned white tuna is a preferred choice b.Avoid shark, swordfish, and mackerel c.Fish caught in local waterways is the safest d.Salmon and shrimp contain high levels of mercury

b. Avoid shark, swordfish, and mackerel

Assessment of a woman's nutritional status includes a diet history, medication regimen, physical examination, and relevant laboratory tests. A maternity nurse performing such an assessment should be aware that: a.Oral contraceptive use may interfere with the absorption of iron b.Illnesses that have created nutritional deficits, such as phenylketonuria (PKU), may require nutritional care before conception c.The woman's socioeconomic status and educational level are not relevant to her examination; they are the province of the social worker d.The only nutrition-related laboratory test most pregnant women need is testing for diabetes

b. Illnesses that have created nutritional deficits, such as phenylketonuria (PKU), may require nutritional care before conception

A 31-year-old woman believes that she may be pregnant. She took an over-the-counter (OTC) pregnancy test 1 week ago after missing her period; the test was positive. During her assessment interview, the nurse inquires about the woman's last menstrual period (LMP) and asks whether she is taking any medications. The woman states that she takes medicine for epilepsy. She has been under considerable stress lately at work and has not been sleeping well. She also has a history of irregular periods. Her physical examination does not indicate that she is pregnant. She has an ultrasound scan, which reveals that she is not pregnant. What is the most likely cause of the false-positive pregnancy test result? a.She took the pregnancy test too early. b.She takes anticonvulsants. c.She has a fibroid tumor. d.She has been under considerable stress and has a hormone imbalance.

b. She takes anticonvulsants

A woman is at 14 weeks of gestation. The nurse expects to palpate the fundus at which level? a.Not palpable above the symphysis at this time b.Slightly above the symphysis pubis c.At the level of the umbilicus d.Slightly above the umbilicus

b. Slightly above the symphysis pubis

In order to reassure and educate pregnant clients about changes in their cardiovascular system, maternity nurses should be aware that: a.A pregnant woman experiencing disturbed cardiac rhythm, such as sinus arrhythmia, requires close medical and obstetric observation no matter how healthy she otherwise may appear b.Changes in heart size and position and increases in blood volume create auditory changes from 20 weeks to term c.Palpitations are twice as likely to occur in twin gestations d.All of the above changes likely will occur

b.Changes in heart size and position and increases in blood volume create auditory changes from 20 weeks to term

Which time-based description of a stage of development in pregnancy is accurate?a.Viability—22 to 37 weeks since the last menstrual period (assuming a fetal weight greater than 500 g) b.Term—pregnancy from the beginning of week 38 of gestation to the end of week 42 c.Preterm—pregnancy from 20 to 28 weeks d.Postdate—pregnancy that extends beyond 38 weeks

b.Term—pregnancy from the beginning of week 38 of gestation to the end of week 42

In order to reassure and educate pregnant clients about changes in the uterus, nurses should be aware that: a.Lightening occurs near the end of the second trimester as the uterus rises into a different position b.The woman's increased urinary frequency in the first trimester is the result of exaggerated uterine antireflexion caused by softening c.Braxton Hicks contractions become more painful in the third trimester, particularly if the woman tries to exercise d.The uterine souffle is the movement of the fetus

b.The woman's increased urinary frequency in the first trimester is the result of exaggerated uterine antireflexion caused by softening

39. Which nursing assessment indicates that a woman who is in second-stage labor is almost ready to give birth? a. Fetal head is felt at 0 station during vaginal examination. b. Bloody mucous discharge increases. c. Vulva bulges and encircles the fetal head. d. Membranes rupture during a contraction.

c (A bulging vulva that encircles the fetal head describes crowning, which occurs shortly before birth. Birth of the head occurs when the station is +4. A 0 station indicates engagement. Bloody show occurs throughout the labor process and is not an indication of an imminent birth. ROM can occur at any time during the labor process and does not indicate an imminent birth.)

11. Which clinical finding indicates that the client has reached the second stage of labor? a. Amniotic membranes rupture. b. Cervix cannot be felt during a vaginal examination. c. Woman experiences a strong urge to bear down. d. Presenting part of the fetus is below the ischial spines.

c (During the descent phase of the second stage of labor, the woman may experience an increase in the urge to bear down. The ROM has no significance in determining the stage of labor. The second stage of labor begins with full cervical dilation. Many women may have an urge to bear down when the presenting fetal part is below the level of the ischial spines. This urge can occur during the first stage of labor, as early as with 5 cm dilation.)

35. Which description of the phases of the second stage of labor is most accurate? a. Latent phase: Feeling sleepy; fetal station 2+ to 4+; duration of 30 to 45 minutes b. Active phase: Overwhelmingly strong contractions; Ferguson reflux activated; duration of 5 to 15 minutes c. Descent phase: Significant increase in contractions; Ferguson reflux activated; average duration varies d. Transitional phase: Woman laboring down; fetal station 0; duration of 15 minutes

c (The descent phase begins with a significant increase in contractions; the Ferguson reflex is activated, and the duration varies, depending on a number of factors. The latent phase is the lull or laboring down period at the beginning of the second stage and lasts 10 to 30 minutes on average. The second stage of labor has no active phase. The transition phase is the final phase in the second stage of labor; contractions are strong and painful.)

28. The first 1 to 2 hours after birth is sometimes referred to as what? a. Bonding period b. Third stage of labor c. Fourth stage of labor d. Early postpartum period

c (The first 2 hours of the birth are a critical time for the mother and her baby and is often called the fourth stage of labor. Maternal organs undergo their initial readjustment to a nonpregnant state. The third stage of labor lasts from the birth of the baby to the expulsion of the placenta. Bonding will occur over a much longer period, although it may be initiated during the fourth stage of labor.)

10. A nulliparous woman has just begun the latent phase of the second stage of her labor. The nurse should anticipate which behavior? a. A nulliparous woman will experience a strong urge to bear down. b. Perineal bulging will show. c. A nulliparous woman will remain quiet with her eyes closed between contractions. d. The amount of bright red bloody show will increase.

c (The woman is able to relax and close her eyes between contractions as the fetus passively descends. The woman may be very quiet during this phase. During the latent phase of the second stage of labor, the urge to bear down is often absent or only slight during the acme of the contractions. Perineal bulging occurs during the transition phase of the second stage of labor, not at the beginning of the second stage. An increase in bright red bloody show occurs during the descent phase of the second stage of labor.)

23. A 25-year-old gravida 3, para 2 client gave birth to a 9-pound, 7-ounce boy, 4 hours ago after augmentation of labor with oxytocin (Pitocin). She presses her call light, and asks for her nurse right away, stating Im bleeding a lot. What is the most likely cause of postpartum hemorrhaging in this client? a. Retained placental fragments b. Unrepaired vaginal lacerations c. Uterine atony d. Puerperal infection

c (This woman gave birth to a macrosomic infant after oxytocin augmentation. Combined with these risk factors, uterine atony is the most likely cause of bleeding 4 hours after delivery. Although retained placental fragments may cause postpartum hemorrhaging, it is typically detected within the first hour after delivery of the placenta and is not the most likely cause of the hemorrhaging in this woman. Although unrepaired vaginal lacerations may also cause bleeding, it typically occurs in the period immediately after birth. Puerperal infection can cause subinvolution and subsequent bleeding that is, however, typically detected 24 hours postpartum.)

Nutrition is one of the most significant factors in influencing the outcome of a pregnancy. Nutrition is alterable and an important preventive measure for a variety of potential problems such as low birth weight and prematurity. While completing the physical assessment of the pregnant client, the nurse can evaluate the client's nutritional status by observing a number of physical signs. Which sign indicates that the client has unmet nutritional needs? a.Normal heart rate, rhythm, and blood pressure b.Bright, clear, shiny eyes c.Alert, responsive, and good endurance d.Edema, tender calves, and tingling

d. Edema, tender calves, and tingling

Human chorionic gonadotropin (hCG) is an important biochemical marker for pregnancy and therefore the basis for many tests. A maternity nurse should be aware that: a.hCG can be detected as early as 2½ weeks after conception b.The hCG level increases gradually and uniformly throughout pregnancy c.Much lower than normal increases in the level of hCG may indicate a postdate pregnancy d.A higher than normal level of hCG may indicate an ectopic pregnancy or Down syndrome

d.A higher than normal level of hCG may indicate an ectopic pregnancy or Down syndrome

21. Which collection of risk factors will most likely result in damaging lacerations, including episiotomies? a. Dark-skinned woman who has had more than one pregnancy, who is going through prolonged second-stage labor, and who is attended by a midwife b. Reddish-haired mother of two who is going through a breech birth c. Dark-skinned first-time mother who is going through a long labor d. First-time mother with reddish hair whose rapid labor was overseen by an obstetrician

d (Reddish-haired women have tissue that is less distensible than darker-skinned women and therefore may have less efficient healing. First-time mothers are also at greater risk, especially with breech births, long second-stage labors, or rapid labors during which the time for the perineum to stretch is insufficient. The rate of episiotomies is higher when obstetricians rather than midwives attend the births. The woman in the first scenario (a) is at low risk for either damaging lacerations or an episiotomy. She is multiparous, has dark skin, and is being attended by a midwife, who is less likely to perform an episiotomy. Reddish-haired women have tissue that is less distensible than that of darker-skinned women. Consequently, the client in the second scenario (b) is at increased risk for lacerations; however, she has had two previous deliveries, which result in a lower likelihood of an episiotomy. The fact that the woman in the third scenario (c) is experiencing a prolonged labor might increase her risk for lacerations. Fortunately, she is dark skinned, which indicates that her tissue is more distensible than that of fair-skinned women and therefore less susceptible to injury.)

7. The nurse performs a vaginal examination to assess a clients labor progress. Which action should the nurse take next? a. Perform an examination at least once every hour during the active phase of labor. b. Perform the examination with the woman in the supine position. c. Wear two clean gloves for each examination. d. Discuss the findings with the woman and her partner.

d (The nurse should discuss the findings of the vaginal examination with the woman and her partner, as well as report the findings to the primary care provider. A vaginal examination should be performed only when indicated by the status of the woman and her fetus. The woman should be positioned so as to avoid supine hypotension. The examiner should wear a sterile glove while performing a vaginal examination for a laboring woman.)

8. A multiparous woman has been in labor for 8 hours. Her membranes have just ruptured. What is the nurses highest priority in this situation? a. Prepare the woman for imminent birth. b. Notify the womans primary health care provider. c. Document the characteristics of the fluid. d. Assess the fetal heart rate (FHR) and pattern.

d (The umbilical cord may prolapse when the membranes rupture. The FHR and pattern should be closely monitored for several minutes immediately after the ROM to ascertain fetal well-being, and the findings should be documented. The ROM may increase the intensity and frequency of the uterine contractions, but it does not indicate that birth is imminent. The nurse may notify the primary health care provider after ROM occurs and the fetal well-being and response to ROM have been assessed. The nurses priority is to assess fetal well-being. The nurse should document the characteristics of the amniotic fluid, but the initial response is to assess fetal well-being and the response to ROM.)

37. A primigravida at 39 weeks of gestation is observed for 2 hours in the intrapartum unit. The FHR has been normal. Contractions are 5 to 9 minutes apart, 20 to 30 seconds in duration, and of mild intensity. Cervical dilation is 1 to 2 cm and uneffaced (unchanged from admission). Membranes are intact. What disposition would the nurse anticipate? a. Admitted and prepared for a cesarean birth b. Admitted for extended observation c. Discharged home with a sedative d. Discharged home to await the onset of true labor

d (This situation describes a woman with normal assessments who is probably in false labor and will likely not deliver rapidly once true labor begins. No further assessments or observations are indicated; therefore, the client will be discharged along with instructions to return when contractions increase in intensity and frequency. Neither a cesarean birth nor a sedative is required at this time.)

33. Which characteristic of a uterine contraction is not routinely documented? a. Frequency: how often contractions occur b. Intensity: strength of the contraction at its peak c. Resting tone: tension in the uterine muscle d. Appearance: shape and height

d (Uterine contractions are described in terms of frequency, intensity, duration, and resting tone. Appearance is not routinely charted.)

In order to reassure and educate pregnant clients about changes in the cervix, vagina, and position of the fetus, nurses should be aware that: a.Because of a number of changes in the cervix, abnormal Papanicolaou (Pap) tests are much easier to evaluate b.Quickening is a technique of palpating the fetus to engage it in passive movement c.The deepening color of the vaginal mucosa and cervix (Chadwick sign) usually appears in the second trimester or later as the vagina prepares to stretch during labor d.Increased vascularity of the vagina increases sensitivity and may lead to a high degree of arousal, especially in the second trimester

d.Increased vascularity of the vagina increases sensitivity and may lead to a high degree of arousal, especially in the second trimester

Which statement about a condition of pregnancy is accurate? a.Insufficient salivation (ptyalism) is caused by increases in estrogen. b.Acid indigestion (pyrosis) begins early but declines throughout pregnancy. c.Hyperthyroidism often develops (temporarily) because hormone production increases. d.Nausea and vomiting rarely have harmful effects on the fetus and may be beneficial.

d.Nausea and vomiting rarely have harmful effects on the fetus and may be beneficial.


Set pelajaran terkait

Med Surg Chapter 35,37,38 Evolve

View Set

Chapter 13: Preparing your Speech

View Set

Econ: Chapter 2 & 4 Study Questions Exam 1

View Set